LET PROF ED and GEN ED with Answers

August 31, 2017 | Author: Brooklyn Peralta | Category: Test (Assessment), Teachers, Id, Learning, Thought
Share Embed Donate


Short Description

Reviewer Good For You...

Description

PROFESSIONAL EDUCATION (Secondary) MULTIPLE CHOICE 1. Which does Naom Chomsky assert about language learning for children? I. Young children learn and apply grammatical rules and vocabulary as they are exposed to them. II. Begin formal teaching of grammatical rules to children as early as possible. III. Do not require initial formal language teaching for children. A. I and III C. I only B. II only D. I and II 2. Which is/are the sources of man’s intellectual drives, according to Freud A. Id C. Id -ego B. Super ego D. Ego 3. Theft of school equipment like TV, computer, etc. by teenagers in the community itself is becoming a common phenomenon. What does this incident signify? A. Prevalence of poverty in the community. B. Inability of school to hire security guards. C. Deprivation of Filipino schools.

D. Community’s lack of sense of co-ownership. 4. As a teacher, you are reconstructionist, Which among these will be your guiding principle? A. I must teach the child every knowledge, skills and value that he needs for a better future. B. I must teach the child to develop his mental powers to the full. C. I must teach the child so he is assured of heaven. D. I must teach the child that we can never have real knowledge of anything. 5. The concepts of Trust vs. mistrust, autonomy vs. shame & self-doubt, and initiative vs. guilt are most closely related with works of _____. A. Erickson C. Freud B. Piaget D. Jung 6. Student Z does not study at all but when the Licensure Examination for Teachers comes, before he takes the LET, he spends one hour or more praying for a miracle, i.e., to pass the examination. Which attitude towards religion or God is displayed?

A. Religion as fake C. Religion as authentic B. Religion as magic D. Religion as real 7. Teacher B engages her students with information for thorough understanding, for meaning and for competent application. Which principle governs Teacher B’s practice? A Constructivist C. Behaviorist B. Gestalt D. Cognitivist 8. Student B claims: “I cannot see perfection but I long for it. So it must be real. “under which group can he be classified? A. Idealist C. Realist B. Empiricist D. Pragmatist 9. What does extreme authoritarianism in the home reinforce in learners A. Doing things on their own initiative B. Ability to direct themselves C. Dependence on others for directions. D. Creativity in work. 10. You arrive at knowledge by re-thinking of latent ideas. From whom does this thought come? A. Experimentalist C. Idealist B. Realist D. Existentialist

11. Behavior followed by pleasant consequences will be strengthened and will be more likely to occur in the future. Behavior followed by unpleasant consequences will be weakened and will be more likely to be repeated in the future. Which one is explained A. Freud’s psychoanalytic theory B. Thorndike’s law effect C. B.F. Skinner’s Operant conditioning theory D. D. Bandura’s social learning theory 12. Principal B tells her teachers that training in the humanities is most important. To which educational philosophy does he adhere? A. Existentialism C. Progressivis B. Perennialism D. Essentialism 13. All subject in Philippine elementary and secondary schools are expected to be taught using the integrated approach. This came about as a result of implementation of ___________. A. Program for Decentralized Education B. School-Based Management

C. Basic Education Curriculum D. D. Schools First Initiative 14. Principal C shares this thought with the teachers: Subject matter should help students understand and appreciate themselves as unique individual who accept complete responsibility for their thoughts, feelings and actions. “ From which philosophy is this thought based? A. Perennialism C. Existentialism B. Essentialism D. Progressivism 15. Based on Piaget’s theory, what should a teacher provide for children in the concrete operational stage? A. Activities for hypothesis formulation. B. Learning activities that involve problems of classification and ordering. C. Games and other physical activities to develop motor skills. D. Stimulating environment with ample objectives to play with. 16. To come closer to the truth we need to “go back to the things themselves.” This is the advice of the ______. A. Behavioris C. idealist

B. phenomenologists D. pragmatists 17. Researches conducted show that teacher’s expectations of students become self-fulfilling prophecies. What is this phenomenon called? A. Halo effect C. Ripple effect B. Pygmalion effect D. Hawthorne effect 18. Teacher F is convinced that whatever a student performs a desired behavior, provided reinforcement and soon the student will learn to perform the behavior on his own. On which principle is Teacher F’s conviction based? A. Cognitivism C. Behaviorism B. Environmentalism D. Constructivism 19. Teacher U teaches to his pupils that pleasure is not the highest good. Teacher’s teaching is against what philosophy? A. Realism C. Epicureanism B. Hedonism D. Empiricism 20. Studies in the areas of neurosciences discloses that the human brain has limitless capacity. What does this imply

A. Some pupils are admittedly not capable of learning. B. Every pupil has his own native ability and his learning is limited to this native ability. C. Every child is a potential genius. D. Pupil can possibly reach a point where they have learned everything. 21. Availment of the Philippine Education Placement Test for adults and out-of-school youths is in support of the government’s educational program towards _____. A. equitable access C. quality and relevance B. quality D. relevance 22. A mother gives his boy his favorite snack every time the boy cleans up his room. Afterwards, the boy cleaned his room everyday in anticipation of the snack. Which theory is illustrated? A. Associative learning B. Classical conditioning C. Operant conditioning D. Pavlonian conditioning 23. Which group of philosophers maintain the “truth exists in an objective order that is independent of the knower”? A. Idealists C. Existentialists B. Pragmatists D. Realists

24. Under which program were students who were not accommodated in public elementary and secondary schools because of lack of classroom, teachers, and instructional materials, were enrolled in private schools in their respective communities at the government’s expense? A. Government Assistance Program B. Study Now-Pay later C. Educational Service Contract System D. National Scholarship Program 25. Which of the following prepositions is attributed to Plato? A. Truth is relative to a particular time and place. B. Human beings create their own truths. C. Learning is the discovery of truth as latent ideas are brought to consciousness D. Sense perception is the most accurate guide to knowledge. 26. In a treatment for alcoholism, Ramil was made to drink an alcoholic beverage and then made to ingest a drug that produces nausea. Eventually, h nauseated at he sight and smell of alcohol and stopped drinking alcohol. Which theory explains this?

A.Operant conditioning C. Associative learning B. Social learning theory D. Attribution theory 27. In a social studies class, Teacher I present a morally ambiguous situation and asks his students what they would do. On whose theory is Teacher I’s technique based A. Kohlberg C. Piaget B. Bandura D. Bruner 28. Quiz to formative test while periodic is to _________. A. criterion-reference test C. norm-reference test B. summative test D. diagnostic test 29. Your teacher is of the opinion that the world and everything in it are ever changing and so you the skill to cope with change. What is his governing philosophy? A. Idealism C. Experimentalism B. Existentialism D. Realism 30. The search for related literature by accessing several data bases by the use of a telephone line to connect a computer that have database is termed ______. A. compact disc search C. on-line search B. manual search D. computer search

31. In which competency do my students find the greater difficulty? In item with a difficulty index of A. 0. 1 C. 0.5 B. 0. 9 D. 1.0 32. We encounter people whose prayer goes like this: O God, if there is a God; save my soul if I have a soul,” from whom is this prayer? A. Stoic C. Agnostic B. Empiricist D. Skeptic 33. If teacher wants to test student’s ability to organize ideas, which type of test should she formulate? A. Short answer C. Essay B. Technical problem type D. Multiple-choice type 34. Who among the following needs less verbal counseling but needs more concrete operational forms of assistance? The child who ________. A. has mental retardation B. has attention-deficit disorder C. has learning disability D. has conduct disorder 35. Which applies when skewness is zero? A. Mean is greater than median. B. Median is greater than the mean. C. Scores have three modes.

D. Scores are normally distributed 36. Which does NOT belong to the group of alternative learning systems A. Multi-grade grouping C. Graded education B. Multi-age grouping D. Non-graded grouping 37. The first thing to do in constructing a periodic test is for a teacher to ______. A. decide on the number of items for the test B. go back to her instructional objectives C. study the content D. decide on the type of test to construct 38. Which one can enhance the comparability of grades? A. Using common conversation table for translating test scores in to ratings. B. Formulating tests that vary from one teacher to another. C. Allowing individual teachers to determine factors for rating D. Individual teachers giving weights to factors considered for rating. 39. The cultivation of reflective and meditative skills in teaching is an influence of _____.

A. Shintoism C. Confucianism B. Zen Buddhism D. Taoism 40. Teacher Y does normreferenced interpretation of scores. Which of the following does she do? A. She describes group performance in relation to a level of mastery set. B. She uses a specified content as its frame of reference. C. She compares every individual student’s scores with others’ scores. D. She describes what should be their performance. 41. The best way for a guidance counselor to begin to develop study skills and habits in underachieving student would be to ______. A. have these underachieving students observe the study habits of excelling students. B. encourage students to talk study habits from their own experiences C. have them view filmstrips about various study approaches D. give out list of effective study approaches 42. Teacher F wanted to teach the pupils the skills to do cross stitching. He check up quiz was a

written test on the steps of cross stitching. What characteristic of a good test does it lack? A. Scorability C. Objectivity B. Reliability D. Validity 43. For which may you use the direct instruction method? A. Become aware of the pollutants around us. B. Appreciate Milton’s Paradise Lost. C. Use a microscope properly. D. Distinguish war from aggression. 44. In the context on the theory on multiple intelligences, what is one weakness of the paper-pencil test? A. It is not easy to administer. B. It puts the nonlinguistically intelligent at a disadvantage C. It utilizes so much time. D. It lacks reliability. 45. NSAT and NEAT results are interpreted against set mastery level. This means that NSAT and NEAT fall under _________. A. intelligence test C. criterion-referenced test B. aptitude test D. normreferenced test

46. Teacher B uses the direct instruction strategy. Which sequence of steps will she follow? I. Independent practice II. Feedback and correctiveness III. Guided student practice IV. Presenting and structuring V. Reviewing the previous day’s work A. V-IV-III-II-I C. V-II-IV-III-I B. III-II-IV-I-V D. I-V-II-III-IV 47. Which guideline must be observed in the use of prompting to shape the correct performance of your students? A. Use the list intrusive prompt first. B. Use all prompts available. C. Use the most intrusive prompt first. D. Refrain form using prompts. 48. What measure/s of central tendency do/es the number 16 represent in the following data: 14,15,17,16,19,20,16,14,16? A. Mode C. Median B. Mode and medianD. Mean 49. Which holds true to standardized tests? A. They are used for comparative purposes. B. They are administered differently.

C. They are scored according to different standards. D. They are used for assigned grades. 50. Study this group of tests which was administered with the following results, then answer the question. Subject Mean SD Ronnel’s scrore Math 56 10 43 Physics 41 9 31 English 80 16 109 In which subject(s) were the scores most homogenous? A. Math C. Physics B. English D. Physics and Math 51. In the parlance of test construction what does TOS mean? A. Table of Specifics C. Table of Specific Test Items B. Term of Specifications D. Table of Specifications 52. Which is a major advantage of a curriculum-based assessment? A. It is informal nature. B. It connects testing with teaching. C. It tends to focus on anecdotal information on student progress. D. It is based on a normreferenced measurement model.

53. Teacher H gave first-grade class a page with a story in which picture take the place of some words. Which method did she use? A. The whole language approach B. The Spaulding method C. The rebus method D. The language experience approach 54. Out of 3 distracters in a multiple choice test items, namely B, C, and D, no pupil choice D as answer. This implies that D is ________. A. an ineffective distracter B. a vague distracter C. an effective distracter D. a plausible distracter 55. The burnout malady gets worse if a teacher doesn’t intervene to change whatever areas he or she in control. Which one can renew a teacher’s enthusiasm? A. Stick to job C. Judge someone else as wrong B. Initiate changes in jobs D. Engage in self-pity 56. If teacher has to ask more higher-order questions, he has to ask more ____ questions. A. closed C. concept B. Fact

D. convergent

57. What can be said of Peter who obtained a score of P75 in a Grammar objective test? A. His rating is 75 items in the test correctly. B. He answered 75 % of the test items correctly. C. He answered 75. D. He performed better that 5% of his classmate. 58. I drew learners into several content areas and encouraged them to solve a complex question for interdisciplinary teaching. Which strategy did I use? A. Problem-centered learning B. Unit method C. Reading-writing activity D. D. Thematic instruction 59. Which guidance in test construction is NOT observed in this test item “ Jose Rizal wrote ____”. A. The central problem should be packed in the stem. B. There must be only one correct answer. C. Alternatives must grammatical parallelism. D. The alternative must be plausible. 60. To elicit more students’ response, Teacher G. made use of covert responses. Which one did she NOT do?

A. She had the students write their response privately. B. She showed the correct answers on the overhead after the students have written their responses. C. She had the students write their responses privately then called each of them. D. She refrained from judging on the student’s responses. 61. Direct instruction id a facts, rules, and actions as indirect instruction is for _____ ,____, _____. A. hypotheses, verified data and conclusions B. concepts, patterns and abstractions C. concepts, processes and generalizations D. guesses, data and conclusions 62. Which test has broad sampling of topics as strength? A. Objective test C. Essay test B. Short answer test D. Problem test . 63. Teacher T taught a lesson denoting ownership by means of possessives. He first introduced the rule, then gave examples, followed by class exercises, then back to the rule before he moved into second rule. Which presenting technique did he use

A. Combinatorial C. Partwhole B. Comparative D. Sequential 64. In his second item analysis, Teacher H found out that more from the lower group got the test item # 6 correctly. This means that the test item _____. A. has a negative discriminating power. B. has a lower validity. C. has a positive discriminating power. D. has a high reability. 65. Teacher E discussed how electricity flows through wires and what generates the electric charge. Then she gave the students wires, bulbs, switches, and dry cells and told the class to create a circuit that will increase the brightness of each bulb. Which one best describes the approach used? A. It used a taxonomy of basic thinking skills. B. It was constructivist. C. It helped students understand scientific methodology. D. None of the above 66. Teacher W wants to review and check on the lesson of the previous day. Which one will be most reliable?

A. Having students identify difficult homework problems. B. Having students correct each other’s work. C. Sampling the understanding of a few students. D. Explicitly reviewing the taskrelevant information necessary for the day’s lesson. 67. Shown a picture of children in sweaters inside the classrooms, the students were asked this question: In what kind of climate do these children live? This is a thought questions on ______. A. inferring C. applying B. creating D. predicting 68. Study this group of tests which was administered with the following results, then answer the question. Subject Mean SD Ronnel’s scrore Math 56 10 43 Physics 41 9 31 English 80 16 109 In which subject(s) did Ronnel perform best in relation to the group’s performance? A.Physics and Math C. Physics B.English D. Math 69. Which criterion should guide a teacher in the choice of instructional devices?

A. Attractiveness C. Novelty B. Cost D. Appropriateness 70. Study this group of tests which was administered with the following results, then answer the question. Subject Mean SD Ronnel’s scrore Math 56 10 43 Physics 41 9 31 English 80 16 109 In which subject(s) did Ronnel perform poorly in relation to the group’s performance? A. English C. Math B. English and Math D. Physics 71. “What is most likely to happen to our economy when export continuously surpasses import” is a thought question on ______. A. creating C. relating cause-and-effect B. synthesizing D. predicting 72. Which method has been proven to be effective in courses that stress acquisition of knowledge? A. Socratic method C. Indirect instruction B. Cooperative learning

D. Mastery learning 73. Which is the first step in planning an achievement test? A. Define the instructional objective. B. Decide on the length of the test. C. Select the type of test items to use. D. Build a table of specification. 74. Which activity should a teacher have more for his students if he wants to develop logicalmathematical thinking? A. Drama C. Problem solving B. Choral reading D. Storytelling 75. Why should a teacher NOT use direct instruction all the time A. It requires much time. B. It requires use of many supplemental materials. C. It is generally effective only in the teaching of concepts and abstractions. D. It reduces student’s engagement in learning. 76. Which are direct measures of competence? A. Personality tests C. Paperand-pencil test B. Performance tests D. Standardized tests

77. Which is one role play in the pre-school and early childhood years? A. Develops competitive spirit. B. Separates reality from fantasy. C. Increase imagination due to expanding knowledge and emotional range. D. Develops the upper and lower limbs. 78. Teacher A discovered that his pupils are very good in dramatizing. Which tool must have helped him discover his pupils’ strength? A. Portfolio assessment C. Journal entry B. Performance test D. Paper-and-pencil test 79. Teacher M’s pupils are quite weak academically and his lesson is already far behind time table. How should Teacher M proceed with his lesson? A. Experientially C. Logically B. Inductively D. Deductively 81. I want to teach concepts, patterns and abstractions. Which method is most appropriate? A. Indirect instruction C. Direct instruction B. Discovery D. Problem solving

82. If your Licensure Examination for Teacher (LET) items sample adequately the competencies listed in the syllabi, it can be said that LET possesses ______ validity. A. concurrent C. content B. construct D. predictive 83. Read the following then answer the questions: TEACHER: IN WHAT WAYS OTHER THAN THE PERIODIC TABLE MIGHT WE PREDICT THE UNDISCOVERED ELEMENTS? BOBBY: WE COULD GO TO THE MOON AND SEE IF THERE ARE SOME ELEMENTS THERE WE DON’T HAVE. BETTY: WE COULD DIG DOWN TO THE CENTER OF THE EARTH AND SEE IF WE FIND ANY OF THE MISSING ELEMENTS. RICKY: WE COULD STUDY DEBRIS FROM THE METEORITES-IF WE CAN FIND ANY. TEACHER: THOSE ARE ALL GOOD ANSWERS. BUT WHAT IF THOSE EXCURSIONS TO THE MOON, TO THE CENTER OF THE EARTH OR TO FIND METEORITES WERE TOO COSTLY AND TIME CONSUMING? HOW MIGHT WE USE THE ELEMENTS WE ALREADY HAVE HERE ON EARTH TO FIND SOME NEW ONES?

Question: The Teacher questions in the above exchange are examples of ____ questions. A. Fact C. Direct B. Concept D. Closed 84. In Krathwoh’s taxonomy of objectives in the affective, which is most authentic? A.Characterization C. Responding B. Valuing D. Organization 85. “In the light of the facts presented, what is most likely to happen when…?” is a simple thought question on _____. A. inferring C. synthesizing B. generalizing D. justifying 86. The teacher’s first task in the selection of media in teaching is to determine the A. choice of the students B. availability of the media C. objectives of the lesson D. technique to be used 87. In self-directed learning, to what extent should a teacher’s “scaffolding” be? A. To a degree the student needs it. B. None, to force the student to learn by himself

C. To the maximum, in order to extend to the student all the help he needs. D. To the minimum, to spend up development of student’s sense of independence. 88. Which is a form of direct instruction? A. Discovery process C. Programmed instruction B. Problem solving D. Inductive reasoning 89. With synthesizing skills in mind, which has the highest diagnostic value? A. Essay test C. Completion tes B. Performance test D. Multiple choice test 90. How can you exhibit legitimate power on the first day of school? A. By making your students feel they are accepted for who they are. B. By informing them you are allowed to act in loco parents. C. By making them realize the importance of good grades. D. By making them feel you have mastery of subject matter. 91. Based on Edgar Dale’s Cone of Experience, which activity is farthest from the real thing? A. View images C. Watch a demo

B. Attend exhibit D. Hear 92. Which can effectively measure student’s awareness of values? A. Projective techniques C. Likert scales B. Moral dilemma D. Anecdotal record 93. I combined several subject areas in order to focus on a single concept for interdisciplinary teaching. Which strategy/method did I use? A. Problem-entered learning B. Thematic instruction C. Reading–writing activity D. Unit method 94. The test item ”Group the following items according to shape” is a thought test item on _______. A. creating C. classifying B. generating D. comparing 95. For maximum interaction, a teacher ought to avoid _____ questions. A. informational C. leading B. rhetorical D. divergent 96. By what name is Socratic method also known ? A. Mastery learning C. Morrison method B. Indirect instruction D. Questioning Method

97. Which is an appropriate way to make manage off-task behavior? A. Make eye contact. B. Stop your class activity to correct a child who is no longer on task. C. Move closer to the child. D. Redirect a child’s attention to task and check his progress to make sure he is continuing to work 98. Which one can best evaluate student’s attitudinal development? A. Essay test C. Observation B. Portfolio D. Short answer test 99. What should a teacher do for students in his class who are on grade level A. Give them materials on their level and let them work at a pace that is reasonable for them, trying to bring them up to a grade level. B. Give them the same work as the other students, because they will absorb as much as they are capable of. C. Give them the same work as the other students, not much, so that they won’t feel embarrassed. D. Give them work on the level of the other students and work a little above the classmates level to challenge them.

100. With-it-ness, according to Kourin, is one of the characteristics of an effective classroom manager. Which phrase goes with it? A. Have hands that write fast. B. Have eyes on the back of your hands. C. Have a mouth ready to speak. D. Have minds packed with knowledge. 101. On whose philosophy was A.S.Neil Summerhill, one of the most experimental schools based? A. A. Rousseau C. Montessori B. B. Pestalozzi D. John Locke 102. Which Filipino trait works against the shift in teacher’s role from teacher as a fountain of information to teacher as facilitator? A. Authoritativeness C. Hiya B.Authoritarianism D. Pakikisama 103. Teacher A is a teacher of English as Second Language: she uses vocabulary cards, fill-in-the blank sentences, dictation and writing exercises in teaching a lesson about grocery shopping. Based on this information, which of the following is a valid conclusion?

A. The teacher is applying Bloom’s hierarchy of cognitive learning. B. The teacher is teaching variety of ways because not all students learn in the same manner. C. The teacher wants to make her teaching easier by having less talk. D. The teacher is emphasizing reading and writing skills. 104. With specific details in mind, which one has (have) a stronger diagnostic value? A. Multiple choice test B. Non-restricted essay test C. Restricted essay test D. Restricted and non-restricted essay tests 105. Teacher B is a teacher of English as Second Language: she uses vocabulary cards, fill-in-the blank sentences, dictation and writing exercises in teaching a lesson about grocery shopping. Based on this information, which of the following is a valid conclusion? A. The teacher is reinforcing learning by giving the same information in a variety of methods. B. The teacher applying Bloom’s hierarchy of cognitive learning.

C. The teacher is wants to do less talk. D. The teacher is emphasizing listening and speaking skills. 106. In a criterion-referenced testing, what must you do to ensure that your test is fair A. Make all of the questions true or false. B. Ask each student to contribute one question. C. Make twenty questions but ask the students to answer only ten of their choice. D. Use objectives for the unit as guide in your test construction. 107. Read this question: “How will you present the layers of the earth to your class?” This is a question that _________. A. directs B. leads the students to evaluate C. assesses cognition D. probes creative thinking 108. In Krathwoh’s affective domain of objectives, which of the following is the lowest level of effective behavior? A. Valuing C. Responding B. Characterization D. Organization 109. Which is NOT a sound purpose for asking questions?

A. To probe deeper after an answer is given B. To discipline a bully in class C. To remind students of a procedure D. To encourage self-reflection 110. It is not wise to laugh at a two-year old child when he utters bad word because in his stage he is learning to __________. A. considered other’s views B. distinguish right from wrong C. socialize D. distinguish sex differences 111. Research tells that teachers ask mostly content questions. Which of the following terms does NOT refer to content questions? A. Closed C. Concept B. Direct D. Convergent 112. In mastery learning, the definition of an acceptable standard of performance is called a A. behavior C. SMART B. condition D. criterion measure 113. Based on Edgar Dale’s Cone of Experience, which activity is farthest from the real thing? A. Read C. View images B. Hear D. Attend exhibit 114. After giving an input on a good paragraph, Teacher W asks

her students to rate a given paragraph along the elements of a good paragraph. The students’ task is in level of _________. A. application C. evaluation B. analysis D. synthesis 115. Which is one characteristic of an effective classroom management? A. It quickly and unobtrusively redirects misbehavior once it occurs. B. It teaches dependence on other for self-control. C. It respects cultural norms of a limited group students. D. Strategies are simple enough to be used consistently 116. Read the following then answer the questions: TEACHER: IN WHAT WAYS OTHER THAN THE PERIODIC TABLE MIGHT WE PREDICT THE UNDISCOVERED ELEMENTS? BOBBY: WE COULD GO TO THE MOON AND SEE IF THERE ARE SOME ELEMENTS THERE WE DON’T HAVE. BETTY: WE COULD DIG DOWN TO THE CENTER OF THE EARTH AND SEE IF WE FIND ANY OF THE MISSING ELEMENTS.

RICKY: WE COULD STUDY DEBRIS FROM THE METEORITES- IF WE CAN FIND ANY. TEACHER: THOSE ARE ALL GOOD ANSWERS BUT WHAT IF THOSE EXCURSIONS TO THE MOON, TO THE CENTER OF THE EARTH OR TO FIND METEORITES WERE TOO COSTLY AND TIME CONSUMING? HOW MIGHT WE USE THE ELEMENTS WE ALREADY HAVE HERE ON EARTH TO FIND SOME NEW ONES? Question: which questioning strategy/ies does/do the exchange of thoughts above illustrate? A. Funneling B. Sowing and reaping C. Nose-dive D. Extending and lifting 117. How can you exhibit referent power on the first day of school? A. By making them feel you know what you are talking about. B. By telling them the importance of good grades. C. By reminding your students your authority over them again and again. D. By giving your students a sense of belonging and acceptance. 118. A sixth grade twelve-year old boy comes from a dysfunctional

family and has been abused and neglected. He has been to two orphanages and three different elementary schools. The student can decode on the second grade level, but he can comprehend orally material at the fourth or fifth grade level. The most probable cause/s of this student’s reading problem is/are ______. A. emotional factors B. C.

poor teaching neurological factors

D. immaturity 119. Which questioning practice promotes more class interaction? A. Asking the question before calling a student B. Focusing on divergent questions. C. Focusing on convergent questions. D. Asking rhetorical questions 120. The following are sound specific purpose of questions EXCEPTS A. To call the attention of an inattentive student B. To teach via student answers C. To stimulate learners to ask questions D. To arouse interest and curiosity

121. Which technique should a teacher use to encourage response if his students do not respond to his question? A. Ask a specific student to respond, state the question, and wait a response. B. Tell the class that it will have detention unless answers are forthcoming. C. Ask another question, an easier one. D. Wait for a response. 122. Which types of play is most characteristic of a four to six-year old child? A. Solidarity plays and onlookers plays B. Associative and cooperative plays C. Associative and onlookers plays D. Cooperative and solidarity plays 123. The principle of the individual difference requires teachers to ___________. A. give greater attention to gifted learners B. provide for a variety of learning activities C. treat all learners alike while in the classroom D. prepare modules for slow learners in class

124. Referring to Teacher S, Nicolle describes her teacher as “fair, caring and someone you can talk to”. Which power or leadership does Teacher S have? A. Referent power C. Reward power B. Legitimate power D. Expert power 125. During the Spanish period, what was/were the medium/media of instruction in schools? A.The Vernacular C. Spanish B.English D. Spanish and the Vernacular 126. Rodel is very aloof and cold in his relationship with classmates. Which basic goals must have not been attained by Rodel during his developmental years, according to Erickson’s theory of psychological development? A. Autonomy C. Initiative B. Trust D. Generativity 127. How can you exhibit expert power on the first day of school? A. By making them feel you know what you are talking about. B. By telling them the importance of good grades. C. By reminding then your students your authority over them again and again.

D. By giving your students a sense of belonging and acceptance. 128. Teacher P wants to develop the skill of synthesizing in her pupil. Which one will she do? A. Ask her students to formulate a generalization from the data shown in graphs. B. Ask her students to answer questions beginning with “What if …” C. Tell her pupils to state data presented in graphs. D. Directs her students to ask questions on the parts of the lesson not understood. 129. John Watson said “Men are built not born. ” What does this statement point to? A. The ineffectiveness of training on a person’s development. B. The effect of environmental stimulation on a person’s development. C. The absence of genetic influence on a person’s development. D. The effect of heredity. 130. Teacher H strives to draw participation of every student into classroom discussion. Which student’s need is she trying to address? The need

A. to show their oral abilities to the rest of the class B. to be creative C. to feel significant and be part of a group D. to get everything out in the open 131. Teacher G’s lessons objective has something to do with the skill of synthesizing? Which behavioral term is most appropriate? A. Test C. Appraisal B. Assess D. Theorize 132. Which is a sound classroom management practice? A. Avoid establishing routines; routines make your student robots. B. Establish routines for all daily needs and tasks. C. Apply rules and policies on a case to case basis. D. Apply reactive approach to discipline. 133. A child who gets punished for stealing candy may not steal again immediately. But this does not mean that the child may not steal again. Based on Thorndike’s theory on punishment and learning, this shows that ______________. A. punishment strengthen e a response

B. punishment remove a response C. punishment does not remove a response D. punishment weakens a response 134. Which assumption underlines the teacher’s use of performance objectives? A. Not every form of learning id observable. B. Performance objectives assure the learner of learning. C. Learning is defined as a change in the learner’s observable performance. D. The success of the learner is based on teacher’s performance. 135. As a teacher, what do you do when you engage yourself in major task analysis? A. Test if learning reached higher level thinking skills B. Breakdown a complex task into sub-skills C. Determine the level of thinking involved D. Revise lesson objectives 136. The following are used in writing performance objectives EXCEPT A. delineate C. integrate B. diagram D. comprehend

137. Teacher B clears his throat to communicate disapproval of a student’s behavior. Which specific influence technique is this? A. Signal interference C. Interest boosting B. Direct appeal D. Proximity control 138. An effective classroom manager uses low-profile classroom control. What is a lowprofile classroom technique? A. Note to parents B. After-school detention C. Withdrawal of privileges D. Raising the pitch of the voice 139. The primary objective of my lesson is: “To add similar fractions correctly.” Before I can do this I must first aim at this specific objective: “to distinguish a numerator from a nominator. “What kind of objective is the latter? A. Major C. Enabling B. Terminal D. Primary 140. With which goals of educational institutions as provided for by the Constitution is the development of work skills aligned? A. To develop moral character B. To teach the duties of citizenship C. To inculcate love of country

D. To develop vocational efficiency 141. In instructional planning it is necessary that parts of the plan from the first to the last have ___________. A. clarity C. coherence B. symmetry D. conciseness 142. All of the following describe the development of children aged eleven to thirteen EXCEPT A. they shift from impulsivity to adaptive ability B. sex differences in IQ becomes more evident C. they exhibit increased objectivity in thinking D. they show abstract thinking and judgement 143. If a teacher plans a constructivist lesson, what will he most likely do? Plan how he can _______. A. do evaluate his student’s work B. do reciprocal teaching C. do lecture to his students D. engage his students in convergent thinking 144. In mastery learning, the definition of an acceptable standard of performance is called a A. SMART C. behavior

B. criterion measure D. condition 145. “A stitch on time saves nine”, so goes the adage. Applied to classroom management, this means that we _______. A. may not occupy ourselves with disruptions which are worth ignoring because they are minor B. must be reactive in our approach to discipline C. have to resolve minor disruptions before they are out of control D. may apply 9 rules out of 10 consistently 146. Ruben is very attached to his mother and Ruth to her father. In what developmental stage are they according to Freudian psychological theory? A. Oedipal stage C. Anal Stage B. Latent stage D. Pre-genital stage 147. What was the prominent educational issues of the mid 1980’s? A. Bilingual Education C. Accountability B. Value Education D. Mainstreaming 148. Which behavior is exhibited by a student who is strong in interpersonal intelligence?

A. Works on his/her own. B. Keeps interest to himself/herself C. Seeks out a classmate for help when problem occurs. D. Spends time meditating. 149. Which is behavioral term describes a lesson outcome in the highest level of Bloom’s cognitive domain? A. Create C. Analyze B. Evaluate D. Design 150. The main purpose of compulsory study of the Constitution is to ___. A. develop students into responsible, thinking citizens B. acquaint student with the historical development of the Phil Constitution C. make constitutional experts of the students D. prepare students for lawmaking 151. A goal-oriented instruction culminates in _______. A. planning activities B. evaluation C. identification of topics D. formulation of objectives 152. Based on Freud’s psychoanalytic theory which component (s) of personality is

(are) concerned with a sense of right and wrong? A. Super ego C. Id B. Super-ego and Ego D. Ego 153. Who among the following puts more emphasis on core requirements, longer school day, longer academic year and more challenging textbooks? A. Perennialist C. Progressivist B. Essentialist D. Existentialist 154. A student passes a research report poorly written but ornately presented in a folder to make up for the poor quality of the book report content. Which Filipino trait does this practice prove? A. art of academics B. substance over “porma” C. art over science D. “porma” over substance 155. Which one may support equitable access but may sacrifice quality? A.Open admission C. Deregulated tuition fee hike B. School accreditation D. Selective retention 156. Based on Piaget’s theory, what should a teacher provide for children in the sensimotor stage? A. Games and other physical activities to develop motor skill.

B. Learning activities that involve problems of classification and ordering. C. Activities for hypothesis formulation. D. Stimulating environment with ample objects to play with. 157. A teacher’s summary of a lesson serves the following functions, EXCEPT A. it links the parts of the lesson. B. it brings together the information that has been discussed. C. it makes provisions for full participation of students. D. it clinches the basic ideas or concepts of the lesson. 158. As a teacher, you are rationalist, Which among these will be your guiding principle? A. I must teach the child that we can never have real knowledge of anything. B. I must teach the child to develop his mental powers to the full. C. I must teach the child so he is assured of heaven D. I must teach the child every knowledge, skills and value that he needs for a better future. 159. Bruner’s theory on intellectual development moves from enactive

to iconic and symbolic stages. In which stage(s) are diagrams helpful to accompany verbal information? A. Enactive and iconic C. Symbolic and enactive B. Symbolic D. Iconic 160. Student’s scores on a test were: 72,72,73,74,76,78,81,83,85. The score 76 is the ___. A. mode C. average B. mean D. median 161. Standard deviation is to variability as mode to _____. A. level of difficulty C. correction B. discrimination D. central tendency 162. A teacher who equates authority with power does NOT usually __________. A.Shame C. develop selfrespect in every pupil B.retaliate D. intimidate 163. Which is a true foundation of the social order? A. Obedient citizenry B. The reciprocation of rights and duties C. Strong political leadership D. Equitable distribution of wealth 164. Standard deviation is to variability as mean is to _______.

A. coefficient of correlation C. discrimination index B. central tendency D. level of difficulty 165. Teacher Q does not want Teacher B to be promoted and so writes an anonymous letter against Teacher B accusing her of fabricated lies. Teacher Q mails this anonymous letter to School Division Superintendent. What should Teacher Q do if she has to act professionally? A. Submit a signed justifiable criticism against Teacher B, if there is any. B. Go straight to the School Division Superintendent and gives criticism verbally. C. Hire a group to distribute poison letters against Teacher B for the information dissemination. D. Instigate student activists to read poison letter over the microphone. 166. Each teacher is said to be a trustee of the cultural and educational heritage of the nation and is under obligation to transmit to learners such heritage. Which practice makes him fulfill such obligation? A. Use the latest instruction technology

B. Observing continuing professional education. C. Use interactive teaching strategies D. Study the life of Filipino heroes 167. Which type of reports to “onthe spot” description of some incident, episode or occurrence that is being observed and recorded as being of possible significance? A. Autobiographical report B. Value and interest report C. Biographical report D. Anecdotal report 168. Teacher A is directed to pass an undeserving student with a death threat. Which advice will a hedonist give? A. Pass the student. Why suffer the threat? B. Don’t pass him. You surely will not like someone to give you a death threat in order to pass. C. Pass the student. That will be use to the student, his parents and you. D. D. Don’t pass him. Live by principle of justice. You will get reward, if not in this life, in the next. 169. If you agree with Rizal on how you can contribute to our nation’s

redemption, which should you work for? A. Opening our doors to foreign influence B. Upgrading the quality of the Filipino through education C. Stabilizing the political situation D. Gaining economic recovery 170. “All men are pretty much alike. It is only by custom that they are set apart, “ said one Oriental philosopher. Where can this thought be most inspiring? A. In a multi-cultural group of learners. B. In multi-cultural and heterogeneous groups of learners and indigenous peoples’ group. C. In a class composed of indigenous people. D. In heterogeneous class of learners. 171. In what way can teachers uphold the highest possible standard of quality education? A. By continually improving themselves personally and professionally B. By wearing expensive clothes to change people’s poor perception of teachers

C. By working out undeserved promotions D. By putting down other professions to lift the status of teaching 172. How would you select the most fit in government position? Applying Confucius teaching, which would be the answer? A. By course accreditation of an accrediting body B. By merit system and course accreditation C. By merit system D. By government examinations 173. The attention to the development of a deep respect and affection for our rich cultural past is an influence of ______ A. Confucius C. Teilhard de Chardin B. Heqel D. Dewey 174. A teacher / student is held responsible for his actions because s/he _______. A. has instincts C. has mature B. has a choice D. is reason 175. The typical autocratic teacher consistently does the following EXCEPT A. intimidating students C. shaming students B. ridiculing students

D. encouraging students 176. Teacher H and teacher I are rivals for promotion. To again the favor of the promotion staff, teacher I offers her beach resort for free for members of he promotional staff before ranking. As one of the contenders for promotions, is this becoming of her to do A. Yes. This will be professional growth for the promotional staff. B. No. This may exert undue influence on the members of the promotional staff and so may fail to promote on the basis of merit. C. Yes. The rare invitation will certainly be welcomed by an overworked promotional staff. D. Yes. There’s nothing wrong with sharing one’s blessings. 177. Rights and duties are correlative. This means that. A. rights and duties regulate the relationship of men in society. B. rights and duties arise from natural law. C. each right carries with it one or several corresponding duties. D. rights and duties ultimately come from God. 178. In the Preamble of Code of Ethics of Professional Teachers, which is NOT said of teachers?

A. LET passer B. Duly licensed professionals C. Possess dignity and reputation D. With high moral values as well as technical and professional competence 179. What should you do if a parent who is concerned about a grade his child received compared to another student’s grade, demands to see both students’ grades? A. Refuse to show either record. B. Show only his child’s record. C. Refuse to show any record without expressing permission from principal. D. Show both records to him. 180. Teacher often complain of numerous non-teaching assignments that adversely affect their teaching. Does this mean that teachers must be preoccupied only with teaching? A. Yes, if they are given other assignments, justice demands that they be properly compensated. B. Yes, because other community leaders, not leaders, not teachers, are asked to lead in community activities. C. Yes, because every teacher is expected to provide leadership and initiative in

activities for betterment of communities. D. No, because teaching is enough full time job. 181. Which illustrates a development approach in guidance and counseling? A. Spotting on students in need of guidance B. Teaching students how to interact in a positive manner C. Acting as a mentor D. Making the decision for the confused student 182. Whose influence is the education program that puts emphasis on self-development through the classics, music, and ritual? A. Buddha C. Confucius B. MohammedD. Lao tsu 183. Helping in the development of graduates who are “maka-Diyos” is an influence of ___. A. naturalistic morality B. classical Christian morality C. situational morality D. dialectical morality 184. What is the mean of this score distribution 4,5,6,7,8,9,10? A. 7 C. 8.5 B. B. 6 D. 7.5

185. Are percentile ranks the same as percentage correct? A. It cannot be determined unless scores are given. B. It cannot be determined unless the number of examinees is given C. No D. Yes 186. Teacher F is a newly converted to a religion. Deeply convinced of his new found religion, he starts Monday classes by attacking one religion and convinces his pupil to attend their religion services on Sundays. Is this in accordance with the Code of Ethics of Professional Teachers? A. Yes. What he does strengthens value education. B. No. A teacher should not use his position to proselyte others. C. Yes. In the name of academic freedom, a teacher can decide what to teach. D. Yes. What he does is a value education. 187. In a study conducted, the pupils were asked which nationality they preferred. If given a choice. Majority of the pupils wanted to Americans. In this case, in which obligation, relative to the state, do

schools seem to be failing? In their obligation to A. respect for all duly constituted authorities B. promote national pride C. promote obedience to the laws of the state D. install allegiance to the Constitution 188. A guest in one graduation rites told his audience: “Reminder, you are what you choose to be”. The guest speaker is more of a/an _____. A. realistic C. pragmatist B. Idealistic D. existentialist 189. From whom do we owe the theory of deductive interference as illustrated in syllogism? A. Plato C. Socrates B. Aristotle D. Pythagoras 190. In what way can teachers uphold the highest possible standard of quality education? A. By continually improving themselves personally and professionally B. By putting down other professions to lift the status of teaching C. By wearing expensive clothes to change people’s poor perception of teachers

D. By working out undeserved promotions 191. Two students are given the WISC III. One has a full scale IQ of 91, while the other has an IQ of 109. Which conclusion can be drawn? A. Both students are functioning in the average range of intellectual ability B. Another IQ test should be given to truly assess their intellectual potential. C. The first student is probably below average, while the second has above average potential. D. The second student has significantly higher intellectual ability. 192. Which describes normreferenced grading? A. What constitutes a perfect score B. The student’s past performance C. An absolute standard D. The performance of the group 193. Teacher A knows of the illegal activities of a neighbor but keeps quiet in order not to be involved in any investigation. Which foundational principle of morality does Teacher A fail to apply? A. Always do what is right

B. The principle of double effect C. The end does not justify the means D. Between two evils, do the lesser evil 194. Teacher Q does not want Teacher B to be promoted and so writes an anonymous letter against Teacher B accusing her of fabricated lies. Teacher Q mails this anonymous letter to School Division Superintendent. What should Teacher Q do if she has to act professionally? A. Hire a group to distribute poison letters against Teacher B for the information dissemination. B. Submit a signed justifiable criticism against Teacher B, if there is any. C. Go straight to the School Division Superintendent and gives criticism verbally. D. Instigate student activists to read poison letter over the microphone 195. A teacher’s summary of a lesson serves the following functions, EXCEPT A. it makes provisions for full participation of students. B. it brings together the information that has been discussed.

C. it links the parts of the lesson. D. it. clinches the basic ideas or concepts of the lesson 196. Soc exhibits fear response to freely roaming dogs but does not show fear when a dog is on a leash or confined to a pen. Which conditioning process is illustrated? A. Extinction C. Generation B. Discrimination D. Acquisitio 197. Ruben is very attached to his mother and Ruth to her father. In what developmental stage are they according to Freudian psychological theory? A. Latent stage C. Anal Stage B. Pre-genital stage D. Oedipal stage 198. The concepts of Trust vs. mistrust, autonomy vs. shame & self-doubt, and initiative vs. guilt are most closely related with works of _____. A. Jung C. Erickson B. Freud D. Piaget 199. A goal-oriented instruction culminates in _______. A. evaluation B. formulation of objectives C. identification of topics D. planning activities

200. Which of the following is considered a peripheral device? A. Keyboard C. Monitor B. CPU D. Printer

21.

A

70.

A

120.

A

170

B

ANSWER KEY

22.

B

71.

C

121.

C

171.

A

PROFESSIONAL EDUCATION

23.

A

72.

C

122.

C

172.

D

(Secondary)

24.

C

73.

A

123.

B

173.

A

25. no answer

74.

A

124.

A

174.

26.

C

75.

D

125.

C

175.

A

1.

A

50.

C

100.

D

150.

B

2.

B

51.

B

101. no answer 151.

3.

D

52.

D

102.

4.

A

53.

5.

A

54.

A

6.

B

55.

7.

D

8.

D

B

152.

A

27.

A

76.

B

126.

B

176.

B

103.

B

153. no answer

28.

D

77.

C

127.

D

177.

C

104.

B

154.

D

29.

C

78.

B

128.

A

178.

A

B

105.

A

155.

A

30.

C

79.

B

129.

B

179.

D

56.

C

106.

D

156.

A

31.

A

80.

C

130.

C

180.

C

C

57.

A

107.

D

157

C

32.

C

81.

C

131.

D

181.

A

9.

C

58.

D

108.

C

158.

B

33.

C

82.

C

132.

C

182.

A

10.

C

59.

B

109.

B

159. no answer

34.

A

83.

C

133.

D

183.

B

11.

C

60.

D

110.

C

160.

D

35.

D

84.

D

134.

C

184.

A

12.

D

61.

B

111.

D

161.

D

36.

C

85.

A

135.

B

185.

C

13.

C

62.

C

112.

C

162.

C

37.

B

86.

C

136.

D

186.

B

14.

C

63.

D

113.

C

163.

D

38.

A

87.

C

137.

B

187.

B

15.

B

64.

A

114.

A

164.

B

39.

B

88.

A

138.

C

188.

D

16.

C

65.

D

115.

D

165.

A

40

C

89.

A

139.

C

189.

B

17.

B

66.

A

116.

A

166.

D

41.

B

90.

B

140.

D

190.

A

18.

C

67.

A

117.

A

167.

D

42.

D

91.

B

141.

C

191.

B

19.

B

68.

C

118.

A

168.

D

43.

A

92.

B

142.

D

192.

D

20.

C

69.

D

119.

B

169.

B

44.

B

93.

B

143.

D

193.

C

no answer

B

45

C

94.

C

144.

B

194.

B

46.

C

95.

B

145.

C

195.

D

47.

A

96.

D

146.

A

196.

B

48.

A

97.

D

147.

B

197.

D

49.

A

98.

B

148.

C

198.

C

99.

A

149.

199.

A

200.

D

General Education (secondary) MULTIPLE CHOICE 1. Which refers to FUNNEL EFFECT?

B

A. The belief that every criminal gets caught and is punished. B. The belief that crime is under control in the United States. C. The idea that only a very few suspects arrested for

committing a crime are actually punished. D. The idea that all crimes put into the same criminal justice system. 2. One of the most outstanding accomplishments of the cooperative movement is the encouragement of thrift. Which

maximum of God puts this into practice? A. “God Helps those who help themselves”. B. “Look at the birds: they do not plant seeds, gather a harvest and put it in barns; yet your Father in heaven takes care of them!” C. “He is near to those who call to Him, who call to Him with sincerity.” D. “Happy are those who are merciful to others; God will be merciful to them!” 3. Which is NOT personal integrity? A. Time C. Order B. Place D. Harmony 4. Carter’s part in relinquishing U.S. control of the Central Zone to Panama is described as a victory for ________. A. conservatism C. isolationism B. anti-imperialism D. imperialism 5. Of the following changes in the socio-economic, political cultural and physical that have occurred in the Filipino family, which one remains to be TRUE? A. The loss of the traditional evening prayer and ritual of blessing (mano)

B. The unity of the family despite competing demands C. The continued parental influence over children’s language dress and other behavior D. The continued support for parents and siblings 6. Mang Tacio has been unemployed for quite sometime due to his negative attitude toward work. Which program of the Department of Social Welfare and Services will help him? A. Income in Kind Program C. HRD Program B. Anti-Medicancy Program D. Social Insurance Program 7. The main message of the Moral Recovery Program launched by Leticia Ramos Shahani starts with the ________. A. world C. self B. family D. nation 8. Which are limited only to the sale of real property and stock transaction? A. Business incomes C. Employment incomes B. Capital gains D. Passive incomes

9. If a farmer would want assistance like pricing, guarantee for all agricultural produce or cooperative management training, where would he go? A. Support services of the Department of Agrarian Reform B. Special Agrarian Court under the Regional Trial Court C. DAR adjudication board D. Land Bank 10. The Soviet Union’s attempt to establish a missile base in Cuba is interpreted as a direct violation of A. The Truman Doctrine B. The Monroe Doctrine C. The Strategic Arms Limitation Talks (SALT) D. The Declaration of Independence 11. As a representative of the Urban Poor Commission of the Association of Religious Superiors (ARS), which action will you most likely take to resolve the long-term roots of structural inequalitiesproliferation of child labor and child prostitution? A. Raise views of human rights abuse. B. Organize regular programs for information and discussion of human rights C. Conduct skills training

D. Raise questions over the government’s commitment to rebuild human rights 12. What values are being given priority by juries in criminal cases? A. The rights of the criminal over the strict interpretation of the law B. The safety of the community over the sympathy for the criminal C. The needs of the criminals over the advice of the judge D. The punishment of the criminal over the safety of the community 13. Lucy’s husband has been a drug dependent. She wanted him rehabilitated to be economically productive. Where will she commit her husband? A. DARN C. DARE B. Bukang Liwayway Ceter D. NFPI 14. Which will solve poverty caused by capitalism? A. Fascism C. Empirism B. Communism D. Socialism 15. Which family obligation is especially valued? A. Supplying groceries to relatives in remote barrios

B. Providing health assistance to relatives living in the same locale C. Sending to college relatives in remote barrios D. Keeping immediate family members out of trouble 16. Which theory was asserted by the Pan-German belief in the superiority of the Aryan race and that the strength of the German culture came from a strong, healthy and rustic lineage? A. Dependency Theory B. Culture of Poverty Theory C. Social Darwinist Theory D. Theory of Capitalism 17. Why was San Andres Cooperative Association of Paco, not exempted from taxation? A. I accumulates reserves and undivided net savings of P8,000,000.00 B. I accumulates reserves and undivided net savings of P10,000.00 C. I accumulates reserves and undivided net savings of P9,000,000.00 D. I accumulates reserves and undivided net savings of P11,000.00

18. Juliet Villaruel was a landowner from Cabio, Nueva Ecija. Under the CARL, she was claiming 8 hectares, 5 hectares of which represented the retention limit and the 3 hectares for her only child. Why was her child denied 3 hectares? A. Her son was 15 years old who was actually tilting the farm B. Her son was 17 years old who was managing the farm C. Her son was 13 years old who has been helping till the farm D. Her son was 19 years old who was actually tilting or managing the farm 19. Which part of the Allied action has been detailed in this passage? After Saddam Hussein violated international arguments by sending Iraq troops to Kuwait and missiles into their neighboring countries, the Allies responded with military action. A. The Allied bases in Saudi Arabia B. The ground was in the desert C. The movement of Allied troops in Iraq D. The campaign in the Baghdad area

20. Which one is the human right to life? A. Peace C. Own Property B. Live in national and international order D. Fair trial 21. Two days after Japan attacked Pearl Harbor, Roosevelt made the following statements: “In the past few years and most violently in the past few days, we learned a terrible lesson. We must begin the great task that is before us by abandoning once and for all the illusion that we can ever again isolate ourselves from the rest of the humanity.” In the statement, Roosevelt is expressing the ideas of ________. A. an internationalist C. an imperialist B. an anti-imperialist D. an isolationist 22. The following are defects present at the time of marriage which is voidable and annullable EXCEPT A. impotence C. fraud B. deceit D. threat 23. Why does a pendulum in a grandfather clock once set in

motion continue to swing, thereby regulating the clock’s movement? This is due to the Law of A. Universal Gravitation C. Applied Force B. Action and Reaction D. Inertia 24. New ponies, perennial plants that produce shows flowers can be propagated from the parent plant by dividing corns that grow underground. The reproductive form resembles a _____________. A. bulb C. seed B. runner D. bud 25. The San Pascual Credit Cooperative of Quezon City wishes to apply for a loan of five pesos from one of the financial institution, EXCEPT? A. Development Bank of the Philippines B. Central Bank of the Philippines C. Philippine National Bank D. Land Bank of the Philippines 26. The following are legitimate children EXCEPT? A. those born by artificial insemination. B. those legitimate. C. those born during a valid marriage of parents.

D. those born out a valid marriage of parents. 27. Of the following, which is imposed a final tax of ten percent (10%) A. PSCO and lotto winnings. B. Books literary works and musical compositions C. Currency banks deposit D. Royalties 28. Which of the following foreign policy actions today is a direct result of early American imperialist policy? A. Station of American troops in West Germany B. American’s military support of Israel in the Middle East C. American’s patrolling of waters of the Libyan coast D. American’s establishment of military bases I nth Philippines 29. Which area of the brain controls feelings on the side of a person’s face? A. The left occipital lobe C. The right parietal lobe B. The left parietal lobe D. The right occipital lobe 30. Why does a bullet when discharged into the air eventually fall to the ground? This is due to the Law of ___________. A. Universal Gravitation

C. Inertia B. Applied Force D. Action and Reaction 31. The principle under which thermostat operates is the same when? A. a gas expands to fill the container in which it is held. B. a pendulum swings when it is set into motion. C. a chemical reaction occurs when two substances combine. D. the level of mercury rises or falls in a glass tube. 32. Which explains the reason why there are continuous and increasing human rights violations? A. The United Nation’s General Assembly approved only resolutions on human rights and the basic freedoms which are not binding B. The solutions used are ineffective. C. The United Nations as an international body is rather slow in the exercise of its powers D. The United Nations uses as a single solution on all forms of human rights violations. 33. In an experiment, a vacuum is created when air is removed from a tube. A coin and bits of confetti are released in the vacuum

at the same time. They fall at the same rate and reach the bottom at the same time. The experiment proves that I. In a vacuum, the rate of accelerator is the same for all objects regardless of weight. II. Outside a vacuum air resistance is what makes different objects fall at different rates. III. Gravity has no effect at all on objects that fall in a vacuum. A. I and II C. I, II and III B. I and III D. II and III 34. Thousands of street children in large Brazilian cities were murdered by parliamentary death squads which includes police officers. What could be the reason why these operations were not suppressed by the government? A. Totalitarian governments do not give protection B. The business people even funded these operations to clean up their streets and neighborhoods. C. The Universal Declaration of Human Rights was only lip service D. To how they treat their people was nobody else’s business. 35. Which of the following procedures used by a farmer is

NOT related directly to preventing erosion? A. Contour plowing around a hill B. Planting more seeds than are necessary to yield a bountiful crop. C. Planting grass in gullies to act as a filter D. Planting crops in alternate rows (Strip farming) 36. You buy a new refrigerator for P12,800.00 and make a down payment of P2,500.00. If you finance the remainder at 8% annually for three years, how much will you actually pay for the refrigerator? A. P12,190.00 D. P12,772.00 B. P10,309.00 E. P15,272.00 C. none of these 37. 4 1/5 or 4 1/5 + 3 2/7 = __________. + 3 2/7 ________ A. 7 3/12 C. 7 17/35 B. 7 3/35 D. 7 1/35 E. none of these 38. In which kingdom should MOLD be classified? A. Protista C. Fungi

B. Plantae D. Animalia 39. Which of the following BEST demonstrates the greenhouse principle? A. A heated aquarium B. A car with rolled-up windows C. A microwave oven D. A solar battery-powered calculator 40. Which of the following should you expect to be true about the rate of cellular respiration for a group of students who are the same age, height, and weight? A. Athletes would tend to have higher rates of cellular respiration than nonathletes. B. Africans would have higher rates of cellular respiration than Asians. C. Boys would have a higher rates of cellular respiration than girls. D. Nonathletes would have higher rates of cellular respiration than Athletes. 41. Which kingdom should STREPTOCOCCUS be classified? A. Protista C. Fungi B. Plantae D. Monera 42. What is the function of DIFFUSION in the human body? A. Regulates blood flow B. Plays an insignificant role in the body’s functioning

C. Allows an even distribution of substances throughout all cells of the body D. Comes into play in times of extreme illness 43. In an experiment, a drop of blue ink is placed on the surface of a glass of water. In a few minutes, the drop of ink is dispersed throughout the water, turning it light blue. The result of the experiment proves that A. molecules of ink and molecules of water are in constant motion B. heat causes the ink to disperse C. a new compound is formed by the combination of ink and water D. ink molecules have less density than water molecules 44. Which Law of force and motion explains this occurrence, when a rocket is propelled upward by the powerful downward discharged of exhaust gases? A. Universal Gravitation C. Applied Force B. Action and Reaction D. Inertia 45. An elderly woman suffered a stroke-a restriction of blood flow to the brain. if the stroke caused to

the right side of her body to become temporarily paralyzed, she most likely experienced a decreased blood flow to A. the left side of her body C. the left side of her brain B. the front of her brain D. the right side of her brain 46.Which of the following methods can all diabetics control their condition and avoid heart disease and blindness? I. Regulates their intake of glucose II. Increase the levels of insulin in the body by taking insulin injections III. Maintaining a reasonable exercise regimen to keep weight down A. I C. I and II B. II D. I and III 47. Scientist also find that other stalky vegetables such as carrots also help lower pressure. This statement is BEST classified as A. experiment C. finding B. nonessential fact D. prediction 48. Each of the following objects is designed to employ the buoyancy principle EXCEPT a A. life preserver C. submarine

B. kite D. canoe 49. What is the difference between the largest 4-digit number and the smallest 4-digit numbers? A. 8999 D. 8888 B. 8000 E. none of these C. 9998 50. Four mangoes cost P29.00 at that price what will 2½ dozen mangoes cost? A. P217.50 C. P348.50 B. P188.50 D. P870.00 E. none of these 51. Which of the following internal forces interrupt the external forces erosion? I. Forces that cause volcanoes II. Forces that cause ocean trenchers III. Forces that cause create mountains A. I, II and III C. II and III B. I and III D. I and II 52. The How many gallons of water will fill a fish tank that is 18 inches by 12 inches by 48 inches (There are 231 cubic inches per gallon) Round your answer to the nearest gallon. A. 45 gallons D. 47 gallons B. 40 gallons E. 37 gallons C. none of these

53. What is the sum of all the two digit numbers which are divisible by 5? A. 945 D. 1050 B. 950 E. none of these C. 960 54. How many whole numbers can divide 30 exactly? A. Eight C. Five B. Six D. Four E. none of these 55. Which one explains why oxygen, a gas is the largest component of the Earth’s crust? A. Oxygen gives Earth’s crust its lightness B. Oxygen is the most abundant element in the world C. Oxygen is capable of combining with most of the elements in the Earth’s crust D. Oxygen is needed to sustain all life on Earth 56. To pass the English Test, Lucille must get 75% of the items correct. Out of 80 questions, how many must she correctly answer? A. 55 D. 65 B. 60 E. 70

C. none of these 57. Employees at Shaira’a Musicmart get a 20% discount on all purchases. If Teresa buys three tapes at P47.49 each. How much will she have to pay after her employee discount? A. P16.98 D. P17.98 B. P19.98 E. none of these C. P18.98 58. One package is 100 pounds, and the other is 150 pounds. The weight of the second package is how many times of the first? A. 1 ½ times heavier D. 10 pounds heavier B. ½ as heavy E. 20 pounds heavier C. none of these 59. A carpenter wanted three piece of wood each 1 5/8 feet long. If he planned to cut them from a 6foot piece of wood, how much of the piece would be left? A. 4 3/8 ft C. 4 7/8 ft B. 1 1/8 ft D. 3 ft E. none of these 60. How much larger is the supplement of a 57 degree angle than the complement of a 75 degree angle?

A. 108 degrees C. none of these B. 18 degrees D. 123 degrees c. 105 degrees 61. If a baseball player hits 10 home runs in the first 45 games, at the same rate how many home runs can he expect to hit during 162-games season? A. 38 C. 36 B. 42 D. 40 E. none of these 62. Which are the next three terms in the progression 1/125, -1/25, 1/5…7 terms? A. –2,6,-26 C. –4,8,-28 B. –3,7,-27 D. –1,5,-25 E. none of these 63. How many ways can a committee of 4 people be selected from a group of 7 people? A. 35 D. 210 B. 70 E. none of these C. 140 64. Which is the length of the hypotenuse of a right triangle with legs 5 inches and 12 inches? A. 17 in. C. 11 in. B. 13 in. D. 20 in.

E. none of these 65. Which of the following is the BEST example of self-preservation? A. A mouse runs when it sees a cat. B. A dog barks when it sees its owner C. A Man decide to quit smoking D. A salmon swims back to the place of its birth to lay eggs. 66. A meter was cut at the 35-cm mark. What is the ratio of the smaller piece to the larger piece? A. 7:13 C. 35:100 B. 65:35 D. 65:100 E. none of these 67. The hypotenuse of a triangle is 25 feet. If one leg is 24 feet, what is the length of the other leg? A. 6 ft D. none of these B. 5 ft E. 7ft C. 20 ft 68. Which is the equivalent common fraction of the repeating decimal 3.242424…? A. 107/33 C. 109/33 B. 110/33 D. 108/33 E. none of these 69. Tides, caused by the moon’s gravity, create a fractional force that is gradually slowing down Earth’s rotational speed. One million years from now, scientist may discover that compared to today, Earth’s

A. day is longer C. day is shorter B. year is shorter D. year is longer 70. How much topsoil is needed to cover a garden 25 feet by 40 feet to a depth of 6 inches? A. 480 cuft D. none of these B. 440 cuft E. 460 cuft C. 500 cuft 71. A car dealer is offering a rebate of P7,500.00 on any new-car purchase. If the purchase price of a car is P200,000.00 more than it was last year. What is the rate of the discount offered by the rebate? A. 10% C. 7.5% B. not enough information is given D. 13.3% E. 14.2% 72. In the progression 18, -12,8……which term is 512/729? A. the 8th C. the 9th B. the 6th D. the 7th E. none of these 73. Which of the following facts support the big bang theory’s explanation of the creation of the universe? A. The universe does not expand nor contract. B. The universe seldom expand. C. The universe will have background radiation.

D. The universe has no beginning nor end. 74. Mr. Garcia owns a 10½ hectares tract of land. He plans to subdivide this tract into ¼ hectare lots. He must first set aside 1/6 of the total land roads. How many lots will this tract yield? A. 30 C. 42 B. 35 D. 45 E. none of these 75. Find m in the proportion m/12=30/24. A. 30 C. 20 B. 15 D. 25 E. none of these 76. If P75,000 is shared among three children in the ratio of 3:7;15, the size of the smaller share is? A. P9 C. P25 B. P15 D. P35 E. none of these 77. In how many ways can you arrange three mathematics books (Algebra, Geometry, Trigonometry) in order on shell? A.6 C. 12 B.8 D. 24 E. none of these 78. Which are the next three terms in the progression 1,4,16…8 terms? A.64,256,1024 C. 66,258,1026 B.67,259,1027 D. 65,257,1025 E. none of these

79. Which one should be TRUE is Earth’s rotational axis not tilt? I. Days and nights would be the same length everywhere on Earth. II. There would be no hours of darkness on points along the equator III. Earth would have no seasons. IV. Each part of the Earth would have the same daily temperature pattern A. II and IV C. IV and I B. III and IV D. I and III 80. The carat is a unit of measure used to weight precious stones. It equals 3.086 grains. How many grains does 2.8 carat diamond weigh? A. 864.08 C. 8.6408 B. 86.408 D 5640.8 E. none of these 81. Robert Frost wrote the poem “Acquainted with the Night” from which the stanza is taken: I have been one acquainted with the night. I have walked out in rain-back in rain. I have outwalked the farthest city light. The poet in the stanza talks of A. isolation and loneliness.

B. happiness in having been acquainted with the night. C. joy getting out of the house. D. youthful delight playing in the rain. 82. What is meant by AT SIX AND SEVENS in this sentence? We moved into the house last week, but I’m afraid everything is still at six and sevens. A. The things have not been shipped C. In an orderly manner B. In a state of confusion D. The boxes are still intact 83. Sa “Espiritu ni Bathala ang nangangalaga ng kanilang kalusugan” ang ipinahihiwatig na katangian ay _______. A. malinis C. maliksi B. mabisa D. makapangyarihan 84. The stanza below is taken from “Barter” by Sara Teasdale Life has loneliness to sell, Music like a curve of gold, Scent of pine trees in the rain, Eyes that love you, arms that hold, And for your spirit’s still delight, Holy thoughts that star the night. To what does Teasdale compare music? A. The scent of pine trees C. Eyes that love

B. A curve of gold D. The rain 85. Which word ends with [S] pronounced [Z]? A. Maps C. Laughs B. Jokes D. Buys 86. Which of the following lines is a simile? A. “Holding wonder like a cup” C. “Eyes that love you, arms that hold” B. “Life has loneliness to sell” D. “Buy it ang never count the cost” 87. Which is the BEST WAY to write the underlined portion of this sentence? A person should keep in mind some basic safety rules when you are deciding whether or not to use a fire extinguisher. A. Rules you decide C. Rules you are deciding B. Rules when you decided D. Rules when deciding 88. What is meant by the expression TO GET BLOOD OUT OF A STONE in this sentence? Geraldo has owned me fifty thousand pesos for over a year now. I‘ve asked him for it on several occasions, but it’s like trying to get blood out of a stone. A. Something is impossible.

B. Someone refuses to cooperative. C. Someone is willing to give what is asked. D. Someone wants revenge. 89. What correction should be made to this sentence? First born often pattern their behavior after they’re parents and other adults. A. Replace their to they’re C. Replace they’re to their B. Change pattern to patterned D. No correction is necessary 90. What is meant by SOFT OPINION in this sentence? Rebecca realized that if she stayed in her present job it would mean competing with an envious rival. Leaving the company would probably be a soft option A. An action that is difficult to take C. An action that is easier B. An action that is not agreeable D. An action that is weakly funded 91. What correction should be made to this sentence? Most State tourism departments and some travel agencies have bed and breakfast listings. A. Insert a comma after agencies C. Change tourism to Tourism B. Change have to has D. Change State to state

92. What is suggested in the opening line? June 13, 1986-they came from all over America- 200,000 heroes strong, with their families. A. The writer holds great admiration for the veterans B. The writer was a veteran of the war C. The writer is opposed to the Vietnam War D. The writer is a flag-waving patriot 93. To gain the attention of the audience, the trick is __________. A. start low, speak hurriedly C. start high, speak rapidly B. start high, speak loudly D. start low, speak slowly 94. What correction should be made to this sentence? Recently, educators exammined the effectiveness of computer instruction in schools. A. Replace educators with educator’s B. Change the spelling of exammined to examined C. Change schools to Schools D. Replace computer with computer’s 95. Which is BEST WAY to write the underlined portion of this sentence?

There is smoke detectors in many homes to warm residents of a fire, but fire extinguishers can actually help people fight fires. A. They’re is C. Their are B. Their is D. There are 96. What do the following lines CONVEY? Midnight, not a sound from the pavement. Has the moon lost her memory? She is smiling alone. In the lamp light the withered leaves Collect at my feet And the wind begins to moan. A. Confusion C. Loneliness B. Optimism D. Eagerness 97. What correction should be made to this sentence? Most fire-related death’s result from households fries, yet many people do not have fire extinguishers in their homes. A. Remove the comma after fires C. Change have to has B. Change result to results D. Replace death’s with deaths 98. Which verb in the sentence is pronounced with the ending as [d]? They laughed and joked as they walked and played. A. Joked C. Walked B. Laughed D. Played

99. Which of the following words DOES NOT contains the [voiceless th]? A. Mouth C. Teet B.Breath D. Health 100. Which is the BEST way to write the underlined portion of this sentence? However, their VCR kept them from missing their favorite prime time shows. A. Keepes C. Had kept B. Keeps D. Keeped 101. Which is the BEST way to write the underlined portion of this sentence? Researchers also speculate that some teachers might have given boys more computer time because parents and teachers expected boys to need computers for future careers. A. Will expect C. Will have expected B. Expected D. Expecting 102. Which word contain the [ae] sound? A. Carriage C. Castle B. Cabin D. Can 103. What correction should be made to this sentence? One of their theories is that the first child receives more of the parents’ attention than other

children so first-borns tend to be more intellectual. A. Change is to are C. Change parents to parent’s B. Insert a comma after children D. Change theories to theory’s 104. What is meant by LAST DITCH in this sentence? The aged bishop prepared to fight to the last ditch to defend his good name. A. One’s last courage C. One’s last hope B. One’s last strategy D. One’s last defense 105. Which is the BEST way to write the underlined portion of this sentence? Recently psychologists have been researching birth order, their research suggests that personality and intelligence are based partly on where a child ranks in the family. A. Order, Their C. Order, or their B. Order and their D. Order, their 106. What is meant by TWO PINS in this sentence? For two pins I could have hit him on the nose. A. A second course of action C. The second chance

B. Without much persuading D. Have a second alternative choice 107. Which is the BEST way to write the underline portion of this sentence? The hosts also benefit from running such a business because they can stay at home make money, and meeting a variety of people. A. Get to meet C. And meet B. To meet D. And be meeting 108. Which is the BEST way to write the underlined portion of the sentence The studies revealing that, for various reasons, girls spent less time working with computers than boys. A. Revealing studies C. Studies’ revelations B. Studies revealed D. Studies will reveal 109. Which word contains the voiced Th? A. Thank C. Think B. These D. Thing 110. Which word is read on a high note to describe the kind of day? This is a cold day. A. Is C. Day B. Cold D. This 111. What is the mood of these lines?

Daylight, I must wait for the sunrise I must think of a new life And I mustn’t give in. When the dawn comes Tonight will be a memory, too And a new day will begin. A. Afraid C. Depressed B. Sarcastic D. Hopeful 112. Which is the BEST way to write the underlined portion of this sentence? However, if a fire extinguisher is handy, a quick-thinking person often can use them to put out small fire. A. They C. Them B. Him D. It 113. What correction should made to the sentence? Buying fire extinguisher knowing how to use it, and placing it in a location familiar to all family members can help protect families against fire. A. Insert a comma after extinguisher B. Change the spelling of families to familys C. Change placing to place D. Insert a comma after help 114. Which one is the right to human dignity? A. Choose the goals and means of development

B. Share in scientific and technological advances of the world C. Right to information D. Sovereignty over our natural resources 115. Which antidote would have a similar effect if vinegar or citrus juice were not available? A. Milk C. Vegetable oil B. Raw egg white D. Water 116. Which method of reproduction provides for the most variety of offspring? A. Cloning C. Asexual reproduction B. Sexual reproduction D. Cellular reproduction 117. Spouses Jose S. Luz and Celerina Luz filed a petition to adopt Gregorio Luz Ona, their nephew. The spouses are childless and they reared from his birth 1971 until 1975 and they continue to support him. Gregorio had to be left in the Philippines when the spouse went to the United States where Jose is employed. The MSSD recommended the adoption to the court on the premise that petitioners are in a better position to provide for the minor child than the natural parents who are impoverished.

What could be the possible decision of the court on he petition? A. The court denied the petition because the spouses are already aliens. B. The petition for adoption was granted because the court finds that it is to the best interest of the child. C. The petition for adoption was denied because the spouses are non-residents of the Philippines. D. The court denied the petition on the premise that the trial custody required by PD 603 cannot be effected for spouses are nonresidents. 118. The undeclared war in Korea most closely resembled the situation of A. The Spanish-American War C. World War I B. Vietnam War D. World War II 119. Which is the MOST important perceived need and problem of the Filipino family? A. Unemployment or financial problem B. Proveness to vices C. Protectiveness of children D. Double standard on the roles of male and female

120. Which has become a prototype of other schemes that defeated the real and true purpose of the CARL? A. The conversion of farmlands to industrial complexes B. The stocks option scheme of Hacienda Luisita C. The conversion from agriculture to subdivision D. The voluntary offer to sell 121. Carolina Diaz filed a petition for habeas corpus against Mr. and Mrs. Ramon Alde to recover custody of Lina Diaz Tan alias "Gracia Alde,” the natural daughter of Carolina Diaz, who was a hostess. What could be the possible action of the court on the petition filed by Carolina Diaz? A. Her petition would be granted because she now works as a clerk in a prestigious office. B. Her petition would be denied because she was a former hostess. C. Her petition would be denied because when Gracia was given to the Aldes it was tantamount to abandonment of the child, resulting to termination of parental authority. D. Her petition would be granted because she is the natural mother.

122. Why did the register of deeds charge Lucio Cruz registration fee the instrument relative to his loan? A. His loan was 30,000.00 C. His loan was P50,000.00 B. His loan was P60,000.00 D. His loan was P40,000.00 123. Which one BEST defines personal integrity? A. The unity between ignorance and reality B. The unity of man’s deeds, words, thoughts and realities C. The unity of mans social, political and physical aspects D. The relationship between virtue and conduct 124. Which of the following ethnocentric behavior? A. A tourist who lectures his foreign hosts on the “uncivilized” nature of their marriage customs B. A student who tutors an immigrant in English C. A Hispanic community group demands that public aid forms be published in English and Spanish D. A peace Corps volunteer who helps dig wells in Central Africa 125. Which one is the right to human dignity? A. Political independence B. Honor and reputation C. Form association

D. Social and economic reforms 126. Where would you commit a drug dependent for him to achieve a natural, tensionless, and anxietyfree state? A. NFPI C. DARE B. DARN D. Bukang Liwayway Center 127. Mary Rose, an 18 year old was sexually abuse by 3 teenagers from well-to-do families from Makati. Despite pressures, she came out into the open to get justice. Which need did Mary Rose satisfy? A. Need for family unity C. Need for universal solidarity B. Need for civic responsibility D. Need for personal integrity 128. 10 - 3 2/7 or 10-3 2/17= __________ A. 7 2/17 C. 6 1/17 B. 6 2/17 D. 6 15/17 E. none of these 129. At which time during the year does the ozone level present a particular health threat in urban areas for people with respiratory problem? A. Spring C. Summer B. Fall D. Winter 130. What day follows the day before yesterday if 2 days from now will be Sunday?

A. Tuesday D. Wednesday B. none of these E. Thursday C. Saturday 131. Which is the BEST evidence that helium gas is lighter than air? A. Helium has the lowest boiling point of all elements. B. Helium atoms do not combine with other air atoms. C. Helium-filled balloons rise in air. D. By volume, helium makes up only 0.0005% of air. 132. During a recent shopping spree, Tomas and Nena bought some new accessories for their apartment. Nena choose a crocheted throw pillow at P24.95, and Tomas purchased a rural landscape painting for P135.00. How much did they actually spend if they paid 7% sales tax on their purchases? A. P 171.15 D. P 159.95 B. P 139.25 E. P 148.75 C. none of these 133. Which location will have most nearly twelve hours of daylight and twelve hours of darkness during December? A town that is located A. halfway between the equator and South Pole B. close to equator C. close to the North Pole D. close to the South Pole

134. A nation in which loess would likely to be found today is no answer A. Iceland C. Japan B. United States D. Ecuador 135. It was also discovered that the chemical 3m butylphthalide can lower the blood pressure of rats. This statement is classified as ________. A. prediction C. finding B. experiment D. nonessential fact 136. Which is NOT among the hazardous effects of water pollution to health? A. The epidemic threat of hepatitis and dysentery B. The increase incidence of liver cancer C. The dumping of mercury in the sea causing blindness, brain damage or death D. The presence of certain bacteria in the digestive tract causing methemoglobinemia 137.Without the process of meiosis, we can infer that offspring from sexual reproduction would A. have a high degree of genetic variety B. have twice assigned number of chromosomes C. be identical D. have a number of mutations.

138. The Jones family has four children, all girls. The fifth child born is a boy. This change is the result of A. conception classes takes by the parents B. the timing of fertility cycles. C. the father’s contribution of a “Y” chromosomes. D. the “law of averages” finally catching up. 139. Which of the following will occur if a cold bottle of soda is left open on a kitchen counter? A. The pressure that the soda exerts on the bottle will increase. B. The temperature of the soda will decrease. C. The amount of dissolved carbon dioxide gas will decrease D. The amount of dissolved carbon dioxide gas will remain the same. 140. What do you predict will happen when you bring two bar of magnets closer together? A. They will repel each other. B. They will create an alternating current. C. Noing will happen. D .They will attract each other. 141. A tightly coiled spring demonstrate? A. Steam energy C. Potential energy

B. Kinetic energy D. Chemical energy 142. What does a stick of unlit dynamite demonstrate? no answer A. Chemical energy C. Kinetic Energy B. Nuclear Energy D. Potential Energy 143. Which is the main goal of drug abuse education? A. Arrest C. Control B. Prevention D. Rehabilitation Situation 1 – Below is the poem written by Edgar Lee Masters in 1915: At first I suspected something __________ She acted so calm and absentminded. And one day I heard the back door shut, As I entered the front, and I saw him slink Back of the smokehouse into the lot,And across the filed. And I meant to kill him on sight. But that day, walking near Fourth Bridge, All of a sudden I saw him standing, Scared to death, holding his rabbits, And all I could say was, “Don’t, Don’t Don’t,”

As he aimed and fired at my heart/ 144. Who is the speaker of this poem? A. Tom Merritt C. God B. Merritt’s wife D. The sheriff 145. The way in which the poet present these words in line 12 implies that Tom A. tried to annoy the other man. B. was shot before he finished the statement C. did not want to hurt the other man. D. begged the man to stop seeing his wife. 146. The poet introduces the poem with lines 1-3 to show us that Tom Merritt A. had suspected that his wife was seeing another man B. was sure that his wife was ill C. was a very suspicious person D. was sure that his wife still loved him 147. Which of the following techniques is used in the poem? A. Verse C. Free verse B. Rhyme D. Personification Situation 2 – below is an excerpt from John F. Kennedy’s “Inaugural Address.” Read the excerpt and answer the questions that follows In your hands, my fellow citizens, more than mine, will rest

the final success or failure of our course. Since this country as founded, each generation of Americas has been summoned to give testimony to its national loyalty. The graves of young Americans into answered the call to service surround the globe. Now the trumpet summons us again-not as a call to bear arms, though arms we need, not as a call to battle, though embattled we are; but a call to bear the burden of a long twilight struggle, year in and year out, “rejoicing in hope, patient in tribulation,” a struggle against the common enemies of many tyranny, poverty, disease, and war itself. Can we forge against these enemies a grand and global alliance, North and South, East and West, that can assure a more fruitful life for all mankind? Will you join me in this historic effort? In the long history of the world, only a few generations have been granted the role of defending freedoms in its hour of maximum danger. I do not shrink responsibility; I welcome it. I do not believe that any of us would exchange places with any other people with any other people or any other generation. The energy,

the faith, the devotions which we bring to this endeavor will light our country and all who serve it, and the glow from that fire can truly light the world. And so, my fellow Americans, ask not what your country can do for you; ask what you can do for your country. My Fellow citizens of the world, ask not what America will do for you, but what together we can do for the freedom of man. 148. One of the purposes of the speech is to motivate listeners to A. serve their country C. prepare themselves for battle B. enlist the armed forces D. preserve the right to bear arms 149. The speech is characterized by all of the following stylistic devices EXCEPT A. the use of the personal pronouns we and us to build rapport with listeners B. catchy turns of phase in which subjects and objects are inverted C. a standard, predictable rhythm and the use of rhyme D. the repetition of key words 150 The tone of the speech can BEST be characterized as A. sad C. light-hearted B. uplifting D. sarcastic

151. In the speech, Kennedy paints a picture of the United States as a nation that is A. longing to return to the past C. struggling to survive B. on the brink of world war D. the leading defender of freedom Situation 3 – Below is a Boigraphical sketch of an American movie writer: How did Elvis Presley Achieve Recognition Success often comes to those with humble beginnings. Elvis Aaron Presley was born on January 8, 1935 in Tupelo, Mississippi. He first sang in a church anf taught himself to play the guitar, but he never learned to read music. By 1953, he had moved to Memphis, Tennessee, graduated from high school. And enrolled in night school to become an electrician. That year, at Sun Records, Presley recorded a personal record for his mother, a song that was heard by the company’s president. As a result of the president’s recognition Presley’s first record “That’s All Right Mama,” was out in 1954. He toured the South, and in 1955 five of his record were released simultaneously. His first national television appearance was that year of Jackie Gleason’s “The

Stage Show.” But Presley became known for his appearance on “The Ed Sullivan Show,” where young singer gyrated as he sang “rock n’ roll” music. During the live television performance, Presley wad photographed only from the waist up because his motions were considered obscene. “Elvis the Pelvis” began his film career in 1956 with LOVE ME TENDER and signed a long-term film contract. The movie critics were not always kind, but teenagers flocked to Presley’s films. Within a few short years, Presley had established a career that would span twenty-five years of ups and down and make him one of the most popular entertainers in history. Long after his untimely death at age 42, Presley would be remembered as “The King of Rock n Roll.” 152. The author uses the phrase ups and downs to refer to Presley’s A. gyrations as performed B. increasing and decreasing finances C. successes and disappointments in his career D. use of drugs, “uppers and downers” 153. The main idea of the sketch is that

A. singers are more successful if they appear in films B. there has always been obscenity on television C. opportunity and luck are often as important as hard work D. celebrities are usually more famous after their death 154. The last sentence reveals that the author’s attitude toward Presley is one of A. indifference C. disbelief B. admiration D. disgust 155. The statement that ”success often comes to those with humble beginnings” would apply best to which of the following figures? A. Ramon Magsaysay C. Corazon C. Aquino B. Gloria M. D. Joseph Estrada Situation 4- The poem below is entitled “Suburban Prophecy” which is written by Howard Nemerov On Saturday, the powermowers’ whine Begins the morning. Over this neighborhood Rises the keening, petulant voice, begin Green oily teeth to chatter and munch the cud.

Monster, crawling the carpets of the world, Still send from underground against your blades The roots of things battalions greens and curled And tender, that will match your blades with blades Till the revolted throats shall strangle on The tickle of their dead, till straws shall break Crankshafts like camels, and the sun go down On dinosaurs in swamps. And night attack Follows and by the time the Sabbath dawns All armored beasts are eaten by their lawns. 156. To what does the phrase “your blades” in line 8 refer? A. Lawmowers C. Carpets B. Roots D. Monster 157. The poet’s use of words such as whine, voice, teeth, chatter and munch is to suggest that the power-mowers are A. very powerful C. like cows B. alive D. green 158. The imagery in the first stanza appeals to the reader’s sense of A. sight C. smell B. touch D. hearing

159. How long does the action of poet take place? A. A week C. An Afternoon B. Twenty-four hours D. A morning Situation 5 – Ang sumusunod ay isang talumpati na may pamagat na SA KABATAAN na isinulat ni Onofre Pagsanghan Isa sa mga salitang napagaralan natin sa wikang Pilipino ay salitang NABANSOT. Kapag ang isang bagay daw ay dapat pang lumaki ngunit ito’y tumigil na sa paglaki, ang bagay na ito raw ay NABANSOT. Marami raw uri ngpagkabansot ngunit ang pinakamalungkot na uri raw ay ang ng isipan, ng puso at ng diwa. Ang panahon ng kabataan ay panahon ng pagklaki, ngunit ang ating paglaki ay kailangang paglaki at pag-unlad ng ating buong katauhan. Hindi lamang ng atinmg sukat at timbang. Kung ga-poste man ang ating at ga-pison man angating bigat ngunit kung ang pag-iisip namat nati’y ga-kulisap lamang kay pangit na kabansutan. Kung tumangkad man tayong tangkad-kawayan at bumilog man tayong bilog-tapayan, ngunit kung tayo nama’y tulad ni “bondying” ay di pagkatiwalaan anong laking

kakulangan. Kung magkakatawan tayong katawang “Tarza” at mapatalas ang ating isipang sintalas ng kay Rizal, ngunit kung ang ating kalooban nama’y itim na duwende ng kasamaan anong kapinsalaan para sa kinabukasan. Kinabukasan, kabataan, tayo raw ang pag-asa ng inang Bayan. Tayo raw ang maghahatid sa kanya sa lagnit ng kaganaan at karangalan o hihila sa kanya sa putik ng kahirapan at kahihiyan. Ang panahon ng pagkilos ay ngayon, hindi bukas, hindi sa isang taon. Araw-araw ay tumatawid tayong palangit o bumabaluktod tayong paputik. Tamang-tama ang sabi ng ating mga ninunong kung ano raw ang kinamihasnan ay siyang pagkakatandaan. Huwag nating akalaing makapagpapabaya tayo ng ating pag-aaral ngayon at sa araw ng bukas ay bigla tayong maging mga dalubhasang magpapaunlad sa bayan. Huwag nating akalaing makapaglulublob tayo ngayon sa kalaswaan at kahalayan at sa mahiwagang araw ng bukas bigla tayong magiging ulirang mga magulang. Kabataan, tunay na pag-ibig sa bayan, ang tunay na nasyonalismo, ay wala sa tamis ng pagnarap wala rin sa pagpag ng

dila. Ang tunay na pag-ibig ay nasa pawis ng gawa. 160. Alin salita ang paulit-ulit na binabangit ni Onofre Pagsanghan? A. Nabansot C. Kabataan B. Bayan D. Kung 161. Sa alin makikita ang tunay na NASYONALISMO? A. Diwa C. Sulat B. Gawa D. Salita 162. Bakit di dapat tumangkad tulad ni “bondying”? A.Di ito mapagkakatiwalaan C. May kakulangan ito B.Di totoo ito D. Magulo kasi ito 163. Alin sa mga sumususnod ang mensahe ng taluimpati? A. Ang mataas na paniniwala at taimtim na pananalig ay kailangang taglayin upang ang hangarin sa buhay ay ating kamtin. B. Ang panahon ng kabataan ay panahon ng paglaki at pagbabagong makabuluhan. C. Ang gawa ang siyang sukat ng kadakilaan. D. Ang kabataan ay siyang pag-asa ng bayan. 164. Anong tayutay ang tinutukoy nito? Durog ang katawang bumagsak sa semento si Miguel A. Pagtutulad C. Pagmamalabis

B. Pagbibigay katauhan D. Pagwawangis 165. Anong aral ang ibinibigay ng sumususnod na salawikain? “Ang taong napapailalim ay naipapaibabaw rin.” A. Maaring ngayon ay hirap pagdating ng bukas ay may ginhawa rin B. Tiyak ang pag-unlad kapag nauna ang hirap C. Kung ano ang ibig natin ay mangyayari D. Magtiis kung dumarating ang hirap. 166. Sa aling salita magkakaroon ng saglit na paghinto kung pinapilitang si Rose ang nakabasag ng pinggan? Hindi si Rose ang nakabasag ng pinggan. A. Rose C. Hindi B. Pinggan D. Nakabasag 167. Alin ang naayong pamagat sa tanagang sinulat ni Jose Villa Panganiban? Ano man sa daigdig, Maaring magamit, Ano mang masaisip; Di sukat maiipit. A. Pagkainip C. Pag-asa B. Paraya D. Pagbibigay 168. Ano ang ipinahihiwatig ng salitang may salungguhit?

Matayog ang lipad ni Pepe kaya’t bata pa siya’y nagsisikap na siya. A. May kayabangan si Pepe. C. Marunong si Pepe. B. Mataas ang pangarap ni Pepe. D. Ibig ni Pepeng maabot ang langit. 169. Alin antas ng tono ng lumilitaw sa bahaging may salungguhit ng pangungusap na nagdududa? Nagpuputol ng puno ang lalaki. A. 1 C. 4 B. 2 D. 3 170. Alin ang kahulungan ng AGAW-BUHAY? A. Masiglang-masigla C. Pagpapatuloy ng buhay B. Malapit ng mamatay D. Mahirap na buhay 171. Kaninong tula hango ang sumusunod? “Ang hindi magmahal sa sariling wika mahigit sa hayop at malansang isda” A. Jose Rizal C. Apolinario Mabini B. Emilio Jacinto D. Graciano Lopez Jaena 172.Ano ang pinakaangkop na kahulugan nito?

“Nagsasaya tayo ngayon sapagkat ang inyong namatay na kapatid ay muling nabuhay; ang nawawala ay muling nakita.” A. Ang pagbabalik ay dapat ipaghanda nang malaki. B. Ang pagbabago ng kapatid ay dapat pahalagahan. C. Dapat silang magsaya sa muli nilang pagsasama-sama D. Ang pagsasama nila ay dahil sa muling pagbabalik ng kapatid. 173. Alin ang kahulugan ng KAHIRAMANG SUKLAY? A. Kakilala C. Karibal B. Kaibigan D. Kalahi 174. Alin sa mga sumusunod ang aral na ibinibigay ng epikong Muslim na INDARAPATRA AT SULAYMAN? A. Pagmamahal C. Katapatan B. Katapangan D. pagtanaw ng utang-na-loob 175. Alin sa mga salita ang kasingkahulugan ng salitang may salungguhit? Ang ama ni Anita ay kilalang bulanggugo sa kanilang lalawigan A. Laging ibinubulong C. Laging handang makipag-away B. Laging handang gumasta D. Laging handang makipagtalo

176. Si Mariano Ponce ay propagandistang may sagisag sa panulat na ________. A. Tamaraw C. Kapre B. Tikbalang D. Kalapate 177. Alin ang di karaniwang anyo ng pandiwang HINTAY KA? A. Tay C.Intay B. Tayka D.Teka 178. Ano ang kahulugan ng taludtod na ito “Ang anak mo ay alagaan sa marubdob na pagsuyo sikapin mo sa sarili’y huwag siyang maging luko talipandas sa paglaki na sa sama marahuyo sa lahi mo’t sa Bayan moy isang tinik sa balaho.” A. Mahalin ang anak ng walang hangganan. B. Tamang pagpapalaki sa anak ang dapat. C. Suyuin ang anak at ibigay lahat ng hilig. D. Paligayahin ang tahanan. 179. Which is the BEST way to write the underlined portion of this sentence? Many viewers taped shoes to watch later. A. Tapped C. Tape B. had taped D. Had tapped 180. Which word in the passage does NOT require a change in pitch to show confidence

“I am the master of my fate, I am the captain of my soul.” A. Am C. Master B. Captain D. Fate 181. Which pitch is used for the word STRANGE in this sentence? What a strange story! A. 3 C. 4 B. 2 D. 1 182. Carl Sundburg wrote “Jazz Fantasia” which has for its first stanza: Drum on your dreams, better on your bajos, sob On the long cool winding saxophones.Go to it, O jazzmen. Which words illustrate alliteration A. Batter and banjos C. Sob and winding B. Long and cool D. to and it 183. Anong uri ng panghalip ang salitang may salungguhit sa pangungusap? May padalang tulong ang pamahalaan para sa kanila. A. Pambalana C. Paari B. Palagyo D. Palayon 184. Alin uri ng parirala ang may salungguhit sa pangungusap? Utang sa kanyang sipag at sikap sa paggawa ang kanilang maalwang pamumuhay A. Pangngalan C. Pawatas

B. Pangngalanng-diwa D. Pangukol 185. Sabihin ang aspekto ng pandiwa sa pangungusap na ito. ”Mag-aral sa bahay ng mga araling ukol sa halaman.” A. Pawatas C. Imperpektibo B. Kontemplatibo D. Perpektibo 186. Kilalanin ang uri ng pariralang may salungguhit. Ang pangangalaga sa mga likas na yaman ay tungkulin nating lahat. A. Pangngalan C. Pang-ukol B. Pangngalang-diwa D. Pawatas 187. Si Dr. Jose Rizal ay sumulat ng aklat ng itinampok sa ibat ibang bansa. Ang pangungusap ay nagagamit bilang _______________. A. panuring C. tuwirang layon B. pamuno D. paksa 188. Lines 11 and 12 are taken from the poem “maggie and milly and molly and may.” For whatever lose (like a you or a me) It’s always ourselves we find in the sea Which of the following ideas is the author expressing?

A. The sea is a source of life and death. B. The sea represents all of our moods. C. The sea is the best place for a person to reflect about life. D. The sea and its surroundings can give people a fresh view on life. 189. Nasa anong kaganapan ng pandiwa ang pangungusap? Naglaro ng basketball sa Rizal Stadium ang koponan ng aming pamantasan. A. Sanhi C. Kagamitan B. Tagaganap D. Ganapan 190. Alin antas ng tono ang lumitaw sa bahaging may salungguhit ng pangungusap na nagsasalaysay? Magbabasa ng mga gawain ang guro A. 1 C. 2 B. 3 D. 4 191. Dadalaw sa mga paaralan si Dr. Filemon S. Salas, ang tagapamanihala ng mga paaralang lungsod, sa lungsod ng Pasay. Ang pangungusap ay nagagamit bilang _____________. A. panuring C. tuwirang layon B. paksa D. pamuno

192. Alin sa mga sumusunod ang aral na ibinigay ng ANG ALAMAT NI MARIANG MAKILING na ikinuwento ni Jose Rizal? A. Pagyamanin at pangalagaan ang ating bayan at lahi pagkat hiyas at yaman natin ito. B. Pag-ibig ang makapagbabago sa mundong ito. C. Kabanalan ang magpatawad at tulungan ang isang nagkasala. D. Dahil sa pagmamalabis at pagsasamantala, maraming biyaya ang sa kanyay nawawala. 193. A Politician wants to get his message to 2/3 of the population of 48,000 in Bulacan. However advertising campaign reaches only 3/ 4 of the number he intended. How many people does he actually reach A. 16,000 C. 24,000 B. 10,000 D. 36,000 E. none of these 194. Alin sa mga sumusunod ang mensahe ng epiko ng Ilokano na BIAG NI LAM-ANG? A. Pinatutunayan ng epiko ang yaman ng Ilokano sa lahat ng bagay. B. Kailangan paniniwalaan ang ukol sa bisa ng mga anting-anting dahil

sa mga pangyayaring nagpapatunay dito. C. Dito nagpapatunay na walang kamatayan. D. Masasalat ang mga katutubong ugali at mga tradisyong dapat pagyamanin at panatilihin upang pakinabangan ng kabataan. 195. Alin ang di karaniwang anyo ng pandiwang WINIKAKO? A. Ikako C. Kako B. Wikako D. Wika ko 196. The Miranda Family purchased a 250-pound side of beef and had it packaged. They paid P365.00 for the side beef. During the packaging, 75 lb of beef were discarded as waste. What was the cost per pound for packaged beef A. P 2.08 per lb D. P 2.06 per lb B. none of these E. P 2.30 per lb C. P 2.50 per lb 197. Which is the sum of the infinite progression 3/2. 1, 2/3, 4/9…? A. 6 1/2 C. 4 ½ B. 5 ½ D. 7 ½ E. none of these

198. What indoor relative humidity range would probably be comfortable to preventing temperature and humidity levels are extremely low? A. 90% to 100 % C. 20% to 30 % B. 60% to 70 % D. 30% to 40 % 199. As a representative of the Urban Poor Commission of the Association of Religious Superiors (ARS), which action will you most likely take to resolve the long-term roots of structural inequalitiesproliferation of child labor and child prostitution?

A. Raise views of human rights abuse. B. Organize regular programs for information and discussion of human rights. C. Conduct skills training. D. Raise questions over government’s commitment to rebuild human rights. 200. What are the next four numbers in this sequence 8,5,4,9,17_____,_____,____ A. 4,3,2,1 D. none of these B. 5,4,3,2 E. 3,2,1,0 C. 6,3,2,0

8. ANSWER KEY GENERAL EDUCATION

9.

Secondary 1. 2. 3. 4. 5. 6. 7.

C 151. A 152. B 153. D 154. D 155. B 156. C 157.

10. 51.

B

101.

A

52.

A

102.

B

53.

A

103.

B

54.

A

104.

D

55.

C

105.

A

56.

B

106.

C

57.

D

107.

C

D

11.

C

12.

C

13.

B

14.

A

15.

NO ANSWER NO ANSWER

16.

B 158.

NO ANSWER

A 159.

NO ANSWER

C 160.

C

D 161.

A

A 162.

C

C 163.

D

D 164.

C

C 165.

A

C 166.

C

58.

A

108.

B

59.

B

109.

A

60.

A

110.

B

61.

C

111.

D

62.

D

112.

D

63.

A

113.

A

64.

B

114.

D

65.

D

115.

A

66.

A

116.

B

17. 18. 19. 20. 21. 22. 23. 24. 25. 26. 27. 28. 29. 30. 31. 32. 33.

B 167.

C

67.

C 168.

B

D 169.

C

B 170.

B

C 171.

A

A 172.

B

A 173.

B

C 174.

A

B 175.

B

D 176.

B

D 177.

A

C 178.

B

B 179.

C

A 180.

D

D 181.

A

B 182.

C

A 183.

C

68 69. 70. 71. 72. 73. 74. 75. 76.

E A A C E C A B B A

117. 118. 119. 120. 121. 122. 123. 124. 125. 126.

C B A A D NO ANSWER D C B C

34. 35. 36. 37. 38. 39. 40. 41. 42. 43.

B 184.

A

A 185.

B

E 186.

B

C 187.

D

C 188.

D

A 189.

D

A 190.

B

D 191.

B

C 192.

D

A 193.

C

77.

A

127.

D

44.

C 194. D

78.

A

128.

D

45.

C 195.

D

129.

C

46.

A 196. A

80.

C

130.

E

47.

C 197.

E

B 198.

?

A 199.

D

D 200.

D

82. 83.

A A D

131. 132. 133.

C A B

48. 49. 50.

B

134.

?

85.

D

135.

C

86.

A

136.

C

87.

D

137.

B

88.

A

138.

C

89.

C

139.

C

90.

C

140.

B

91.

D

141.

C

92.

A

142.

?

93.

B

143.

B

94.

B

144.

D

95.

D

145.

A

96.

C

146.

A

97.

D

147.

C

98.

D

148.

A

99.

D

149.

B

100.

B

150.

B

C

79.

81.

84.

PART II: ANALYZING TEST ITEMS Direction: Encircle the letter of the best answer. 1. The following statements refer to the characteristics of the Philippines as one of the important territories of Asia, except for one A. It is a tropical country and geographically located in the northeastern part of the world. B. Philippines has one of the highest literacy rates in Asia with existing policy of free and compulsory six years in elementary. C. The establishment of the first Philippine Republic gave birth to the first constitutional democracy in Asia

D. The free port of Manila is one of the world’s busiest ports, compared with other Southeast Asian nations 2. Globes are essentially important tool in the study of Geography. A map is more convenient to use but all map projections have some errors in presentation of distance and shapes. Given these conditions, which among the known mapping projections will we use to give the true size and shape of the earth’s land masses? A. Azimuthal Mapping C. Equal Area Mapping B. Conformal Mapping D. Equidistant Mapping 3. Most scientists believe that El Niño phenomenon which was greatly experiences in 1982, 1983 occurred when trade winds that normally blow from east to west slow down and actually change direction. The winds blow warm water toward South America where the normal weather patterns are changed for a time. Given this global change, which among the following conditions is not considered as direct effect of the El Niño phenomenon?

A. Occurrences of hurricanes and heavy rainfall in the mid-Pacific region in a normal condition B. Some species of fish like cold-water loving salmon disappeared along the west coast of the United States but tuna and shrimp were attracted to the warm waters C. The gradual warming of the earth is known as the Greenhouse effect could dramatically change climates worldwide D. Water temperature in some parts of the Pacific Ocean rose to as much as 14 degrees higher than to normal condition 4. China has over one fifth of the world’s population and its leaders believe that its large population hinders the economic progress of the country and to address this problem they want to achieve equal death rate and birth rate in population. This government action to limit the population is known as the A. Baby Boom C. Population pyramid B. Population distribution D. Zero population growth 5. Given the current condition of Africa, it is still considered as the most rural and least urbanized continent in the world. With these

characteristics, which among the following sentences is the least contributory factor to the current socio-political and economic problems of Africa? A. Few states are ethnically homogenous and infancy of a strong sense of national unity among tribal leaders B. Insufficient capital technology, political instability and poorly trained workforce to push development among the African nations C. The increasing percentage of school-age children who did not attend school low literacy rate and a grate number of world’s AIDS deaths D. The replacement of Christianity among the Islamized society in Africa along with the continuous practice of animism among few villages 6. The biggest and most populated island in the Philippine Archipelago A. Cebu D. Mindanao B. Leyte E. Negros C. Luzon 7. As of 1996, what sector of Philippine economy shows an increase near to 50%? A. Agricultural Sector D. Service Sector

B. Energy Sector E. Tourism Sector C. Mining Sector 8. Which of the following is not included in the main geographical and political-cultural subdivisions of Asia? A. North Asia D. Southeast Asia B. Northwest Asia E. Southwest Asia C. South Asia 9. This continent is the largest and it covers almost 33% of the earth’s surface. It is consider as the most diverse continent and home for earliest civilization and major religions of the world A. Africa D. Europe B. Asia E. North America C. Austria and Oceania 10. Which area of the world has been described as the “symbol of worldwide demographic, environmental and societal stress/problems”? A. Africa D. North America B. Asia E. South America C. Austria/Oceania 11. Which of the following continent is considered as the world’s most productive in agriculture? A. Africa D. North America

B. Australia and Oceania E. South America C. Europe 12. If the Caribbean Islands may be combined with Mexico and Central America, it could be collectively and properly described as: A. Latin America D. South America B. Middle America E. The Americas C. North America 13. The name “Oceania” is widely used to refer to the scattered group of islands A. Central and Southern Pacific Ocean D. Micronesia (Little islands) B. Central and Western Pacific Ocean E. Polynesia (Many islands) C. Melanesia (Black Islands) 14. What country is considered as largest producer of Diamond, although few of its diamonds are of gem quality? A. Australia D. New Zealand B. Canada E. South Africa C. Mexico 15. A.

The world’s largest island is: Australia D. Greenland

B. China E. Madagascar C. Iceland 16. What sea is often included in the physical definition of North America? A. Baltic Sea D. Read Sea B. Caribbean Sea E. South China Sea C. Mediterranean Sea 17. It is the longest mountain chain in the world A. Andes D. Kunlun B. Himalayas E. Pamir C. Karakoram 18. What religious group suffered heavily in Europe during the Nazi Holocaust? A. Atheist D. Jewish B. Buddhist E. Muslim C. Christian 19. Most of the people in Latin America during the colonial period came from what known place in the world? A. Amazon Rainforest D. Oceania B. Andes Mountain E. Scandinavian C. Iberian Peninsula 20. It is considered as one of the worst eruption of the century that almost affects the earth’s atmosphere due to its ash falls

A. Mt. Hibuk-Hibok D. Mt. Pinatubo B. Mt. Kanlaon E. Taal Volcano C. Mt. Mayon 21. Which part of the earth where we can experience six (6) months of continuous daylight and six (6) months continuous darkness each year? A. Greenland D. North Pole B. Greenwich, England E. Pacific Ocean C. Iceland 22. In what part of the world can you approximately find the International Date Line (IDL)? A. Atlantic Ocean D. Pacific Ocean B. Greenwich, England E. Pole C. North Pole 23. In what part of the world can you find the longest and largest coral reef which is also known s the “Great Barrier Reef”? A. Africa D. North America B. Australia E. South America C. Europe 24. It is a common knowledge that all bodies of water are interconnected with one another but this ocean covers 1/3 of the world from Arctic to Antartica

A. Arctic Ocean D. Pacific Ocean B. Indian Ocean E. South Atlantic Ocean C. North Atlantic Ocean 25. Which of the following countries is not part of the Association of the South East Asian Nations (ASEAN)? A. Malaysia D. Taiwan B. Philippines E. Thailand C. Singapore 26. Among the South American nations, this country is the only Portuguese speaking country A. Argentina D. Mexico B. Brazil E. Peru C. Chile 27. The first African nation that tried to establish industrialization to lessen the worsening unemployment rate of the country A. Bostwana D. South Africa B. Egypt E. Zaire C. Kenya 28. It is known to the ancient and modern world that this is the home of rich and key cities of the world. It is also of the major tourist destination and it provides a well

designed airports and railways across the continent A. Africa D. Europe B. Asia E. South America C. Australia 29. The opening of the Suez Canal connects the trading route between these two bodies of water A. Atlantic Ocean and Pacific Ocean D. Mediterranean Sea and Red Sea B. Caribbean Sea and Gulf of Mexico E. South China Sea and Philippine Sea C. Indian Ocean and Bay of Bengal 30. This is the known Peninsula in Europe that occupied by Norway, Sweden, Denmark also included is Iceland and Finland A. Apennines D. Jutland B. Balkan E. Scandinavian C. Iberian PART III: ENHANCING TEST TAKING SKILLS Direction: Encircle the letter of the best answer. 1. Map projections have particular purposes; specifically what do we call projections that show all areas on Earth’s surface in proper proportion to visualize

patterns of distribution with spatial area? A. Albers conic projection D. Lamber azimuthal projection B. Equal area projection E. Mercator projection C. Gnomonic projection 2. When these imaginary lines are used in combination, it gives you a unique destination to every point on Earth? A. International Date Line (IDL) D. North Pole and South Pole B. Lines of Longitude and Latitude E. Prime Meridian & Equator C. North, East, West and South Hemisphere 3. Earthquakes and volcanic activities were frequently experienced by the Philippines due to: A. Climatic change in Asia for a long period of time B. Erosion caused by heavy winds and typhoons all year round C. High and low humidity brought by Northeasterly and Southeasterly wind D. Short, sharp and shift flowing rivers from Luzon to Mindanao E. Unstable Pacific floor under the Philippine territory 4. Manila has a good medical facilities but there is a great need

for medical people specially in the rural areas, the main factor for this situation is: A. Great exodus of medial practitioners like Nurses, Medical Technician and Medical Doctors going abroad B. Low quality of graduates that cannot passed both local and international standards for medical practices C. Political unrest and security of rural hospital across the nation due to terrorism and local conflicts D. Small number of medical graduates like medical doctors and nurses to support the needs of the country E. Traditional medicine is a strong competitor for the scientific advancement of our Medicinal Science in rural areas 5. The “Rood of the World” may be associated to: A. Diverse ethnic group of Asia B. Frequent occurrence of earthquakes and volcanic eruptions C. Numerous islands, arranged in a series of arcs D. Series of high mountains and plateaus E. Super continent Pangaea and its cultural heritage

6. In the 19th century, most Europeans considered Africa as “The Dark Continent” because: A. Most African nations had been subjected to European domination B. Most inhabitants are literally black or dark people of this continent C. Most of its economic and political activities were dependent to industrialized nations D. Most of the African land area is covered by vast desert and rugged mountains E. Most of the African interior was unexplored and not colonized by European and other neighboring nations 7. The following statement refers to the general characteristics of African people in terms of their cultural traits except for: A. Few nations have developed a strong sense of national unity among its people B. Imposition of colonial boundaries among African people further divide the continent C. Indigenous churches brought common traditions and practices among African people D. Traditional values prevailed over the African Tribes

E. Various languages further perpetual tribal identities over and above national identity 8. The population statistics as of 2004 shows evidences that population explosion will bring: A. A 100% increase by 2050 in terms of population B. Africa to its greatest annual increase among the other continents C. Declined life expectancy in developing countries due to famine and diseases D. One child policy to Asian nations E. Stability in terms of population growth rate 9. One of the major issues between US and Mexico which led US government to take action against the growing population of Mexico. A. Assist rapid economic growth of Mexico to eliminate poverty B. Control the great demand of farmers for the south flowing Colorado River for dry region C. Control the increase of illegal crossing of Mexicans to the US border in search of employment D. Manifest a virtual veto power of US towards Mexico’s economic policy

E. Support peasant rebels stage war against the state 10. What do you call the new racial type created due to intermarriages of most Caucasian and African slaves brought to Brazil and Colombia? A. Aborigine D. Mulatto B. Indian E. Native American C. Mestizo 11. Which of the following is not a human activity that threats the incredible biological diversity of the Amazon Wildlife? A. El Niño Phenomenon D. Massive deforestation B. Development of rural settlement E. Road construction C. Increased in demand for lumber 12. The word “Narcotraficantes” is associated with: A. Annual occurrences of El Niño Phenomenon B. Colombia’s illegal drug trade C. Destruction of the Amazon Forest D. Trading of Black slaves from Africa E. Volcanic and earthquake activities in the Andes mountains 13. Which of the following reasons why people cannot

establish a permanent settlement in Antartica? A. Danger of nuclear weapons testing over the territory B. Environmental problems like oil spills C. It is the coldest place on earth and experiencing the strongest wind D. Ozone concentration in the area above the continent E. The growing controversy over the claims of different nations over the territory 14. The three leading financial centers of the world included the following key cities of: A. Amsterdam, Frankfurt, Seoul D. Hong Kong, Paris, Rome B. Beijing, Sydney, Washington DC E. London, New York, Tokyo C. Berlin, Mexico City, Singapore 15. The leading industry both for value and employment of many people in Australia. A. Electronics & information technology B. Food processing C. Forestry and fishing D. Production of machinery for transportation E. Tourism 16. Chernobyl will be remembered as

A. Commonwealth of Independent States B. Economic bloc of former USSR to European Community C. Place of the nuclear reactor explosion D. Site of the 1992 Olympics E. Space launcher of Russian Cosmonauts that blasted 17. The following are general characteristics of an industrialized country except for one. A. Economic and political stability B. High literacy rate C. Longer life expectancy D. Low infant mortality E. One child policy per family 18. Below are listed 5 continents 1 Africa 4 North America 2 Antartica 5 South America 3 Europe Arrange the continents from biggest to smallest in terms of land area. A. 31254 D. 4 3 2 5 1 B. 23415 E. 5 2 4 1 3 C. 14523 19. Based on the early historical accounts of Europe these two nations had strong ties with the

European culture and the Asian nations. A. Australia and New Zealand D. North Korea and South Korea B. India and Sri Lanka E. Turkey and Russia C. Iran and Iraq 20. Among the 5 continents below 1 Africa 4 Australia/Oceania 2 Antartica 5 Europe 3 Asia Arrange the continents from smallest to biggest in terms of its population based on the estimated 2004 data A. 53124 D. 1 3 4 2 5 B. 31452 E. 4 5 1 3 2 C. 24513 21. The following statements refer to the African condition that hinders its potential towards economic growth except for one: A. Diverse natural resources B. Insufficient capital technology C. Political instability D. Poorly trained workforce E. Small purchasing power 22. Which of the following Asian countries has historical ties with European city and it was

considered as the heart of the Byzantine and Ottoman Empire? A. Iraq (Mesopotamia) D. Saudi Arabia B. Israel E. Turkey C. Russia 23. The following occurrences are all natural hazards that the world is experiencing across the time except for one: A. Deforestation D. Tsunami

B.

Diastrophism E. Volcanic Eruption C. El Niño and La Niña 24. This is known super continent that broke up million and million years ago. A. Atlantis D. Pangaea B. Gondwanaland E. Tethys C. Laurasia 25. Among the developing Asian nations, it is widely promoted to be

one of the better solution to poverty and unemployment problems A. Cooperation D. Privatization B. Importation E. Urbanization C. Industrialization

PART III: Enhancing Test Taking Skills KEY TO CORRECTION PART II: Analyzing Test Items

1.

B2.

B

1.

D 2.

C

3.

E4.

A

3.

C4.

D

5.

D6.

E

5.

D6.

C

7.

C8.

B

7.

D8.

B

9.

C10. D

9.

B10.

A

11.

A12.

11.

D12. B

13.

C14. E

13.

A14.

A

15.

B16.

C

15.

D16. B

17.

E18.

C

17.

A18.

D

19.

E20.

C

19.

C20. D

21.

A22.

E

21.

D22. D

23.

A

23.

B24.

D

24.

D

25.

D26. B

25.

C

27.

D28. D

29.

D30. E

B

PRACTICE TEST I 1. The adoption of scientific techniques to control and manipulate environment such as modern methods for farming and fishing is called A. social change C. cultural change B. technological change D. discovery change 2. Heavily sanctioned folkway which no one dares to question because they embody moral laws are called A. beliefs mores B.

C.

customs

D. tradition

3. What is that inevitable folkway which no one dares to question because they embody moral laws are called A.

change

C. progress

B. development evolution

D.

4. Nora Aunor became a legend in the showbusiness because of her talent, determination and hard work. What role does this illustrate?

A. achieve role hereditary role

C.

B. ascribed role temporary role

D.

5. Pinning veil around the bride and the groom/exchange of rings, releasing of doves, etc. during wedding ceremony show that culture is A.

symbolic

C. historical

B.

continuous D. changeless

6. Aling Maria has taught her daughter how to prepare good and delicious desserts from local fruits and vegetables. What cultural transmission process is this? A. observation enculturation

C.

B. acculturation indoctrination

D.

7. Mr. Francisco was teaching something on mores, folkways, customs and beliefs in his Social Studies class. What was his lesson about? A. material culture C. tangible culture B. non-material culture D. hereditary culture

8. It was the first day of school. Miss Dela Rosa prepared activities which will make her Grade III children sing, play, talk, learn and introduce themselves to the class. What process did the teacher emphasize? A. enculturation indoctrination

C.

B. socialization acculturation

D.

9. Mildred, an accountant, who now lives with a married man was disowned by her family and friends. This punishment is a form of A. isolation B. deportation assassination

C. ostracism D. character

10. When a certain culture is acceptable to one group and questionable to others, it shows that: A. culture is borrowed C. culture is varied B. culture is a shared product D. culture is relative 11. The teaching and learning of religion and values mostly make use of

A. enculturation ulturation

C.

B. indoctrination acculturation

D.

12. In order to achieve the goals for change, all include the following strategies EXCEPT A. Change must involve all sectors of the society B. Change must be centralized where only a few initiate the change

14. Central to cultural renewal and social change is a revolution of expectations from ourselves as well as from our institutions. This concept includes the following resources from Filipinos, EXCEPT: A. Assertiveness of Filipino families B.

A colonized education

C.

A cultural-fair media

D. A religious that upholds justice and righteousness

C. Change must be holistic that focuses on the individual and the system

15. Which of the following factors is the reason for communication gap in many homes?

D. Change should come from the leaders and the people

A.

Lack of parental guidance

B.

Modern technology

C.

Inadequate education

D.

Highly urbanized lifestyle

13. Which change is bought about by discovery or modernization to increase production?

D. It is flexible that suits to every human individual’s needs 17. The goals of Values Education that need to be changed include all the following EXCEPT: A. To become aware of the National Issues and Problems that beset the country B. To encourage OFWs to continuous work abroad in order to help the National Economy. C. To develop deep sense of responsibility and accountability D. To initiative action-centered community organizations 18. Which of the following is NOT an example of economic values? A.

Hard work and frugality

C. Self-determination B.

Work values

A. cultural change C. technological change

16. Which of the following statements regarding the Values Conceptual Framework is TRUE?

B. societal change D. social change

A. It is imposing on every student

19-20. Which among the strategies show negative concerns for environmental protection?

B. It is prescriptive of many desirable traits

A. Campaign against wrong health and environmental practices

C. It is specific for practical usefulness

B. Suggest the use of synthetic substances instead of organic ones

D. Responsible parenthood

C. Link with organizations with projects on waste management D. Cut tall trees and plant ornamentals to beautify the surroundings

A. There is peace. progress.

21. Filipino’s love for socials, fiestas and celebrations is manifestations of which Filipino’s character? A. Spirituality Superstitious

3. What will surely takes place whenever society adapts modernization abruptly?

C.

B. Family Solidarity D. Joy and Humor

C. There is

B. There is harmony. confusion.

C. culture

B. severe headache loneliness

D. extreme

PRACTICE TEST II

5.

1. An organized and systematized manner of learning where expectancies are controlled by the teacher is termed as:

A. Handing down of culture by a teacher

A. teaching schooling

C.

B. education motivation

D.

A.

home

B. school society

What is enculturation?

B. Handing down of culture without any question C. Handing of culture from one generation to the next

2. A social agency that makes a child learn to value himself and eventually others is C. group D.

D. Handing down of culture from one who knows to somebody who doesn’t know 6. Which of the following functions cannot be done by the school? A.

changing cultural practices

C.

socialization among children

D. development of attitudes and skills

D. There is

4. When a person fails to adjust to the fast changing world, he is likely to suffer from A. culture shock change

B.

perpetuation of specie

7. Filipinism could be taken as having the same connotation as a sense of A. responsibility nationhood B. professionalism philosophy

C. D.

8. The process of individual and group change caused by contact with various people A. acculturation culturation

C.

B. enculturation indoctrination

D.

9. All people have culture and therefore share a common humanity. This shows that A.

culture is universal

B.

culture is organized

C.

culture is a product of change

D. culture is the sum total of human experiences

10. Which of the following sciences below is more connected with the study of social traditions? A.

theology

C. sociology

B.

psychology D. anthropology

11. Which of the following situations presents a value conflict? A. The parent expects her child to do what he tells him; the child obeys so. B. The parent wants to his child to be in the Dean’s list; the child studies hard. C. The child has many friends; the parent allows him to spend time with them during week-ends D. The parent’s priority is for her child to be an A student, the child’s predominant interest 12. Why can’t the school implement programs for social reconstruction? A. Most of the students are poor and passive

D. Most teachers belong to Low Socio Economic Status (SES) 13. Which of the following best illustrates a sense of responsibility that a professional teacher is expected to have? A.

Is humble

C. Has pleasant disposition B.

Makes good use of time

D. Tutors own student for a fee 14. When an individual or a group adapts the culture of others, practice them and becomes habitual, this is: A.

culture lag C. culture shock

B. culture change D. culture difference

15. The proper sequencing of grade levels according to chronological ages of learners is called A.

status

C. agency

B. Some teachers do not like teaching career

B. organization institution

C. Some teachers have no dedication to the profession

16. Divorce is legal in the United States but is not acceptable in the Philippines. What does it show?

D.

A. culture is illegal C. culture is relative B. culture is uniform D. culture is phenomenon 17. A school operates not only to cater her students but for all the people where it is located can be called a A. public school school

C. societal

B. private school community school

D.

18. What is the responsibility of a newspaper editorial in values processing? A. Interprets the news for the sale of readability B. Analyzes the relevance of the personal experience with the news material C. Explains the message of the editorial D. Introduces socialized recitation 19. When does the “Kanyakanya” syndrome become positive? A. When one can discover what he likes in life

B. When the benefit of others is viewed as one loss

B. Emphasize on the effective component of learning

C. When one protects the individual interests

C. Emphasize on the processing rather than on concept

D. When one becomes selfreliant and can stand on his own.

D. Concentrate on the concept, process and behavioral components of value

20. Which situation shows a desirable relationship between teachers and other groups of people? A. Miss Sison issues report cards only if she remembers. B. Mr. Fajardo never entertains parents’ complains; he feels they are only a pest C. As a class adviser, Mr. Magalang has not had a single conference with the parents of his students. D. Mrs. Vergara is glad that the parents of her Grade III class volunteer to her in certain class activities 21. Does valuing process used as a teaching strategy? A. Focus on the affective processing of value learning

22. When statement can be considered features of our present educational system? A.

Knowledge is power

B.

Knowledge does not change

C. Knowledge begins in the senses D. Knowledge can be known only by reason 23. When does a teacher of Values Education a good facilitator of learning?

24. “One cannot give what he does not have” is a popular statement which presupposes the following: A. Facilitators of values must grow in their own personal total development B. Values education is a lifelong process C. The home is the primary source of basic values D. The school has the sole responsibility to educate the youth

25. Which is not an anthropological view of education? A. Education is a social institution

A. Monitors action learning plans for the students

B. Education is the process of teaching and learning

B. Provides cognitive inputs such as lecturette

C. Education is an agent of change and modernization

C. Clarifies the students values by the use of processing questions

D. Education is boundary breaking between social classes

D. Imposes her own beliefs on her students

KEY TO CORRECTION PRACTICE TEST I 1.

B

2.

C

3.

A

4.

A

5.

A

6.

C

7.

B

8.

B

9.

A

10.

D

11.

B

12.

D

13.

C

8.

A

14.

C

9.

A

15.

A

10.

D

16.

D

11.

D

17.

B

12.

A

18.

D

13.

B

19-20. B, D

14.

B

21. D

15.

B

PRACTICE TEST II

16.

C

1.

C

17.

D18. C

2.

B

19.

D

3.

D

20.

C21. D

4.

A

22.

C23. C

5.

C

24.

A

6.

A

25.

B

7.

C

Licensure Examination for Teachers General Education: Social Science Focus: Economics (Majorship) Prepared by: Prof. Serafin A. Arviola Jr.

ECONOMIC ISSUES AND CONCERNS 1. Poverty and Income Distribution

• Poverty line – an amount of income below which a family is considered “poor” • Income Poverty – based from the United Nations, are those individuals living on an income of $1 a day or less than $1/day. • Human Development Index – measure of human development looking at the following indexes:

longevity, income and educational attainment • Human Poverty Index – a measure of poverty looking at the income of individuals 2. Globalization - The movement of money, goods, information and people across nations made possible by the rapid

advances in travel and communication. • Globalization of Goods – through importation and exportation • Globalization of Money – through official development assistance (ODA), investments, grants, donations, etc. • Globalization of Information – via internet, cable TV, education, etc. • Globalization of People – through travel, scholarships, fellowships, job, etc. 3. a.

Development Sustainable Development

Development that meets the needs of the present generation without compromising the ability of the future generations to meet that own needs. United Nations declared 2005-2014 as the Decade on Education for Sustainable Development. It pursues the following global vision: The vision of education for sustainable development is a world where everyone has the opportunity to benefit from quality education and learn the values, behavior and lifestyles required for

a sustainable future and for positive societal transformation. Dimensions of Sustainable Development  Economic Dimension  Social Dimension  Institutional Dimension  Environmental Dimension  Cultural and Spiritual Dimension b. The 8 Millennium Development Goals 1. Eradicate extreme poverty and hunger  Reduce by half the production of people living on less than a dollar a day  Reduce by half the proportion of people who suffer from hunger 2. Achieve universal primary education  Ensure that all boys and girls complete a full course of primary schooling 3. Promote gender equality  Eliminate gender disparity in primary and secondary education preferably by 2005, and at all levels by 2015

4. Reduce child mortality  Reduce by two thirds the mortality rate among children under five 5. Improve women’s reproductive health  Reduce by three quarters the maternal mortality ratio 6. Combat HIV/AIDS, Malaria and other diseases  Halt and begin to reverse the spread of HIV/AIDS  Halt and begin to reverse the incidence of malaria and other major diseases 7. Ensure environmental sustainability  Integrate the principles of sustainable development into country policies and programmes; reverse loss of environmental resources  Reduce by half the proportion of people without sustainable access to safe drinking water  Achieve significant improvement in lives of at least 100 million slum-dwellers by 2020 8. Develop a global partnership for development

 Develop further an open trading and financial system, that is rule-based, predictable and nondiscriminatory. Includes a commitment to good governance, development and poverty reduction – nationally and internationally  Address the least developed countries’ special needs. This includes tariff-and quota-free access for their exports; enhanced debt relief for heavily indebted poor countries; cancellation of official bilateral debt; and more generous official development assistance for countries committed to poverty reduction  Address the special needs of landlocked and small island developing States

• It is an inherent power of the state to impose and collect revenues to defray the necessary expenses of the government. • It is compulsory contribution imposed by a public authority irrespective of the amount of services rendered to the payer in return. • It is compulsory level on private individuals and organization by the government to raise revenue to finance expenditure on public goods and services. Purpose of Taxation

c. Philippine Initiatives on Sustainable Development 1. Philippine Agenda 21 in 1996 2. Philippine Council for Sustainable Development in 1997 3. Major legislations on Sustainable Development Clean Air Act of 1999 Ecological Solid Waste Management Act of 2001

• To collect revenue for the government • To redistribute income • To combat inflation • To correct an adverse balance of payment • To check consumption of goods which are considered undesirable • To protect local infant industries • To influence population trend • To improve unfavorable terms of trade • To reallocate resources to create a sense of identity

TAXATION

Sources and Origin of Taxation

• The Constitution • Statutes or Presidential Degrees • Bureau of Internal Revenue regulations • Judicial Decision • Provincial, Municipal and Barrio Ordinances • Observance of International Agreement • Administrative Ruling and Opinions Classification of Tax System • Progressive Income Tax – the Higher the income the higher the tax rate. • Proportional Tax – The tax rate is constant and unaffected by the level of income. • Regressive Tax – The higher the income the lower the tax rate. Classification of Taxes • As to the subject matter – Personal Property, Capitation and Poll Tax; Property Tax; Excise Tax • As to who bears the burden – Direct Tax and Indirect Tax • As to determination of the amount – Specific and Ad valorem • As to purpose – General and Specific

• As to Scope – National and Local Types of Taxes • Direct Taxes • The burden cannot be shifted to the third party • Direct taxes are based on income and wealth • In most cases, direct taxes are progressive in nature • Direct taxes are compulsory in nature • Examples: income tax, residence tax, real state, immigration tax, estate/gift/inheritance tax.

of money the government must spend to collect taxes. • Equity – individual and groups belonging to the same income bracket must be taxed equally while belonging to different income groups must be taxed differently. • Convenience – to set up measures and procedures that will make it more convenient for taxpayers to pay. • Stability – tax system must not be too often or it will encourage tax payers to withhold tax payment until a more preferred system is put in place AGRARIAN REFORM

• Indirect Taxes • The tax burden can be shifted to the third party • Indirect taxes are based on expenditure and consumption • All indirect taxes are regressive in nature • Indirect taxes are optional in the sense that they can be avoided • Examples: sales tax, import tax, VAT/EVAT Characteristics of a Sound Tax System • Efficiency – must generated revenue greater than the amount

Republic Act No. 6657 – The Comprehensive Agrarian Reform Law of 1988 which was signed into law by Pres. Corazon Aquino Meaning - The redistribution of lands, regardless of crops or fruits produced to farmers and regular farm workers who are landless, irrespective of tenurial arrangement to include the totality of factors and support services designed to lift their economic status of the beneficiaries and all other arrangements alternative to

physical redistribution of lands, such as production, profit sharing, labor administration and the distribution of shares of stocks, which will allow beneficiaries to receive a just share of the fruits of the land they work. Principles of Agrarian Reform The policy of the state to pursue a comprehensive Agrarian Reform Program (CARP) to: • To promote social justice • To move the nation toward sound rural development and industrialization • To establish ownercultivatorship of economic sized farms as basis of Philippine agriculture. Coverage of CARP • All alienable and disposable lands of the public domain devoted to or suitable for agriculture • Stability – tax system must not be too often or it will encourage tax payers to withhold tax payment until a more preferred system is put in place AGRARIAN REFORM

Republic Act No. 6657 – The Comprehensive Agrarian Reform Law of 1988 which was signed into law by Pres. Corazon Aquino Meaning - The redistribution of lands, regardless of crops or fruits produced to farmers and regular farm workers who are landless, irrespective of tenurial arrangement to include the totality of factors and support services designed to lift their economic status of the beneficiaries and all other arrangements alternative to physical redistribution of lands, such as production, profit sharing, labor administration and the distribution of shares of stocks, which will allow beneficiaries to receive a just share of the fruits of the land they work. Principles of Agrarian Reform The policy of the state to pursue a comprehensive Agrarian Reform Program (CARP) to: • To promote social justice • To move the nation toward sound rural development and industrialization • To establish ownercultivatorship of economic sized

farms as basis of Philippine agriculture. Coverage of CARP • All alienable and disposable lands of the public domain devoted to or suitable for agriculture • All lands of the public domain in excess of the specific limits as determined by the Congress • All other lands owned by the governments devoted to or suitable for agriculture • All public lands devoted to or suitable for agriculture regardless of the agricultural products raised or can be raised. Retention Limits • Five hectares for land owners • Three hectares to be awarded to each child of the landowner subject to the following qualification: o At least 15 years old o Actually tilling the soil or directly managing the farm Beneficiaries • Agricultural lessees and share tenants • Regular farm workers • Seasonal farm workers • Other farm workers

• Actual tillers or occupants of public lands • Collectives or cooperatives • Other directly working on the land Salient Features of CARP • CARP covers all agricultural lands and not only devoted to rice and corn • CARP covers not only those privately owned tenanted lands but also that of agricultural land owned by Multinational Corporations and commercial farms. • Lower retention limits of three hectares • Rights of indigenous communities, to their ancestral lands are protected to ensure their economic, social and cultural well being • In determining just compensation, the cost of acquisition of the land, the current value of like properties, its nature, actual use and income, the sworn valuation of the owner, the tax declarations and the assessment made by the government assessors shall be considered. • Lands awarded to beneficiaries shall be paid to the Land Bank of the Philippines in 30 annual amortization at six percent interest per annum.

COOPERATIVES New Cooperative Laws – Cooperative Code of the Philippines (RA 6938), Cooperative Development Authority (RA 6939) and Executive Order 95 and 96 issued by President Fidel Valdes Ramos. Definition - A cooperative is: • A free association of persons voluntarily joined together • With common bond of interest • Legally constituted • Purpose of conducting an economic enterprise • Owned, controlled and administered democratically • Making equitable contributions to the capital required • Accepting a fair share of the risks and benefits • Organized in accordance with generally accepted principles Universal Principles of Cooperativism • Open and Voluntary Membership - No artificial discrimination against individuals

because of their race, creed or political affiliation, freedom of entry and exit of any member of the cooperative • Democratic Control – In order for members to gain entry to the cooperatives, they must purchase shares of the cooperative, obtain the right to govern the organization, voting rights of the owner are on the basis of one person, one vote. • Limited Interest on Capital – Capital in a cooperative is like a loan because the owners of the capital can expect to received a rate of return not exceeding that of the prevailing market interest rates on investing. • Division on Net Surplus – Net surplus should be distributed as follows: Item % allocation General Reserve Fund At least 10% Education/Training Fund At Least 10% Optional Fund At Least 10% Dividend/Patronage Refund Remaining Balance of Savings

 General Reserve Funds – cover losses in operation  Education/Training Funds – for members and Management trainings  Optional Funds – discretion of cooperatives for purposes of acquiring land construction of a building or community development  Dividends/Patronage Refunds – the volume of transaction that members have with the cooperative

• Continuing Membership  Pre-membership education seminar as required for entry to the cooperative  Special trainings for the cooperative leadership and members • Cooperation Among Cooperative – interlending and pooling of funds Typologies of Cooperative 1. According to Level of Cooperatives • Primary – members of which are natural

• Secondary – members of which are primaries • Tertiary – members of which are secondary upward to one or more apex organization 2. According to Services Rendered • Credits – is one, which promotes thrift among its members, and creates funds in order to grant loans for productive and provident purposes. • Consumer – is one wherein the primary purpose is to procure and distribute commodities to members and non-members.

• Producers – is one which undertakes joint production whether agricultural or industrial; • Marketing Cooperative – is one which engages in the supply of production inputs to members and in turn market their products. • Service – is one engages in medical and dental care, hospitalization, transportation, insurance, housing, labor, electricity, communications and other services. • Multipurpose – is one which combines two or more activities o these different types of cooperatives

3. According to Scope of Membership • Institutional – Members are employees of a specific institution or corporation • Associational – Members are those who have their own enterprise and belong to specific sector or organization. • Community-Level – Members are based on a defined geographical area.

PREBOARD EXAMINATION

PROFESSIONAL EDUCATION

The Teaching Profession, Social Dimensions for Education 1. To whom does the word teacher refer? I.

Full time teachers

II.

Part time teachers

III.

Guidance counselors

IV.

Librarians

V.

Division Superintendent

a.

I, II, and III

b.

I and III

c. I, II, III, and IV –I,II,V BEST ANSWER

b. Yes, when the teacher is already teaching

d.

c.

III and IV

2. Teacher Kevin has not practiced his profession for the past five years. Can he go back to teaching immediately? a. Yes, if nobody can take his place b. No, unless she has enrolled in refresher course of 12 units c.

No

d.

Yes

3. Is membership to the accredited professional organization for teachers mandatory for all LET passers? a.

No

Yes

d. Only for LET passers who are not repeaters 4. Which is true of the periodic merit exam for teacher provided for in RA 7836? I.

Consist of oral exam

II.

Consist of written exam

III. May serve as additional basis for merit promotion in addition to performance rating IV. Taken with fee of P 1000 per examinee a. I only b. I and IV c. II and III – I,II,III BEST ANSWER

d. II only 5. Can Manny Pacquiao be given a special permit to teach boxing in a special school? a. No, he is not a teacher education graduate b. LET

No, he has not passed the

c.

Yes, he is a graduate of ALS

d. Yes, he has excelled and gained international recognition 6. Is it professional for a teacher to receive gifts from the student and parents? a.

Not at all

b. No, especially if done in exchange for requested concessions c.

Yes, if deserved

d. Yes, in-season and out-ofseason gifts 7. An Education graduate without a license is accepted to teach in a private school? Is this in violation of RA 7836? a. No provided he has taught for at least 3 years

b. Yes. No one may teach without a license c.

No

d.

Yes

8. For relevance to business and industry, what did the First Biennial National Education on Education (2008) impose for updating the Licensure Examination for teachers? a.

Moral or ethical values

b. Technical and scientific competencies c.

Upgraded laboratory facilities

d.

Vocational skills

9. What does the Teacher Education Development Program signify as a prerequisite for employment of teachers in basic education schools? a. National Standard Competencies among teachers b. Licensure Examination for Teachers c.

Induction of new teachers

d. Job interviews for teacher applicants

10. Among active participation of school officials and teachers in the community, which of the following is not appropriate due to prevailing religious sentiments? a. Literacy assistance for out of school children/youths b. Household campaign for healthful practice c. Promoting contraceptives for planned parenthood d. Introducing cooperative thrift practices 11. Which of the following is not John Dewey’s contribution to the sociological foundation of education? a. Facilitating learning along social conditions of the learner b. As a social process, education begins at birth c. True education is transmission of knowledge d. The school is a continuation of home 12. Of the following, which is most fundamental to building up a strong school culture of excellence? a. High standards of performance

b.

Student-centered curriculum

c.

Mission and core values

d.

Student handbook of conduct

13. Among rights of the schools, which is not provided by the law? a. Right for basic education to determine subjects of the study b. Right to enforce administrative systems c. Right to provide proper governance d. Right for institutions of higher learning to determine academic grounds for admission 14. What kind of grassroots model best advances Education for All as served children of slum city dwellers? a.

Mobile education on Kariton

b. Leaf flyers for out-of-school children c.

Radio education modules

d.

Educational television

15. After the implementation of NCBTS, results of LET still reveal low performance among examinees. What can teacher education institutions do to

upgrade their graduates’ LET performance? a. Review curriculum vis-à-vis TOS b.

Intensify Field Study Courses

c. Hire expensive review trainers d. Implement selective admission in TEIs 16. What is the cultural trait of conflicting values that aims to please people in different venues and situations rather than abide by principles? a.

Crab mentality

b.

Split personality

c.

Kanya-kanya system

d.

Bahala na mentality

17. Among qualities which employers look for in the 21st century workplace, which is the most challenging and demanding? a.

Aptitude for teamwork

b.

Skills and social behavior

c.

Readiness to take risks

d. Specific competencies for work

18. In educating the whole person as demanded by the “Learning to be” pillar of the 21st century education, where does the concept of meaning, purpose and engagement belong? a.

Mind and body

b.

Aesthetic sense

c.

Spiritual values

d.

Personal responsibility

19. Which program directly embodies both the pre-service and in-service programs? a. BESRA – Basic Education Sector Reform Agenda b. TEDPA – Technical Education Development Program c.

K-12

d. BEC – Basic Education Curriculum 20. How can the efforts of four agencies (DepEd, CHED, PRC, CSC) be best achieved for the training and development of teachers? a.

Synchronization

b.

cost-reduction

c.

streamlining

d.

sharing of resources

21. What is the core of the Teacher Education Development Program?

b. setting up centered for excellence in teacher education centers

d. outreach by educating the villagers on protection of coral reefs

a. high order thinking skills or HOTS

c.

26. In a tertiary school, the President organized a Fun Run for students, faculty and personnel to enjoy camaraderie, physical exertion under the sun, sense of engagement and achievement. What does the activity promote?

b.

student-centered learning

c. National Competency-Based Teaching Standards d. Technology integration in instruction 22. What is known as a selfappraisal for professional growth that is acceptable and useful for recognizing weakness and strengths for a new beginning teacher? a.

master teacher’s evaluation

b.

student’s evaluation

c.

principal’s evaluation

d.

self-evaluation

23. Among reforms for enhancing teacher professionalism, which has been implemented by law in order to determine whether prospective teachers have acquired professional competencies prior to granting them a permit to teach? a. accrediting a national organization for teachers

licensure examination

d. creation of a professional board for teachers 24. From global competence as defined by international educators, which is the most appropriate characteristic of globally competent individual? a.

familiarity with new culture

b. open-mindedness to new culture c. adaptability to new work environment d.

foreign-language policy

25. For a school, which of the following is most significant in repairing shorelines with depleted coral reefs? a. outreach by depositing rubber tires as artificial coral reefs b. implement reporting system against dynamite fishermen c. legislative lobby to disallow tourism in endangered shorelines

a.

spiritual vigor

b.

cultural consciousness

c.

national integrity

d.

moral integrity

27. In the Education Act of 1901 which established a free public education in the Philippines, what language was imposed under the one-language policy? a.

Spanish

b.

English

c.

Tagalog

d.

Filipino

28. Of the following, which is the most functional intervention in order to achieve a basic right of every Filipino Child under the Constitution and Magna Carta for Disabled Persons?

a.

Philosophy of education

b.

policy for curricular reform

c.

home study program

d.

structural organization

29. Of the following interventions, which is directly aimed at responding to the transitional gap between academic achievement and employment? a. identification of centers of excellence b.

deregulation of tuition fees

c. school networking with business and industry d. voluntary accreditation of schools 30. In the formal education system during Hispanic times in the Philippines, what was not implement but which we enjoyed during the American period? a.

vocational education

b.

private education

c.

religious education

d.

public education

31. If Dr. Jose Rizal lives in the 21st century, what character

expression and commitment would have shown our generation? a.

inventor of techniques

b.

citizen and producer

c. member of family and community d.

creative dreamer

32. In the learning to do pillar of new education, what is the enabling factor that can make the learner fully contribute to a peaceful and just society? a.

knowledge

b.

skills

c.

insights

d.

values

33. Before being able to fully learn to live and work together under the pillar of the 21st century education, what must the learner attain for himself? a.

find peace within oneself

b.

attain an altruistic mind

c.

love his fellowmen

d.

become self-actualized

34. The Transparency International’s perception that the

Philippines suffers a cultural malaise of corruption, what component of our character needs to be further developed along the Learning To Be Pillar of education in the 21st century? a.

Familial-social component

b. Physical-economic component c. Intellectual-emotional component d.

Ethical-spiritual component

35. This powerful European country supplied arms to Afghanistan rebels who were fighting a terrorist war in the Middle East. What was the principle of moral discernment applicable in this case? a.

Principle of double effect

b.

Principle of lesser evil

c. Principle of material cooperation d. Principle of moral cooperation 36. Which of the following best defines a morally mature person? a.

Cultural values clarification

b. Unhampered exercise of one’s right

sensitivity to the welfare of other people?

c. Transmittal of one’s moral viewpoint

a.

Pharisaical

b.

Strict

c.

Lax

d.

Callous

d. Knowledge and practice of universal moral values 37. Educated in a religious school, Sansa goes to confession every day to be free of any kind of sin. How do you characterize Dona’s moral attitude? a.

Callous

b.

Pharisaical

c.

Scrupulous

d.

Strict

38. How would you characterize the moral attitude of Hispanic friars who taught religion but were unfaithful to their vow of property by amassing the land properties of natives? a.

Scrupulous

b.

Strict

c.

Lax

d.

Pharisaical

39. How would you characterize the moral attitude of prisoners with criminal minds, who have no

40. What was the degree of moral certitude when U.S. statement decided to drop the atomic bombing on Hiroshima and Nagasaki to prevent mass deaths by a land invasion of Japan? a.

Doubtful

b.

Certain

c.

Perplexed

d.

Probable

41. Teacher Slash is of the thinking that from the very start students must be made to realize study is indeed hard work. To which philosophy does Teacher Susan adhere? a.

Essentialism

b.

Perennialism

c.

Progressivism

d.

Reconstructionism

42. If your students appear to be more interested in a topic outside your planned lesson for the day, you set aside your lesson plan for that day and grasp the opportunity to discuss the topic of particular interest to your students. Strike the iron while it is hot! Which philosophy governs for your action? a.

rationalism

b.

empiricism

c.

existentialism

d.

progressivism

43. Students must be taught selfresponsibility is the desire of the ___________ teacher. a.

Existentialist

b.

Utilitarianist

c.

Pragmatic

d.

Constructivist

44. Who asserts that teaching is not just depending knowledge into the empty minds of the learners? It is helping students create knowledge and meaning of their experiences? a.

Constructivist

b.

Essentialist

c.

Existentialist

a.

1 and 3

d.

Pragmatist

b.

2 and 4

SITUATIONAL

c.

1 and 2

In a faculty meeting, the principal told his teachers: We need to improve our school performance in the National Achievement Test. What should we do? The teachers gave varied answers as follows:

d.

3 and 4

a.

4

1. Let’s give incentives and rewards to students who get a rating of 85%

b.

2

c.

1

d.

3

2. Let’s teach them to accept complete responsibility for their performance 3. Let’s make the school environment conducive for learning 4. Let’s make use of the experiential methods of teaching 45. On which educational philosophy is response #1 anchored? a.

Behaviorism

b.

Progressivism

c.

Existentialism

d.

Essentialism

46. Which response/s come/s from a behaviorist?

47. If you lean toward a progressivist philosophy, with which response do you agree?

How a teacher relates to his/her pupils depends on his/her concepts about him/her. In a faculty recollection, the teachers were asked to share their thoughts of the learner, their primary customer. What follows are the gists of what were shared:

Teacher A – The learner is a product of his environment. Sometime he has no choice. He is determined by his environment. Teacher B – The learner can choose what he can become despite his environment.

Teacher C – The learner is a social being who learns well though an active interplay with others Teacher D – The learner is a rational being. Schools should develop his rational and moral powers

48. Whose philosophical concept is that of Teacher A? a.

Behaviorist’s

b.

Existentialist’s

c.

Progressivist’s

d.

Rationalist’s

49. If you agree with Teacher C, you are more of a/an a.

Progressivist

b.

Perrenialist

c.

Essentialist

d.

Rationalist

50. Whose response denies man’s freewill? a.

Teacher A’s

b.

Teacher C’s

c.

Teacher B’s

d.

Teacher D’s

Human Growth and Development, Facilitating Learning, Developmental Reading

c. To process and select information

obsession of the young with technology gadgets?

d.

a.

Family social life

b.

Family economic life

c.

Discipline and obedience

d.

Parent-child relationship

To read with understanding

51. From a broad vantage view of human development, who has the primary duty to educate the youths or children?

54. Of the following effects on learning, what is the effect of simulations that make students feel and sense experience in the classroom?

I.

Parents

a.

II.

Teachers

III.

the state

b. Providing experiences that otherwise might not be had

IV.

the schools

52. Of the three aspects of learning, which is not mentioned as needed so that the individual learner in the 21st century can learn how to learn? a.

Ability to think

b.

Mathematical skills

c.

Memory skills

d.

Concentration

53. Which of the following belongs to the more sophisticated learning-to-learn skills for the individual learner? a.

To ask and gather data

b.

To listen and observe

c.

Reinforcing learning

Motivating students

d. Changing attitudes and feelings 55. Of the following effects on learning, what is the effect of assigning various sections of the newspaper, and allowing choice depending on the learner’s choice? a.

Encouraging participation

b.

Reinforcing learning

c.

Allowing different interests

d. Changing attitudes and feelings 56. A young mother observes her seven year old girl glued to her computer games. What aspect of the family life may suffer due to

57. Which of the following is not an advanced process of metacognition among learners? a. Learning how to recognize thoughts b.

Acquisition of new knowledge

c.

Assessing own thinking

d.

Learning how to study

58. Of comprehension or thinking strategies, which is relating one or two items, such as nouns and verbs? a.

Basic elaboration strategies

b.

Complex rehearsal strategies

c. Complex elaboration strategies d.

Affective strategies

59. Of skills teacher should understand and students need to acquire, which is the ability to integrate complex information into

categories through its attributes (characteristics, principles or functions)? a.

Scanning

b.

Complex cognitive

c.

Sharpening-leveling

d.

Complexity-simplicity

60. Inculcating moral maturity among students, which of the following relates to belief and ideals? a.

Promoting human equality

d.

Listening comprehension

62. When preacher Xian read the Genesis story on creation, he explained that God is so powerful he created the universe in only seven days. What level of reading comprehension did preacher John apply? a. Evaluative reading on character, plot or style b.

Literal reading the lines

c. Applied reading beyond the lines

b. Refraining from prejudiced action

d. Interpretative reading between the lines

c. Avoiding deception and dishonesty

63. What is the main organization and orientation of science and social studies reading materials?

d. Respecting freedom of conscience 61. Research studies showed that children in slums generally have lower reading achievement then children in urban schools. What factor is shown to affect reading achievement? a.

Mobility

b. Personality and emotional factors c.

Socio-economic status

a.

Knowledge or recall

b.

Perpetual abilities

c.

Application

d.

Responding

65. In Erikson’s stage theory of development questionnaire, which affirmation does not belong to the stage of initiative vs. guilt? a.

People can be trusted

b.

In difficulty, I will not give up

c. I feel what happens to me is the result of what I have done d.

I am prepared to take a risk

a.

Expository

66. For cognitive learning, what are sets of facts, concepts, and principles that describe underlying mechanism that regulate human learning, development and behavior?

b.

Descriptive

a.

Facts

c.

Narrative

b.

Concepts

d.

Argumentative

c.

Theories

d.

Hypothesis

64. In his History class, teacher Naomi used a current events IQ contest to determine champions in identifying people, places, and events. What learning objective outcome does she aim to achieve?

67. Literature teacher Kim introduced figures of speech in poetry to improve ability of her students to interpret verses. What

kind of thinking is she developing in her students? a.

Critical thinking

b.

Metaphoric thinking

c.

Convergent thinking

d.

Divergent thinking

68. Of clusters of meaningful learning activities, which does not belong to spatial learning activities? a.

Visualization

b.

Concept-mapping

c.

Peer tutoring

d.

Art projects

69. From cluster of meaningful learning activities, which does not belong to verbal-linguistic intelligence learning? a.

Ecological field trip

b.

Debates

c.

Journal writing

d.

Reading

70. Which of the following violates the principle that “each child’s brain is unique and vastly different from one another”?

a. Giving ample opportunity for a pupil to explore rather than simply dish out information b. Employing principles in multiple intelligence in teaching c. Making a left-handed pupil write with her right hand as this is better d. Allowing open dialogue among students of various cultural backgrounds 71. Of the following which is normally expected of Grade VI pupils?

b. Speech and communication disorders c.

Emotional/conduct disorders

d.

Autism

73. Of the following, which is most true of adolescents? a.

Hormonal changes

b.

Last splurge of dependence

c.

Unruly behavior

d.

Defiance of peer group

b.

Being independent of parents

74. Research says, “people tend to attribute successes to internal causes and their failures to external causes.” What does this imply as a most potent key to success?

c.

Showing class leadership

a.

Reasoning

d. Displaying a male or feminine social role

b.

Imagination

c.

Application

d.

Motivation

a. Getting along with classmates

72. From categories of exceptionalities in the young child and adolescents what involves difficulties in specific cognitive processes like perception, language, memory due to mental retardation, emotional/behavioral disorder, or sensory impairment? a.

Learning disabilities

75. From Kohlberg’s theory of moral development, what is the moral reasoning or perspective of Mother Teresa who pledged her life to serve the sick and very old? a.

Social contract

b.

Universal principles

c.

Obedience

d.

Law and order

76. Blind cyclist and teacher Maria Bunyan won 8th place in the able-bodied Sydney 2000 Olympics. Of the following, which is the central and fundamental quality she displayed by never thinking that blindness is an impediment to becoming a great athlete?

and non-discursive communication belong? a.

Psychomotor

b.

Affective

c.

Cognitive

d.

Reflective

79. In what development stage is the pre-school child? a.

Early childhood

a.

Perseverance

b.

Babyhood

b.

Passion

c.

Infancy

c.

Dedication

d.

Late childhood

d.

Self-belief

80. What is mainly addressed by early intervention program for children with disabilities, ages 0 to 3 years old?

77. How can new information be made more meaningful to students? a. Relating it to knowledge they already know b.

Valuing new knowledge

c. Demonstrating novelty of new knowledge d. Increasing retention of new knowledge 78. Under the domains of learning, to what domain do Reflex movements, perceptual abilities,

a. Ensuring inclusion for special children b. lag

Early growth development

c. Identifying strengths and weaknesses in special children d. Preventing labeling of disabled children 81. What is the degree of moral certitude of Jade Althea who entered into marriage only out of

obedience to her parents, but uncertain whether she wanted marriage at all? a.

Certain

b.

Lax

c.

Probable

d.

Doubtful

82. On categories of exceptionality in the young, what is difficulty in focusing and maintaining attention, and/or recurrent hyperactive and impulsive behavior? a.

ADHD

b.

Emotional/conduct disorders

c.

Autism

d. Speech and communication disorders 83. What kinds of skills are commonly dominant in subjects like Computer, PE, Music, and the like? a.

Problem-solving skills

b.

Manipulative skills

c.

Affective skills

d.

Thinking skills

84. How is the disorderly behavior of children classified when they tell lies? a.

Moral

b.

Intellectual

c.

Social

d.

Psychological

85. Which of the following is not among the major targets of the child-friendly school system (CFSS)? a. All school children are friendly b. All children complete their elementary education within six years c. All children 6-12 years old are enrolled in elementary schools d. All grade six students pass the division, regional, and national tests 86. Research studies that reading power affects college students who have insomnia, conflicts with parents, poor rapport with other people. What factor(s) is shown to effect reading achievement? a.

Home conditions

b.

Socio-economic status

c. Personality and emotional factors d. Perception and comprehension 87. Among the following, which is the abstract form of learning, parents teach their children? a. Tumulong ka sa paglinis ng bahay b.

Magbasa ka ng libro

c.

Palagi kang magdasal

d.

Mapakabuti ka

88. What characteristic differentiate spiritual intelligence or spiritual quotient as developed by Harvard University, from sectarian religion (E.g. Christian, Buddhist, Jewish, etc.)? a.

Authoritarian values

b.

Universal values

c.

Creedal values

d.

Sectarian values

89. Among models of reading strategies, what did student Jk adopt when she reads back and forth, attending to both what is in her mind and what’s on the page? a.

Bottoms-up

b.

Interactive

c.

Down-top

d.

Top-down

90. Of the following, how can self-esteem be best developed among learners? a. Doing fair share in community work b.

Fulfilling commitments

c. Through relationships with others d.

Displaying self-control

91. Of Piaget’s Cognitive Concepts, which refer to the process of fitting a new experience to a previously created cognitive structure or schema? a.

Assimilation

b.

Schema

c.

Accommodation

d.

Equilibrium

92. In Piaget’s stages of cognitive development, which is the tendency of the child to only see his point of view and to assume that everyone has the same point of view? a.

Reversibility

b.

Egocentrism

c.

Linguists

d.

c.

Symbolic function

d.

Sociologists

d.

Centration

96. Sequence the following events on the historical development of reading:

98. How do you describe transfer of learning across subject matter, e.g value of thrift in Economic and Social Science?

93. Which is the most basic in Maslow’s hierarchy of needs? a.

Socialization

b.

Actualization

c.

Self-esteem

d.

Altruism

94. Which aspect of multiintelligence is enhanced by asking students to work on a physical model of the atom after a teacher’s discussion on the subject of the atom?

I. Greek letters and the Roman alphabet were developed II. Through the Semite’s ingenuity, sounds, and symbols gave rise to the Phoenician alphabet III. People used pictures and characters to convey messages IV. Researchers showed the processes of reading, comprehension, and interpretation

Interpersonal

a.

I, II, III, and IV

b.

Linguistic

b.

I, II, IV and III

c.

Kinesthetical

c.

III, II, I and IV

d.

Mathematical

d.

IV, II, I and III

a.

95. Among specialist in reading, who are mainly concerned about reading as a thinking process that involves the recognition of printed or written symbols which serve as thought stimuli? a. b.

Semantics Psychologists

97. How is the disorderly behavior of children classified when they don’t focus and lack attention? a.

Intellectual

b.

Social

c.

Moral

Psychomotor

a.

Horizontal

b.

Spiral

c.

Vertical

d.

Cyclic

99. What broad learning is needed for a learner to desire to learn throughout life? a.

Four basic Rs

b.

Basic education

c.

General education

d.

Pre-school system

100. What observation attests to the fact that the sudden student’s motivation vary according to sociocultural background? a. Females mature earlier than boys b. Children from low-income household meet more obstacle in learning c. Genetic endowments may show gifted endowments among the young

d. Brains of boys are bigger and better than those of females

a. Helping hands after a natural crisis, e.g. devastating storm

conditions, e.g. time. What is she achieving for her tests?

Assessment of Learning, Field Study, Practice Teaching

b.

Attending regular meetings

a.

Efficiency

c.

Fund raising for PT funds

b.

Usability

d.

Running the school canteen

c.

Reliability

d.

Validity

101. Of the types of validity tests, what is concerned with the relation of test scores to performance at some future time, e.g. Freshmen college test can show success in college? a.

Curriculum validity

b.

Criterion validity

c.

Content validity

d.

Predictive validity

102. The test questions in Teacher Dae Dae’s test were confusing and subject to wrong understanding, especially to poorer students. What was wrong with the test? a. Inappropriate level of difficult of items

104. Among standardized tests, which reveals strengths and weaknesses for purposes of placement and formulating an appropriate instructional program? a.

Personality tests

107. Of the following subjects, which does not belong to performance-based subjects in which direct instruction is effectively used?

b.

Achievement tests

a.

Values education

c.

Diagnostic tests

b.

Music

d.

Competency tests

c.

Science

d.

Mathematics

105. Among standardized tests, which can show how students perform in comparison with each other and to students in other schools? a.

Competency tests

b.

Unclear directions

b.

Subject exit tests

c.

Ambiguity

c.

Achievement tests

d.

Diagnostic tests

d. Test items inappropriate for outcomes being measured 103. Of the following, which exemplifies the best example of cooperation and voluntarism in the Parent-Teacher Associations?

106. Teacher Bea Bunana makes her tests easy for students to understand, easy to administer and score and suitable to test

108. Which of these approaches would reform assessment outcomes? a. Apply sanctions on low performing schools b. Focus on testing without investing the learner’s needs c. Use understanding as means of giving feedback on students learning d. Compare results of performance of all schools

109. Using extrinsic motivational assessment, what could be the most noble motive in students pursuing a lifetime work and mission for the teaching profession? a. Promise of high rank and prestige b. Social service to upcoming generations c. Economic security and welfare d.

Respected position in society

110. To what process of evaluation does determining the extent objectives are met belong?

112. What primary response factor is considered by Essay questions?

d. Retaining it in memory for a long period of time

a.

Factual information

b.

Wide sampling of ideas

115. What does it mean if student Pete got a 60% percentile rank in class?

c.

Originality

d. Less time for construction and scoring 113. Among written categories of assessment methods, what did teacher Maggie Lagid use when she assessed the stock knowledge of her students through questioning in an open class? a.

Oral questioning

b.

Performance test

a.

Authentic

c.

Product rating scale

b.

Formative

d.

Observation and self-report

c.

Criterion-referenced

d.

Norm-referenced

114. In the context of the 6 facets of understanding cited by Wiggins and McTIghe, what is a proof of a student’s understanding a principle?

111. Which form of the foundation of all cognitive objects without which the next level of higher thinking skills cannot be attained? a.

Knowledge

b.

Synthesis

c.

Application

d.

Analysis

a.

Stating given examples

b. Repeating it as given by the teacher c. Applying it to solve his problem

a. He scored better than 60% of the class b. He scored less than 60% of the class c. He got 40% of the test wrongly d. He got 60% of the items correctly 116. Which of the following may not be adequately assessed by a paper and pencil test? a.

Sight reading in music

b.

Multiplication skills

c.

Subject-verb agreement

d.

Vocabulary meaning

117. What should be done with test item whose difficulty index is . 98? a.

Revise it

b.

Retain it

c.

Reject it

d. Reserve it for another group of students

were the students were weakest. What type of test is this?

118. What is known as the scoring guides for rating open-ended questions?

a.

Aptitude test

b.

Remedial test

c.

Diagnostic test

d.

Readiness test

a. b.

Rubrics Outcomes

c.

Scales

d.

Outputs

119. What does it mean to say that the facility index of a test item is .50? a.

It is reliable

b.

It is valid

c.

It is moderate in difficulty

d.

It is very easy

120. With the mode of answering as a point of reference, which of the following does not belong to this test group? a.

Completion

b.

Essay

c.

Problem-solving

d.

Matching

121. One half of the class scored very low. Teacher Janus gave another tests to determine where

122. On what is normative marking based?

c.

-10

d.

-25

125. What is the graphic illustration for the relationship between two variables? a.

Histogram

b.

Normal curves

c.

Frequency polygons Scatter diagram

a.

High marks of few students

d.

b.

Failure of some students

126. What does a negative discrimination index mean?

c. Normal curve of standard distribution d. Student achievement relative to other students 123. What cognitive domain is involved in the student’s clarifying information from conclusion? a.

Synthesis

b.

Evaluation

c.

Analysis

d.

Application

124. Which of the following indicates a strong negative correlation? a.

-75

b.

-15

a. The test item has low reliability b. More from the lower group answered the test item correctly c. More from the upper answered the test correctly d. The test could not discriminate between the upper and lower group 127. What is the deviation from a standard or desired level of performance? a.

A problem

b.

A deficit

c.

A defect

d.

A gap

128. How does a student’s 80 percentile score interpreted? a. High in all the skills being tested b. Higher than 80% of the members of the group c. Better relative to the competencies targeted d.

80% of the specified content

129. Of the types of validity for tests, what is focused on the extent to which a particular tests correlates with acceptable measure of performance? a.

Curricular validity

b.

Content validity

c.

Criterion validity

d.

Predictive validity

130. Among general categories of assessment methods, what instruments did pre-school teacher Justine use when he rated the handwriting of his students using a prototype handwriting model? a.

Product rating scale

b.

Performance test

c.

Written response instruments

d.

Observation and self-reports

131. On what should teacher’s evaluation of a learner’s work be based? i.

Attendance

ii.

Merit

iii. Quality of academic performance iv.

Behavior in class

a.

I and II

b.

II, III, and IV

c.

II and III

d.

I, II, III, and IV

132. Self-evaluation can be done in various ways, but this is not one of them: a. Use of an evaluation instrument b.

Written reflection

c. Self-videotape of class performance d.

Per feedback session

133. In her test, Teacher Marian R unknowingly gave clues to the answers that reduce usability of the test. What was wrong with the test? a.

Ambiguity

b.

Unclear directions

c.

Poorly constructed test items

d.

Test too short

134. In preparing classroom tests, which of the following checklists is the LAST among steps in tests preparation? a. How are the objective items to be scored? b. How are the test results to be reported? c. How I have prepared a table of specifications? d. How are the test scores to be tabulated? 135. What formula is used to total and compute test scores at the end of the year? a. [Test scores = transmutation table] x 100 b. [Highest score + Lowest possible score] x 100 c.

[Student’s score x 100]

d. [Student’s score + Highest possible score] x 100 136. What can be said of student performance in a positively skewed score distribution?

a. A few students performed excellently

progress accomplishment by a targeted clientele?

b.

a.

Evaluation instrument

b. None. It is best to look at individual scores

b.

Rubric

c.

Mean

c.

Achievement test

d.

Median

d.

Portfolio

143. Self-evaluation has become an important kind of performance assessment among teachers, useful as an honest self-criticism and a starting point to removal evaluation by supervisors, peers, or students. How is self-evaluation described?

Most students performed well

c. Almost all students had average performance d. Most students performed poorly 137. Which is true when the standard deviation is small? a. Scores are toward both extremes b.

Scores are spread apart

c. Scores are tightly bunched together d.

The bell curve is relatively fat

138. In her tests, Teacher Tomden made tests that were either too difficult or too easy. What was wrong with her tests? a.

Unclear directions

b. Inappropriate level of difficulty of the test items c.

Ambiguity

d. Identifiable patterns of answers 139. What is an alternative assessment tool that consists of a collection of work artifacts or in

140. What computation did teacher Panny use in getting the difference between the highest and lowest scores in each class? a.

Mean

b.

Range

c.

Standard deviation

d.

Median

141. Which measure of central tendency is most reliable when scores are extremely high and low? a. Cannot be identified unless individual scores are given b.

Median

c.

Mode

d.

Mean

142. Which measure of central tendency is most reliable to get a picture of the class performance whose raw scores in a quiz are: 97, 95, 85, 86, 77, 75, 50, 10, 5, 2, 1?

a.

Mode

a. Evidence of teaching performance b. Substitute to supervisor’s rating c.

Guide for self-adjustment

d.

Tool for salary adjustment

144. What is the common instrument used in measuring learning in the affective domain? a.

Multiple choice

b.

Checklist

c.

Scaling

d.

Questionnaire

145. On the test giver’s list of Do’s, which of the following is not

relative to motivating students to do their best? a.

Read test directions

b. Reduce test anxiety, e.g. “Take a deep breath.” c. test

Explain the purpose of the

d. Tell students: “I will be proud of you if you perform well.” 146. What is the range if the score distribution is: 98, 93, 93, 93, 90, 88, 87, 85, 85 , 85, 70, 51, 34, 34, 34, 20, 18, 51, 12, 9, 8, 6, 3, 1? a.

93

b.

85

c.

97

d.

Between 51 and 34

147. What does the test mean if the difficulty index is 1?

a. The increase in the amount of practices does not at all affect the number of errors

Principles and Methods of Teaching, Educational Technology, Curriculum Development

b. As the amount of practice increases, the number of errors decreases

151. Facilities such as classrooms, fixtures, and equipment can often damage the morale of new teachers and become an obstacle for adapting well to the school environment. What should be the policy for assigning said physical facilities?

c. The decrease in the amount of practice sometimes affects the number of errors d. Decrease in the amount of practice goes with decrease in the number of errors 149. An entering college would like to determine which course is best suited for him. Which test is appropriate for this purpose? a.

Aptitude test

b.

Intelligence test

c.

Achievement test

d.

Diagnostic test

a.

Very difficult

b.

Missed by everyone

c.

Very easy

150. Which of the following criteria is the basis for selecting tests that yield similar results when repeated over a period of time?

d.

A quality item

a.

Efficiency

b.

Validity

c.

Usability

d.

Reliability

148. What is the meaning of a negative correlation between amount of practice and number of errors in tennis?

a.

needs of student’s basis

b.

position ranking basis

c.

first-come, first-served basis

d.

service seniority basis

152. There are various functions a fellow teacher or peer coach can help new teachers. What role does a peer coach play by being present/available to share ideas, problems and success with a new teacher? a. a provider of technical feedback b.

a facilitator of strategies

c.

an analyzer of teaching job

d.

a close peer or companion

153. Teacher Princess sees to it that her classroom is clean and orderly so her pupils will less likely

disarrange seats and litter on the floor. On which thought is her action based? a.

existentialism

b.

progressivism

c.

behaviorism

d.

reconstructionism

154. Teacher Nancy is directed to pass an undeserving student with a death threat. Which advise will a utilitarian give? a. Don’t pass him. You surely will not like someone to give you a death threat in order to pass b. Pass the student. That will be off use to the student, his parents and you. c. Pass the student. Why suffer the threat? d. Don’t pass him. Live by your principle of justice. You will get reward, if not in this life, in the next! 155. In what setting is differentiated and multi-lingual teaching most effective?

c. children with diverse cultural backgrounds

that the teacher should give the student the benefit of the doubt?

d.

a. Make sure facts are right before punishing

pre-school children

156. After the embarrassing incident, Teacher Kevin vowed to himself to flunk the student at the end of the school term. What has Dante done that is against the guidelines for using punishment? a. Punishing immediately in an emotional state b. Using double standards in punishing c.

Doing the impossible

d. Holding a grudge and not starting with a clean slate

b. Doubt the incident really happened c. Don’t punish and doubt effectiveness of punishment d. Get the side of the students when punishing 159. Which of the following guidelines for punishment may be done? a. Don’t punish students outside of school rules on punishment

157. Following the principles for punishing students, which of the following is the LEAST desirable strategy for classroom management?

b.

a. Punishing while clarifying why punishment is done

160. For group guidance in classroom management, what element is lacking when there is too much competitiveness and exclusiveness with the teacher being punitive and partial to some students?

b.

Punishing while angry

c. Punishing the erring student rather than the entire class

a.

special children with classes

d.

Give punishment sparingly

b.

multi-grade classes

158. According to the guidelines on punishment, what does it mean

Don’t threaten the impossible

c. Don’t use double standards for punishing d.

Don’t assign extra homework

a. Dissatisfaction with classroom work

b.

Poor interpersonal relations

c.

Poor group organization

d.

Disturbance in group climate

161. To demonstrate here authority Teacher Kokeyni made an appeal to undisciplined students. What kind of appeal did she make by saying, “Ladies and gentlemen, don’t engage in that kind of behavior, you can do much better?” a.

Invoke peer reaction

b.

Exert authority

c. Internalizing student’s image of themselves d.

Teacher-student relationship

162. What is the term for the leap from theory to practice in which the teacher applies theories to effective teaching methods and theories? a.

Integration process

b.

Informational process

c.

Conceptualization process

d.

Construction process

163. Of subcategories of movement behavior, what is

happening when the teacher ends an activity abruptly? a.

Thrust

b.

Truncation

c.

Stimulus-bounded

d.

Flip-flop

164. Of subcategories of teacher movement behavior, what is happening when the teacher goes from topic or activity to other topic or activities, lacking clear direction and sequence of activities? a.

Truncation

b.

Dangle

c.

Thrust

d.

Flip-flop

165. Of subcategories of teacher movement behavior, what is happening when the teacher is too immersed in a small group of students or activity, thus ignoring other students or activity?

166. From classroom management strategies applied on erring students, which of the following should not be done? a.

Surprise quiz

b. Communicating problems to parents c.

Parent-principal conference

d. Shaming erring student before the class 167. Among mistaken goals in the Acceptance Approach to discipline, what happens when students defy adult by arguing, contradicting, teasing, temper tantrums, and low level hostile behavior? a.

Power seeking

b.

Withdrawal

c.

Revenge seeking

d.

Attention getting

a.

Truncation

b.

Flip-flop

168. Teacher Ann Patuan dealt effectively with a minor infraction of whispering by a student to a neighbor during class. Which of the following did she do?

c.

Stimulus-bounded

a.

d.

Thrust

b. Continue to teach and ignore infraction

Reprimand quietly

c. Reprimand student after class

and stories. What is this quality of lesson content?

d. Use nonverbal signals (gesture or facial expression)

a.

Interest

b.

Feasibility

c.

Self-sufficiency

d.

Balance

169. What mistake is teacher Senemin Basic trying to avoid by never ignoring any student or group of students in her discussions and other activities? a.

Non-direction

b.

Dangled activity

c.

Divided attention

d.

Abrupt end

170. Teacher Dra D Explorer is a great lecturer and so she is invited to speak and represent the school on many occasions. What is one quality of her lecturers when she follows a planned sequence, not diverting so as to lose attention of her listeners?

172. Teaching English, teacher Krizzy is careful about her lesson content. What quality of content did he achieve when she made certain her information came with the “information explosion” which she got in the Internet, such as how to effectively teach phonetics? a.

Learnability

b.

Significance

c.

Balance

d.

Interest

a.

Explicit explanations

b.

Continuity

c.

Inclusion of elements

173. Teacher Kevin made certain his lesson content can be useful to his students, taking care of their needs in a student-centered classroom. What is this kind of quality content?

d.

Fluency

a.

Utility

b.

Balance

c.

Self-sufficiency

d.

Interest

171. Teacher Aldub makes certain content interesting to his students. Focusing on learners, he also uses many simple examples, metaphors

174. In the implementation of the curriculum at the classroom level, effective strategies are called “Green”. Which of the following belongs to the Green Flag? a. Homogenous students grouping b. Content delivery based on lessons c.

Excess in chalkboard talk

d. Student interest and teacher enthusiasm e.

Rigidity if movement

175. In the implementation of the curriculum at the classroom level, ineffective strategies are called “Red”. Which of the following belongs to the Red Flag? a. Content applied to real-life situations b. Overemphasis on drill and practice c. Available enrichment activities d. Integration of problem solving 176. Teacher Maggie explains by spicing her lectures with examples, descriptions and stories. What is this quality in her lectures?

a.

Planned sequence

b.

Elaboration through elements

c.

Use of audiovisuals

179. In avoiding implying sickness or suffering, which of the following is the most preferable way to refer to those with disabilities like polio?

d.

Simple vocabulary

a.

“Is polio-stricken”

177. Can technology take the place of the teacher in the classroom? Select the most appropriate answer:

b.

“Had polio”

c.

“Polio victim”

d.

“Suffers from polio”

a. No. It is only an instrument or a tool

180. If threat of punishment is necessary on erring students, how should this best be done?

b. Yes, when they hire less teachers and acquire more computers

a. Make the threat and reinforce with warning

c. Yes. When teachers are not competent

b. Make the threat with immediate punishment

d. Yes, such as in the case of Computer-assisted instruction (not teacher-assisted instruction)

c. Ward and threat at the same time

178. What kind of tool is technology as evidenced by its use in word processing databases, spreadsheets, graphics design and desktop publishing? a.

Analyzing tool

b.

Encoding tool

c.

Productivity tool

d.

Calculating tool

d. First a warning before the threat 181. Among cognitive objectives, what is also known as an understanding and is a step higher than more knowledge of facts? a.

Comprehension

b.

Analysis

c.

Synthesis

d.

Application

182. What is the quality of teacher Pining Garcia’s lecture when she makes use of various pictures, charts, graphs, videos to support her lectures? a.

Simplified vocabulary

b. Enrichment through visual aids c. Causal and logical relationships d.

Continuing sequence

183. In determining the materials and media to use, what consideration did Teacher Ina A. Mag adopt when he chose materials that can arouse and sustain in curiosity? a.

Satisfaction

b.

Interest

c.

Expectancy

d.

Relevance

184. Which of the following is true of a democratic classroom? a. Teacher acts as firm decision maker b. Students decide what and how to learn c.

Consultation and dialogue

d. Suggestions are sent to higher officials for decisions 185. This is appropriate use of technology which can unite people of the world rather than exploit them? a.

For pornography

b.

For social media

c.

For financial fraud

d.

For propaganda

186. From structures in Multifunctional Cooperative Learning, which involves each student writing in turn one answer as a paper and pencil is passed around the group?

d.

Sensitivity

188. Teacher Lester Cruz Valdez gets more information about how his students learn in order to upgrade his pedagogy. What principle is he following? a. Teachers should keep track of learning outcomes b. Teachers should value information c. Teachers should document information data on students d. Teachers should teach and test learning

a.

Jigsaw

b.

Inside-outside circle

189. In order to assist new teacher, which is the most effective way to clarify the schools’ goals and responsibilities early in the first year?

c.

Roundtable

a.

Student’s handbook

d.

Partners

b.

Orientation

c.

Principals’ memorandum

d.

School curriculum

187. How does the “humaneness” of the teacher best described when he/she is full interest and enthusiasm in the work of teaching? a.

Responsiveness

b.

Perceptiveness

190. Of components of direct instruction, which involves teachers and students working together on a skill or task and figuring out how to apply the strategy?

c.

Knowledge

a.

Consolidation

b.

Guided practice

c.

Application

d.

Modeling

191. In direct/expositive instruction, what is the logical pattern of procedures in a lesson adopted? I. Provide motivation and draw commitments II. Explain rationale and objectives III.

Provide feedback

IV.

Practice for mastery

a.

II, I, IV, and III

b.

IV, I, II and III

c.

I, IV, III, and II

d.

I, II, IV and III

192. Teacher JanJan made certain his lesson content is within the capacity of his young forum grade learners. What is the quality of John’s lesson content when he fits lesson to learner’s capacity to absorb lesson content? a.

Learnability

b.

Balance

c.

Validity

d.

Interest

193. From structures of Multifunctional Cooperative Learning, which makes each group to produce a group product to share with the whole class?

c. The teaching profession is the lowest paid profession

selection and organization of lesson content is she following?

d. She takes no pride in the teaching profession

a.

Significance

b.

Self-sufficiency

c.

Feasibility

d.

Balance

a.

Coop-coop

196. In the guided exploratory approach to learning, which is not the term used for Inquiry learning?

b.

Think-pair-share

a.

Heuristic learning

c.

Team Word-Webbing

b.

Problem-solving learning

d.

Partners

c.

Discovery learning

d.

Expository learning

194. This is the more appropriate understanding of technology in education? a.

Methods and process

b.

Inventions and equipment

c.

Channels and instruments

197. What is another quality of teacher Lassie Pecson’s lectures when she used words that are within the grasp of her listeners, avoiding technical terms and jargons?

d. Hardware, designs, and environment

a. Use of specific descriptions and examples

195. A teacher introduces herself as teacher only. What does this imply?

b.

Enriched audiovisuals

c.

Normal vocabulary

d.

Planned sequence

a. She must have been forced to pursue a career in teaching. b. The teaching profession is not a very significant one

198. In delivering her lessons, teacher Blackie Lou Blanco is careful that no topic is extensively discussed at the expense of other topics. That guiding principle in

199. In determining materials and media to use, what consideration did Teacher Grachie adopt when she gave importance to the level of outcome and the learner’s sense of fulfillment in performing the task? a.

Expectancy

b.

Satisfaction

c.

Interest

d.

Relevance

200. In the inductive approach to learning, what is not among the facilitating skills needed on the part of the teacher? a. Teacher giving generalization of principles b. Commenting to pave way for generalizations or principles c.

Organizing answers

d.

Asking the right questions

FINAL REVIEW

1. A guest speaker in one graduation rites told his audience: "Reminder, you are what you choose to be." The guest speaker is more of a/an . a.

Realist b. Pragmatist

c.

Idealist d. Essentialist

2. A teacher who equates authority with power does NOT . a. Shame b. develop self-respect in every pupil c.

retaliate

d. intimidate

3. Based on Edgar Dale's Cone of Experience, which activity is farthest from the real thing? a. Read images

b. Hear c. View d. Attend exhibit

4. Based on Piaget's theory, what should a teacher provide for children in the concrete operational stage? a. Activities for hypothesis formulation. b. Learning activities that involve problems of classification and ordering.

English

80

16

109

In which subject(s) did Ronnel perform best in relation to the group's performance? A. Physics and Math b. English c. Physics d. Math

c. Games and other physical activities to develop motor skills.

6. Who among the following puts more emphasis on core requirements, longer school day, longer academic year and more challenging textbooks

d. Stimulating environment with ample objects to play with.

A. Perennialist Essentialist

5. Study this group of tests which was administered with the following results, then answer the question.

C. Progressivist

B. D. Existentialist

7. I drew learners into several content areas and encouraged them to solve a complex question for Inter-disciplinary teaching. Which strategy did I use?

Subject Score

Mean SD

Ronnels

Math

56

10

43

A. Problem-centered learning

Physics

41

9

31

B. Unit method

C. Reading-writing activity D. Thematic instruction 8. The concepts of trust vs. maturity, autonomy vs. self-doubt, and initiative vs. guilt are most closely related with the works of A. Erikson

B. Piaget D. Jung

cause/s of this student's reading problem is/are. A. emotional factors teaching

B. poor

C. neurological factor Immaturity

D.

C. Freud

9. I want to teach concepts, patterns and abstractions. Which method is most appropriate? A. Indirect instruction Discovery

B.

C. Direct instruction solving

D. Problem

12. Principal B tells her teachers that training in the humanities is most important. To which educational philosophy does he adhere? A. Existentialism B. Perennialism

10. Which behavioral term describes a lesson outcome in the highest level of Bloom's cognitive domain? A. Create B. Evaluate C. Analyze D. Design 11. A sixth grade twelve-year old boy comes from a dysfunctional family and has been abused and neglected. He has been to two orphanages and three different elementary schools. The student can decide on the second grade level, but he can comprehend orally material at the fourth or fifth grade level. The most probable

C. Progressivism D. Essentialism 13. Which is the first step in planning an achievement test? A. Define the instructional objective. B. Decide on the length of the test. C. Select the type of test items to use.

C. Possess dignity and reputation D. With high-moral values as well as technical and professional competence 15. Teacher Q does not want Teacher B to be promoted and so writes an anonymous letter against Teacher B accusing her of fabricated lies Teacher Q mails this anonymous letter to the Schools Division Superintendent. What should Teacher Q do if she has to act professionally? A. Submit a signed justifiable criticism against Teacher B, if there is any. B. Go straight to the Schools Division Superintendent and gives criticism verbally. C. Hire a group to distribute poison letters against Teacher B for information dissemination.

D. Build a table of specification.

D. Instigate student activists to read poison letters over the microphone.

14. In the Preamble of the Code of Ethics of Professional Teachers, which is NOT said of teachers?

16. As a teacher, what do you do when you engage yourself in major task analysis?

A. LET passers

A. Test if learning reached higher level thinking skills.

B. Duly licensed professionals

B. Breakdown a complex task into sub-skills.

B. Ask each student to contribute one question.

B. Ask her students to answer questions beginning with What if ...

C. Determine the level of thinking involve

C. Make twenty questions but ask the students to answer only ten of their choice.

C. Tell her pupils to state data presented in graphs.

D. Revise lesson objectives 17. Which is a sound classroom management practice? A. Avoid establishing routines B. Establish routines for all daily needs and tasks. C. Apply rules and policies on a case to case basis. D. Apply reactive approach to discipline. 18. What does extreme authoritarianism in the home reinforce in learners? A. Doing things on their own initiative B. Ability to direct themselves. C. Dependence on others for direction. D. Creativity in work. 19. In a criterion-referenced testing, what must you do to ensure that your test is fair? A. Make all of the questions true or false.

D. Use the objectives for the units as guide in your test construction. 20. Teacher M's pupils are quite weak academically and his lesson is already far behind his time table. How should Teacher M proceed with his lesson? A. Experientially B. Inductively C. Logically

D. Deductively

21. Soc exhibits fear response to freely roaming dogs but does not show fear when a dog is on a leash or confined to a pen. Which conditioning process is illustrated A. Generalization B. Extinction C. Acquisition Discrimination

D.

22. Teacher P wants to develop the skill of synthesizing in her pupils. Which one will she do? A. Ask her students to formulate a generalization from the data shown in graphs.

D. Directs her students to ask questions on the parts of the lesson not understood. 23. Studies in the areas of neurosciences disclosed that the human brain has limitless capacity. What does this imply? A. Some pupils are admittedly not capable of learning. B. Every pupil has his own native ability and his learning is limited to this native ability. C. Every child is a potential genius. D. Pupils can possibly reach a point where they have learned everything. 24. What should you do if a parent who is concerned about a grade his child received compared to another student's grade, demands to see both students' grades? A. Refuse to show either record. B. Show both records to him. C. Refuse to show any record without expressing permission from principal.

D. Show only his child's records. 25. Rights and duties are correlative. This means that . A. rights and duties regulate the relationship of men in society B. rights and duties arise from natural law C. each right carries with it one or several corresponding duties D. rights and duties ultimately come from God 26. If you agree with Rizal on how you can contribute to our nation's redemption, which should you work for? A. Opening our doors to foreign influence B. Upgrading the quality of the Filipino through education C. Stabilizing the political situation D. Gaining economic recovery 27. Teacher A is directed to pass an undeserving student with a death threat. Which advice will a hedonist give? A. Pass the student. Why suffer the threat?

B. Don't pass him. You surely will not like someone to give you a death threat in order to pass. C. Don't pass him. Live by your principle of justice. You will get reward, if not in this life, in the next! D. Pass the student. That will be of use to the student, his parents and you. 28. Are percentile ranks the same as percentage correct?

30. To elicit more students’ response, Teacher G made use of covert responses. Which one did she NOT do? A. She had the students write their response privately. B. She showed the correct answers on the overhead after the students have written their responses.

A. It cannot be determined unless scores are given.

C. She had the students write their responses privately then called each of them.

B. It cannot be determined unless the number of examinees is given.

D. She refrained from judging on the student's responses.

C. No

31. Which is a form of direct instruction?

D. Yes

29. Teacher W wants to review and check on the lesson of the previous day? Which one will be most reliable? A. Having students identify difficult homework problems. B. Having students correct each other's work. C. Sampling the understanding of a few students. D. Explicitly reviewing the taskrelevant information necessary for the day's lesson.

A. Discovery process Problem solving

B.

C. Programmed instruction Inductive reasoning

D.

32. Which test has broad sampling of topics as strength? A. Objective test answer test

B. Short

C. Essay test type

D. Problem

33. Teacher B uses the direct instruction strategy. Which sequence of steps will she follow?

C. Yes. The rare invitation will certainly be welcomed by an overworked promotional staff.

I. Independent practice

D. Yes. There's nothing wrong with sharing one's blessings.

II. Feedback and correctiveness

IV. Presenting and structuring

35. Which illustrates a developmental approach in guidance and counseling?

V. Reviewing the previous day's work

A. Spotting on students in need of guidance

A. V-II-IV-III-I V

B. III-II-IV-I-

B. Teaching students how to interact in a positive manner

C. V-lV-III-II-I IV

D. I-V-II-III-

C. Acting as a mediator

III. Guided student practice

34. Teacher H and Teacher I are rivals for promotion. To gain the favor of the promotional staff, Teacher I offers her beach resort for free for members of the promotional staff before the ranking. As one of the contenders for promotion, is this becoming of her to do?

D. Making the decision for the confused student 36. The test item "Group the following items according to shape" is a thought test item on . A. creating C. generalizing comparing

B. classifying D.

A. Yes. This will be professional growth for the promotional staff. B. No. This may exert undue influence ori the members of the promotional staff and so may fail to promote someone on the basis of merit.

37. Direct instruction is for facts, rules, and actions as indirect instruction is for , , ,. A. hypotheses, verified data and conclusions

B. concepts, patterns and abstractions C. concepts, processes and generalizations D. guesses, data and conclusions 38. Which group of philosophers maintains that truth exists in an objective order that is independent of the knower? A. Idealist C. Existentialists

B. Pragmatists D. Realist

39. On whose philosophy was A. S. Neil's Summerhill, one of the most experimental schools, based? A. Rousseau Pestalozzi

B.

C. Montessori Locke

D. John

40. Teacher H strives to draw participation of every student into her classroom discussion. Which student's need is she trying to address? The need A. to show their oral abilities to the rest of the class B. to be creative

C. to feel significant and be part of a group D. to get everything out in the open 41. Out of 3 distracters in a multiple choice test item, namely B, C, and D, no pupil chose D as answer. This implies that D is A. an ineffective distracter vague distracter

B. a

C. an effective distracter D. a plausible distracter 42. Which describes normreferenced grading?

44. Teacher G's lesson objective has something to do with the skill of synthesizing? Which behavioral term is most appropriate? A. Test Appraise

B. Assess D. Theorize

C.

46. To come closer to the truth we need to go back to the things themselves. This is the advice of the

C. The students' past performance

47. Which is NOT a sound purpose for asking questions?

C. Hiya Pakikisama

D.

D. Realism

refer to content question?

B. Socrates C. D. Pythagoras

B. What constitutes a perfect score

B.

C. Experimentalism

A. Plato Aristotle

C. idealists

A. Authoritativeness Authoritarianism

B. Existentalism

49. Research tells that teachers ask mostly content questions. Which of the following terms does NOT

A. The performance of the group

43. Which Filipino trait works against the shift in teacher's role from teacher as a fountain of information to teacher as facilitator?

A. Idealism

45. From whom do we owe the theory of deductive interference as illustrated in syllogisms?

A. behaviorists phenomenologists

D. An absolute standard

you the skill to cope with change. What is his governing philosophy?

B.

D. pragmatists

A. To probe deeper after an answer is given. B. To discipline a bully in class. C. To remind students of a procedure. D. To encourage self-reflection. 48. Your teacher is of the opinion that the world and everything in it are ever changing and so teaches

A. Closed Concept

B. Direct C. D. Convergent

50. Whose teaching is in support of Education for All (EFA), he asserted that in teaching there should be no distinction of social classes. A. Sun Yat Sen Confucius C. Mencius

B. D. Lao Tsu

51. As a teacher, you are a Reconstructionist. Which among these will be your guiding principle? A. I must teach the child every knowledge, skill, and value that he needs for a better future. B. I must teach the child to develop his mental powers to the full.

C. I must teach the child so he is assured of heaven. D. I must teach the child that we can never have real knowledge of anything. 52. If a teacher plans a constructivist lesson, what will he most likely do? Plan how he can A. do evaluate his students' work B. do reciprocal teaching C. lecture to his students D. engage his students in convergent thinking 53. Based on Piaget's theory, what should a teacher provide for children in the sensorimotor stage? A. Games and other physical activities to develop motor skill. B. Learning activities that involve problems of classification and ordering. C. Activities for hypothesis formulation. D. Stimulating environment with ample objects to play with. 54. Teacher H gave her first-grade class a page with a story in which pictures take the place of some words. Which method did she use?

A. The whole language approach B. The Spaulding method

57. In what way can teachers uphold the highest possible standards of quality education?

C. The rebus method D. The language experience approach 55. A stitch on time saves nine, so goes the adage.. Applied to classroom management, this means that we

A. By continually improving themselves personally and professionally B. By wearing expensive clothes to change people's poor perception of teachers

A. may not occupy ourselves with disruptions which are worth ignoring because they are minor

C. By working out undeserved promotions

B. must be reactive in our approach to discipline

D. By putting down other professions to lift the status of teaching

C. have to Resolve minor disruptions before they are out of control D. may apply 9 rules out of 10 consistently 56. The attention to the development of a deep respect and affection for our rich cultural past is an influence of A. Confucius Hegel

B.

C. Teilhard de Chardin Dewey

D.

58. Student B claims: I cannot see perfection but I long for it. So it must be real.Under which group can he be classified? A. Idealist

B. Empiricist

C. Realist

D. Pragmatist

59. In Krathwohl's taxonomy of objectives in the affective, which is most authentic? A. Characterization Organization

B.

C. Responding Valuing

D.

60. NSAT and NEAT results are interpreted against set mastery level. This means that NSAT and NEAT fall under

.

A. intelligence test aptitude test

B.

C. criterion-referenced test norm-referenced test

D.

61. Each teacher is said to be a trustee of the cultural and educational heritage of the nation and is, under obligation to transmit to learners such heritage. Which practice makes him fulfill such obligation? A. Use the latest instructional technology. B. Observe continuing professional education. C. Use interactive teaching strategies. D. Study the life of Filipino heroes. 62. A teacher's summary of a lesson serves the following functions, EXCEPT

C. it makes provisions for full participation of students. D. it clinches the basic ideas or concepts of the lesson. 63. In which competency do my students find the greatest difficulty? In the item with a difficulty index of A. 0.1

B. 0.9

C. 0.5

should help students understand and appreciate themselves as unique individuals who accept complete responsibility for their thoughts, feelings, and actions. From which philosophy is this thought based? A. Perennialism Essentialism

B.

C. Existentialism D. Progressivism

D. 1.0

A. Socratic method Cooperative learning

B.

67. Whose influence is the education program that puts emphasis on self-development. through the classics, music, and rituals?

C. Mastery learning Indirect instruction

D.

A. Buddha Confucius

64. Which method has been proven to be effective in courses that stress acquisition of knowledge?

65. Why should a teacher NOT use direct instruction all the time? A. It requires much time. B. It requires use of many supplementary materials. C. It is generally effective only in the teaching of concepts and abstractions.

A. it links the parts of the lesson

D. It reduces student’s engagement in learning.

B. lt brings together the information that has been discussed

66. Principal C shares this thought with his teachers: Subject matter

B. Mohammed D. Lao Tsu

C.

68. Teacher A is a teacher of English as a Second Language. She uses vocabulary cards, fill-in-theblank sentences, dialogues, dictation and writing excercises in teaching a lesson about grocery shopping. Based on this information, which of the following is a valid conclusion? A. The teacher is applying Bloom's hierachy of cognitive learning. B. The teacher is teaching in a variety of ways because not all students learn in the same manner.

C. The teacher wants to make herteachirig easier by having less talk.

A. The second student has significantly higher intellectual ability

D. The teacher is emphasizing reading and writing skills.

B. The first student is probably below average, while the second has above average potential

69. Which one may support equitable access but may sacrifice quality? A. Open admission School accreditation

B.

C. Deregulated tuition fee hike D. Selective retention 70. Teacher A discovered that his pupils are very good in dramatizing. Which tool must have helped him discover his pupils' strength?

A. Portfolio assessment B. Performance test C. Journal entry D. Paper-and-pencil test 71. Two students are given the WISE II. One has a full scale IQ of 91, while the other has an IQ of 109. Which conclusion can be drawn?

A. It is not easy to administer. B. It puts the non-linguistically intelligent at a disadvantage. C. It utilizes so much time. D. It lacks reability.

C. Both students are functioning in the average range of intellectual ability

75. Which criterion should guide a teacher in the choice of instructional devices?

D. Another IQ test should be given to truly assess their intellectual potential

A. Attractiveness

72. Helping in the development of graduates who a maka-Diyos is an influence of A. naturalistic morality B. classical Christian morality C. situational morality D. dialectical morality 73. Teacher B clears his throat to communicate disapproval of a student's behavior. Which specific influence technique is this? A. Signal interference Direct appeal

B.

C. Interest boosting Proximity control

D.

74. In the context on the theory on multiple intelligences, what is one weakness of the paper-pencil test?

B. Cost

C. Novelty D. Appropriateness 76. What is most likely to happen to our economy when export continuously surpasses import is a thought question on . A. creating and-effect C. synthesizing predicting

B. relating causeD.

77. The teacher's first task in the selection of media in teaching is to determine the . A. choice of the students availability of the media

B.

C. objectives of the lesson technique to be used

D.

78. All men are pretty much alike. It is only by custom that they are

set apart, said one Oriental philosopher. Where can this thought be most inspiring? A. In a multi-cultural group of learners B. In multi-cultural and heterogeneous groups of learners and indigenous peoples' group C. In a class composed of indigenous peoples 79. If teacher has to ask more higher-order questions, he has to ask more questions. A. closed

B. fact C. concept D. convergent

80. Student Z does not study at all but when the Licensure Examination for Teachers (LET) comes, before he takes the LET, he spends one hour or more praying for a miracle, i.e. to pass the exam. Which attitude towards religion or God is displayed? A. Religion as fake B. Religion as magic C. Religion as authentic D. Religion as real 81. Teacher T taught a lesson denoting ownership by means of possessives. He first introduced the

rule, then gave examples, followed by class exercises, then back to the rule before he moved into the second rule. Which presenting technique did he use? A. Combinatorial B. Comparative C. Part-whole Sequence

D.

82. All subjects in Philippine elementary and secondary schools are expected to be taught using the integrated approach. This came about as a result of the implementation of . A. Program for Decentralized Education

C. Ask another question, an easier one. D. Wait for a response. 84. Standard deviation is to variability as mean is to A. coefficient of correlation central tendency

B.

C. discrimination index level of difficulty

D.

85. Quiz is to formative test while periodic is to A. criterion-reference test B. summative test C. norm-reference test

B. School-Based Management

D. diagnostic test

C. Basic Education Curriculum

86. Which teaching activity is founded on Bandura's Social Learning Theory?

D. Schools First Initiative 83. Which technique should a teacher use to encourage response if his students do not respond to his question?

A. Lecturing

B. Modeling

C. Questioning Inductive Reasoning

D.

A. Ask a specific student to respond, state the question, and wait a response.

87. In mastery learning, the definition of an acceptable standard of performance is called a

B. Tell the class that it will have detention unless answers are forthcoming.

A. SMART

B. criterion measure

C. behavior D. condition

88. Students' scores on a test were: 72, 72, 73, 74, 76, 78, 81, 83, 85. The score 76 is the . A. mode B. average mean D. median

C.

89. Test norms are established in order to have a basis for . A. establishing learning goals B. interpreting test results C. computing grades D. identifying pupils' difficulties 90. Which behavior is exhibited by a student who is strong in interpersonal intelligence? A. Works on his/her own. B. Keeps interest to himself/herself. C. Seeks out a classmate for help when problem occurs. D. Spends time meditating. 91. All of the following describe the development of children aged eleven to thirteen EXCEPT A. they shift from impulsivity to adaptive ability B. sex differences in IQ becomes more evident

C. they exhibit increase objectivity in thinking

A. Existentialism philosophy

D. they show abstract thinking and judgement

C. Idealism

92. A student passes a research report poorly written but ornately presented in a folder to make up for the poor quality of the book report content. Which Filipino trait does this practice prove? Emphasis on . A. art over academics B. substance over porma C. art over science over substance

D. porma

93. The principle of individual differences requires teachers to . A. give greater attention to gifted learners B. provide for a variety of learning activities C. treat all learners alike while in the classroom D. prepare modules for slow learners in class 94. Value clarification as a strategy in Values Education classes is anchored on which philosophy?

B. Christian D. Hedonism

95. Teacher E discussed how electricity flows through wires and what generates the electric charge. Then she gave the students wires, bulbs, switches, and dry cells and told the class to create a circuit that will increase the brightness of each bulb. Which one best describes the approach used? A. It used taxonomy of basic thinking skills B. It was contructivist C. It helped students understand scientific methodology D. It used cooperative learning 96. Rodel is very aloof and cold in his relationships with his classmates. Which basic goal must haye not been attained by Rodel during his developmental years, according to Erikson's theory on psychological development? A. Autonomy C. Initiative

B. Trust D. Generativity

97. Which type of report refers toon-the-spot description of some incident, episode or occurrence

that is being observed and recorded as being of possible significance?

C. Likert scales Anecdotal record

101. The first thing to do in constructing a periodic test is for a teacher to

A. Autobiographical report B. Biographical report

A. decide on the number of items for the test

C. Value and interest report D. Anecdotal report

B. go back to her instructional objectives

98. The main purpose of compulsory study of the Constitution is to

C. study the content D. decide on the type of test to construct

A. develop students into responsible, thinking citizens

C. Epicureanism Empiricism

D.

104. Teacher F wanted to teach the pupils the skill to do cross stitching. Her checkup quiz was a written test on the steps of cross stitching. Which characteristic of a good test does it lack? A. Scorability B. Reliability Objectivity D. Validity

C.

105. In self-directed learning, to what extent should a teacher's scaffolding be? A. To a degree the student needs it.

B. acquaint students with the historical development of the Philippine Constitution C. make constitutional experts of the students D. prepare students for law-making 99. The following are used in writing performance objectives, EXCEPT A. delineate

B. diagram

C. integrate

D. comprehend

100. Which can effectively measure students' awareness of values? A. Projective techniques Moral dilemma

D.

B.

102. Teacher F is convinced that whenever a student performs a desired behavior, provided reinforcement and soon the student will learn to perform the behavior on his own. On which principle is Teacher F's conviction based? A. Cognitivism Environmentalism C. Behaviorism

B.

D. Constructivism

103. Teacher U teaches to his pupils that pleasure is not the highest good. Teacher's teaching is against what philosophy? A. Realism

B. Hedonism

B. None, to force the student to learn by himself. C. To the minimum, to speed up development of student's sense of independence. D. To the maximum, in order to extend to the student all the help he needs. 106. What can be said of Peter who obtained a score of 75 in a Grammar objective test? A. He answered 75 items in the test correctly. B. He answered 75% of the test items correctly. C. His rating is 75.

D. He performed better than 5% of his classmates. 107. Which applies when skewness is zero? A. Mean is greater than the median

C. By reminding them your students your authority over them again and again. D. By giving your students a sense of belonging and acceptance.

C. Scores have three modes

111. Which types of play is most characteristic of a four to six-year old child?

D. Scores are normally distributed

A. Solitary and onlooker plays

108. Which measure(s) of central tendency separate(s) the top half of the group from the bottom half?

B. Associative and cooperative plays

B. Median is greater than mean

A. Median C. Median and Mean Mode

B. Mean D.

109. What was the most prominent educational issue of the mid 1980s? A. Bilingual Education Values Education

B.

C. Accountability Mainstreaming

D.

110. How can you exhibit expert power on the first day of school? A. By making them feel you know what you are talking about. B. By making them realize the importance of good grades.

C. Associative and onlooker plays D. Cooperative and solitary plays 112. For which may you use the direct instruction method?

A. Become aware of the pollutants around us. B. Appreciate Milton's Paradise Lost. C. Use a microscope properly. D. Distinguish war from aggression. 113. Read the following then answer the question TEACHER: IN WHAT WAYS OTHER THAN THE PERIODIC TABLE MIGHT WE PREDICT THE UNDISCOVERED ELEMENTS?

BOBBY: WE COULD GOTO THE MOON AND SEE IF THERE ARE SOME ELEMENTS THERE WE DON'T HAVE. BETTY: WE COULD DIG DOWN INTO THE CENTER OF THE EARTH AND SEE IF WE FIND ANY OF THE MISSING ELEMENTS RICKY: WE COULD STUDY DEBRIS FROM THE METEORITES IF WE CAN FIND ANY. TEACHER: THOSE ARE ALL GOOD ANSWERS. BUT WHAT IF THOSE EXCURSIONS TO THE MOON, TO THE CENTER OF THE EARTH, OR TO FIND METEORITES WERE TOO COSTLY AND TIME CONSUMING? HOW MIGHT WE USE THE ELEMENTS WE ALREADY HAVE HERE ON EARTH TO FIND SOME NEW ONES? Question: The Teacher's questions in the above exchange are examples of questions. A. fact direct

B. concept D. closed

114. The following are sound specific purposes of questions EXCEPT A. to call the attention of an inattentive student

C.

B. to teach via student answers C. to stimulate leamers to ask questions D. to arouse interestand curiosity 115.Teacher B engages her students with information for thorough understanding for meaning and for competent application. Which principle governs Teacher B's practice? A. Contructivist

B. Gestalt

C. Behaviorist Cognitivist

D.

116. In a study conducted, the pupils were asked which nationality they preferred, if given a choice. Majority of the pupils wanted to be Americans. In this case, in which obligation relative to the state, do schools seem to be failing? In their obligation to A. respect for all duly constituted authorities B. promote national pride C. promote obedience to the laws of the state D. instill allegiance to the Constitution

117. Read this question: How will you present the layers of the earth to your class? This is a question that A. directs B. leads the student to evaluate C. assesses cognition creative thinking

D. probes

118. A mother gives his boy his favorite snack everytime the boy cleans up his room. Afterwards, the boy cleaned his room every day in anticipation of the snack. Which theory is illustrated?

C. Give them the same work as the other students, not much, so that they won't feel embarrassed. D. Give them work on the level of the other students and work a little above the classmates level to challenge them 120. Which holds true to standardized tests? A. They are used for comparative purposes B. They are administered differently

A. Associative Learning B. Classical Conditioning

C. They are scored according to different standards

C. Operant Conditioning Pavlonian Conditioning

D. They are used for assigning grades

D.

119. What should a teacher do for students in his class who are not on grade level? A. Give them materials on their level and let them work at a pace that is reasonable for them, trying to bring them up to a grade level. B. Give them the same work as the other students, because they will absorb as much as they are capable of.

121. Shown a picture of children in sweaters inside the classroom, the students were asked this question: "In what kind of climate do these children live?" This is a thought question on A. inferring B. applying creating D. predicting

C.

122. In his second item analysis, Teacher H found out that more from the lower group got the test item # 6 correctly. This means that the test item .

A. has a negative discriminating power B. has a lower validity C. has a positive discriminating power D. has a high reability 123. Teacher A knows of the illegal activities of a neighbor but keeps quiet in order not to be involved in any investigation. Which foundational principle of morality does Teacher A fail to apply?

D. I must teach the child every knowledge, skill, and value that he needs for a better future. 125. Which guideline in test construction is NOT observed in this test item: Jose Rizal wrote. A. The central problem should be packed in the stem B. There must be only one correct answer.

B. The principle of double-effect

126. I combined several subject areas in order to focus on a single concept for inter-disciplinary teaching. Which strategy/method did I use?

124. As a teacher, you are a rationalist. Which among these will be your guiding principle? A. I must teach the child that we can never have real knowledge of anything. B. I must teach the child to develop his mental powers to the full. C. I must teach the child so he is assured of heaven.

A. Problem-entered learning B. Thematic instruction C. Reading-writing activity Unit method

D.

127. Which is a major advantage of a curriculum-based assessment? A. It is informal in nature. B. It connects testing with teaching.

128. Based on Edgar Dale's Cone of Experience, which activity is closest to the real thing?

A. View images

B. Attend exhibit D. Hear

129. Teacher Y does normreferenced interpretation of scores. Which of the following does she do?

D. The alternates must be plausible.

D. Between two evils, do the lesser evil.

D. It is based on a norm-referenced measurement model.

C. Watch a demo

C. Alternatives must have grammatical parallelism.

A. The end does not justify the means.

C. Always do what is right.

C. It tends to focus on anecdotal information on student progress.

A. She describes group performance in relation to a level of mastery set. B. She uses a specified content as its frame of reference. C. She compares every individual student’s scores with others' scores. D. She describes what should be their performance. 130. The typical autocratic teacher consistently does the following EXCEPT A. encouraging students.

B. shaming students. C. ridiculing students. D. intimidating students. 131. Which is one role of play in the pre-school and early childhood years? A. Develops competitive spirit. B. Separates reality from fantasy. C. Increases imagination due to expanding knowledge and emotional range. D. Develops the upper and lower limbs. 132. During the Spanish period, what was/were the medium/media of instruction in schools? A. The Vernacular B. English C. Spanish the Vernacular

D. Spanish and

134. The primary objective of my lesson is:To add similar fractions correctly.Before I can do this I must first aim at this specific objective:To distinguish a numerator from a nominator.What kind of objective is the latter?

B. The effect of environmental stimulation on a person's development.

A. Major Enabling

138. Which does NOT belong to the group of alternative learning systems?

B. Terminal D. Primary

C.

135. Which is/are the sources of man's intellectual drives, according to Freud? A. Id Id and ego

B. Superego D. Ego

C.

136. Which is an appropriate way to manage off-task behavior? A. Make eye contact. B. Stop your class activity to correct a child who is no longer on task.

C. The absence of genetic influence on a person's development D. The effect of heredity.

A. Multi-grade grouping B. Multi-age grouping C. Graded education D. Non-graded grouping 139. Which activity should a teacher have more for his students if he wants them to develop logicalmathematical thinking?

C. Move closer to the child.

B. Choral

133. An effective classroom manager uses low-profile classroom control. Which is a lowprofile classroom technique?

A. Problem solving reading

D. Redirect a child's attention to task and check his progress to make sure he is continuing to work.

C. Drama Storytelling

D.

A. Note to parents school detention

137. John Watson said: Men are built not born. What does this statement point to?

B. After-

C. Withdrawal of privileges D. Raising the pitch of the voice

A. The ineffectiveness of training on a person's development.

140. A goal-oriented instruction culminates in . A. planning of activities B. evaluation C. identification of topics

D. formulation of objectives 141. A teacher/student is held responsible for his actions because s/he _. A. has instincts mature

B. is

C. has a choice reason

D. has

142. The burnout malady gets worse if a teacher doesn't intervene to change whatever areas he or she can control. Which one can renew a teacher's enthusiasm? A. Stick to job B. Initiate changes in jobs C. Judge someone else as wrong D. Engage in self-pity 143. With indirect instruction in mind, which does NOT belong to the group? A. Problem solving recitation

B. Lecture-

C. Inductive reasoning D. Discovery 144. History books used in schools are replete with events portraying defeats and weaknesses of the

Filipino as a people. How should you tackle them in the classroom? A. Present them and express your feelings of shame. B. Present facts and use them as means in inspiring your class to learn from them. C. Present them and blame those people responsible or those who have contributed. D. Present them as they are presented, and tell the class to accept reality. 145. Teachers often complain of numerous non-teaching assignments that adversely, affect their teaching. Does this mean that teachers must be preoccupied only with teaching? A. Yes, if they are given other assignments, justice demands that they be properly compensated. B. Yes, because other community leaders, not teachers, are tasked to leadin community activities. C. NO, because every teacher is expected to provide leadership and initiative in activities for betterment of communities. D. Yes, because teaching is enough full time job.

146. "In the light of the facts presented, what is most likely to happen when ... ?" is a sample thought question on A. inferring

B. generalizing

C. synthesizing

D. justifying

147. Which is a true foundation of the social order? A. Obedient citizenry B. The reciprocation of rights and duties C. Strong political leadership D. Equitable distribution of wealth 148. For maximum interaction, a teacher ought to avoid questions. A. informational rhetorical C. leading

B. D. divergent

149. It is not wise to laugh at a two-year old child when he utters bad word because in his stage he is learning to . A. consider other's views distinguish sex differences C. socialize right from wrong

B.

D. distinguish

150. Which guideline must be observed in the use of prompting to shape the correct performance of your students?

A. Use the least intrusive prompt first. B. Use all prompts available. C. Use the most intrusive prompt first. D. Refrain from using prompts. 151. Bruner's theory on intellectual development moves from enactive to iconic and symbolic stages.

153. Teacher F is newly converted to a religion. Deeply convinced of his new found religion, he starts Monday classes by attacking one religion and convinces his pupils to attend their religious services on Sundays. Is this in accordance with the Code of Ethics of Professional Teachers? A. Yes. What he does is values education. B. No. A teacher should not use his position to proselyte others. C. Yes. In the name of academic freedom, a teacher can decide what to teach.

In which stage(s) are diagrams helpful to accompany verbal information?

D. Yes.What he does strengthens values education.

A. Enactive and iconic Symbolic

154. Which does Noam Chomsky, assert about language learning for children?

B.

C. Symbolic and enactive D. Iconic 152. If your Licensure Examination Test (LET) items sample adequately the competencies listed in the syllabi, it can be said that the LET possesses validity. A. concurrent construct C. content

B. D. predictive

I. Young children learn and apply grammatical rules and vocabulary as they are exposed to them. II. Begin formal teaching of grammatical rules to children as early as possible. III. Do not require initial formal language teaching for children. A. I and III B. II only D. I and II

C. I only

155. In a social studies class, Teacher I presents a morally ambiguous situation and asks his students what they would do. On whose theory is Teacher I's technique based? A. Kohlberg C. Piaget

B. Bandura D. Bruner

156. You arrive at knowledge by rethinking of latent ideas. From whom does this thought come? A. Experimentalist C. Idealist

B. Realist

D. Existentialist

157. A child who gets punished for stealing candy may not steal again immediately. But this does not mean that the child may not steal again. Based on Thorndike's theory on punishment and learning, this shows that A. punishment strengthens a response B. punishment removes a response C. punishment does not remove a response D. punishment weakens a response 158. After giving an input on a good paragraph, Teacher W asks her students to rate a given

paragraph along the elements of a good paragraph. The students' task is in level of A. application

B. analysis

C. evaluation synthesis

D.

159. Which is most implied by a negatively skewed score distribution? A. The scores are evenly distributed from left to the right B. Most pupils are achievers C. Most of the scores are low D. Most of the scores are high 160. How can you exhibit referent power on the first day of school?

A. By making them feel you know what you are talking about. B. By telling them the importance of good grades. C. By reminding your students your authority over them again and again.

ideas, which type of test should she formulate?

164. Which one can enhance the comparability of grades?

A. Multiple-choice type

A. Using common conversion table for translating test scores in to ratings

B. Short answer C. Essay

B. Formulating tests that vary from one teacher to another

D. Technical problem 162. Ruben is very attached to his mother and Ruth to her father. In what developmental stage are they according to Freudian psychological theory? A. Oedipal stage stage

B. Latent

C. Anal stage genital stage

D. Pre-

163. Behavior followed by pleasant consequences will be strengthened and will be more likely to occur in the future. Behavior followed by unpleasant consequences will be weakened and will be less likely to be repeated in the future. Which one is explained? A. Freud's Psychoanalytic Theory B. Thorndike's Law of Effect

D. By giving your students a sense of belonging and acceptance.

C. B. F. Skinner's Operant Conditioning Theory

161. If teacher wants to test students' ability to organize

D. Bandura's Social Learning Theory

C. Allowing individual teachers to determine factors for rating D. Individual teachers giving weights to factors considered for rating 165. Based on Freud's psychoanalytic theory which component(s) of personality is (are) concerned with a sense of right and wrong? A. Super-ego Ego C. Id

B. Super-ego and D. Ego

166. Which assumption underlies the teacher's use of performance objectives? A. Not every form of learning is observable. B. Performance objectives assure the learrier of learning. C. Learning is defined as a change in the learner's observable performance.

D. The success of learner is based on teacher performance. 167. Who among the following needs less verbal counseling but needs more concrete and operational forms of assistance? The child who . A. has mental retardation B. has attention-deficit disorder C. has learning disability

B. encourage students to talk about study habits from their own experiences C. has them view film strips about various study approaches D. gives out a list of effective study approaches 170. To promote effective practice, which guideline should you bear in mind? Practice should be

D. has conduct disorder

A. done in an evaluative atmosphere

168. We encounter people whose prayer goes like this: "O God, if there is a God; save my soul, if I

B. difficult for students to learn a lesson

have a soul" From whom is this prayer? A. Stoic Empiricist C. Agnostic

B. D. Skeptic

169. The best way for a guidance counselor to begin to develop study skills and habits in underachieving student would be to .

C. arranged to allow students to receive feedback D. take place over a long period of time 171. In Krathwohl's affective domain of objectives, which of the following is the lowest level of affective behavior? A. Valuing B. Characterization C. Responding Organization

A. has these underachieving students observe the study habits of excelling students

D.

172. With specific details in mind, which one has (have) a stronger diagnostic value?

A. Multiple choice test B. Non-restricted essay test C. Restricted essay test D. Restricted and non-restricted essay tests 173. What is the mean of this score distribution: 4, 5, 6, 7, 8, 9, 10? A. 7

B. 6

C. 8.5

D. 7.5

174. Availment of the Philippine Education Placement Test (PEPT) for adults and out-of-school youths is in support of the government’s educational program towards . A. equitable access quality

B.

C. quality and relevance D. relevance 175. With-it-ness, according to Kounin, is one of the characteristics of an effective classroom manager. Which phrase goes with it? A. Have hands that write fast. B. Have eyes on the back of your heads. C. Have a mouth ready to speak.

D. Have minds packed with knowledge.

176. Teacher B is a teacher of English as a Second Language. She uses vocabulary cards, fill-in-theblank sentences, dictation and writing exercises in teaching a lesson about grocery shopping. Based on this information, which of the following is a valid conclusion? A. The teacher is reinforcing learning by giving the same information in, a variety of methods. B. The teacher is applying Bloom's hierarchy of cognitive learning. C. The teacher wants to do less talk. D. The teacher is emphasizing listening and speaking skills. 177. With synthesizing skills in mind, which has the highest diagnostic value? A. Essay test Performance test

B.

C. Completion test choice tests

D. Multiple

178. How can you exhibit legitimate power on the first day of school? A. By making your students feel they are accepted for who they are. B. By informing them you are allowed to act in loco parentis. C. By making them realize the importance of good grades. D. By making them feel you have mastery of subject matter.

B. Inability of school to hire security guards. C. Deprivation of Filipino schools. D. Community's lack of sense of coownership. 181. Which of the following propositions is attributed to Plato? A. Truth is relative to a particular time and place. B. Human beings create their own truths.

179. Which questioning practice promotes more class interaction?

C. Learning is the discovery of truth as Latent ideas are brought to consciousness.

A. Asking the question before calling on a student.

D. Sense perception is the most accurate guide to knowledge.

B. Focusing on divergent question

182. In the parlance of test construction what does TOS mean?

C. Focusing on convergent questions. D. Asking rhetorical questions. 180. Theft of school equipment like tv, computer, etc. By teenagers in the community itself is becoming a common phenomenon. What does this incident signify? A. Prevalence of poverty in the community.

A. Table of Specifics Specifications

B. Table of

C. Table of Specific Test Items D. Team of Specifications 183. Read the following then answer the question: TEACHER: IN WHAT WAYS OTHER THAN THE PERIODIC TABLE MIGHT WE PREDICT THE UNDISCOVERED ELEMENTS?

BOBBY: WE COULD GO TOTHE MOON AND SEE IF THERE ARE SOME ELEMENTS THERE WE DON'T HAVE. BETTY: WE COULD DIG DOWN TO THE CENTER OF THE EARTH AND SEE IF WE FIND ANY OF THE MISSING ELEMENTS. RICKY: WE COULD STUDY DEBRIS FROM THE METEORITES IF WE CAN FIND ANY. TEACHER: THOSE ARE ALL GOOD ANSWERS BUT WHAT IF THOSE, EXCURSIONS TO THE MOON, TO THE CENTER OF THE EARTH, OR TO FIND METEORITES WERE TOO COSTLY AND TIME CONSUMING? HOW MIGHT WE USE THE ELEMENTS WE ALREADY HAVE HERE ON EARTH TO FIND SOME NEW ONES? Question: Which questioning strategy/ies does/do the exchange of thoughts above illustrate? A. Funneling and reaping

B. Sowing

C. Nose-dive Extending and lifting

D.

power or leadership does Teacher S have? A. Referent powerB. Legitimate power C. Reward power power

D. Expert

A. compact disc search B. manual search

185. By what name is Indirect instruction the Socratic method also known? A. Mastery learning Indirect Method

B.

C. Morrison method Questioning method

D.

186. In a treatment for alcoholism, Ramil was made to drink an alcoholic beverage and then made to ingest a drug that produces nausea. Eventually, he was nauseated at the sight and smell of alcohol and stopped drinking alcohoL Which theory explains this?

A. Operant conditioning B. Social Learning Theory

184. Referring to Teacher S, Nicolle describes her teacher asfair, caring and someone you can talk to.Which

telephone line to connect a computer library with other computers that have database is termed

C. on-line search D. computer search 188. Which one can best evaluate students' attitudinal development? A. Essay test

B. Portfolio

C. Observation answer test

D. Short

189. Which are direct measures of competence? A. Personality tests Performance tests

B.

C. Paper-and-pencil tests D. Standardized test 190. In instructional planning it is necessary that the parts of the plan from the first to the last have .

C. Associative Learning D. Attribution Theory 187. The search for related literature by accessing several databases by the use of a

A. clarity B. symmetry C. coherence D. conciseness

191. The cultivation of reflective and meditative skills in teaching is an influence of . A. Shintoism Buddhism C. Confucianism

B. Zen D. Taoism

192. With which goals of educational institutions as provided for by the Constitution is the development of work skills aligned? A. To develop moral character B. To teach the duties of citizenship C. To inculcate love of country D. To develop vocational efficiency 193. Researchers conducted show that teacher's expectations of students become. Do not require initial formal language teaching for children self-fulfilling prophecies. What is this phenomenon called? A. Halo effect Pygmalion effect C. Ripple effect effect

B. D. Hawthorne

194. Under which program were students who were not accommodated in public elementary and secondary schools because of lack of classroom,

teachers, and instructional materials, were enrolled in private schools in their respective communities at the government's expense? A. Government Assistance Program B. Study Now-Pay Later C. Educational Service Contract System D. National Scholarship Program 195. How would you select the most fit in government positions? Applying Confucius teachings, which would be the answer? A. By course accreditation of an accrediting body B. By merit system and course accreditation C. By merit system D. By government examinations 196. How can a teacher enhance his/her questioning technique for an effective teacher-student interaction? A. Immediately B. You may answer your own question if no one can answer

C. Allow sufficient “think time” at least 7 to 10 seconds D. Extend wait time until the students respond 197. A teacher who advocates the pragmatic philosophy of education believes that experience should follow learning thus she has to? A. Equip her students with basic skills and abilities B. Encourage her students to memorize facts C. Provide her students opportunities to apply their skills and abilities D. require her students mastery of the lesson 198. Respect for honest differences of opinions is one objective A. Human relationship B. self-actualization C. Civic responsibility and conscience D. Economic self sufficiency 199. In mastery learning, the definition of an acceptable standard of performance is called A. SMART

B. Condition C. Criterion Measure D. Behavior 200. Teacher Luke clears his throat to communicate disapproval of a student’s behavior. Which specific influence technique is this? A. Proximity control B. Interest boosting C. Signal Inference D. Direct appeal 201. The main purpose of compulsory study of the Constitution is to __________ A. develop students into responsible, thinking citizens B. acquaint students with the historical development of the Philippine Constitution C. makes constitutional experts of the students D. prepare students for law-making 202. In self-directed learning, to what extent should a teacher's scaffolding be? A. To a degree the student needs it.

B. None, to force the student to learn by himself.

fulfillment of man’s primordial obligations are called

C. To the minimum, to speed up development of student's sense of independence.

A. Perfect rights rights

D. To the maximum, in order to extend to the student all the help he needs 203. Which is the first step in planning an achievement test? A. Select the type of test items to use. B. Decide on the length of the test. C. Define the instructional objective D. Build a table of specification 204. Which may NOT be a benefit derived from the use of graphic organizers? A. Make relationships among detail clear B. Enable students to identify important ideas and details C. Strengthen team work D. Represent stated information in concrete form 205. Rights which cannot be renounced or transferred because they are necessary for the

B. Alienable

C. Acquired rightsD. Inalienable rights 206. Principal Miguel shares this thought with his teachers: "Subject matter should help students understand and appreciate themselves as unique individuals who accept complete responsibility for their thoughts, feelings, and actions." From which philosophy is this thought based? A. Perennialism B. Essentialism C. Existentialism D. Progressivism 207. How can a teacher establish the reliability of a test? 1) Repeat the same test 2) Administer a parallel test 3) Split the test 4) Vary the number of items A. 1, 2 and 3 4

B. 2,3 and

C. 1 and 4 only only

D. 2 and 4

A. Students are governed by fixed and rigid standards

C. Involve students actively in the questioning process

208. On which policy is R.A. 4670 known as the Magna Carta for Public Teachers focused?

C. Life-like situations introduced as learning experience in the classroom

D. as teacher, you ask no questions

A. Right to establish or join an organization

C. Students are given more opportunity to act or experience learning.

B. Code of ethics for professional teachers C. Recruitment shall take place after training D. Promotion and improvement of social and economic status of public school teachers 209. the 19th century saw the development of state controlled and state supported public school system. Existing ideology was the concept of national sovereignty. Under a democracy, education equips citizens with the principles and duties of citizenship to guarantee national stability. Such is the ideology of __________ .

D. Evaluation is made as an integral part of teaching procedures. 211. Which of the following is NOT a principle of development? A. Development follows an orderly, predictable sequence B. Numerous studies show how individual develop C. Early development is more critical than later development D. Social expectations influence development in early stage

A. Citizenship Progressivism

B.

212. Which questioning technique would be appropriate for inductive lessons?

C. Nationalism Education

D.

A. Use questions requiring only memory responses

210. The following are the characteristics of a good teaching/learning practice EXCEPT?

B. Expect participation only among the more motivated students

213. What principle is reflected when a teacher always provides for the development of all essential knowledge manipulative skills and attitudes? A. Principle of Needs of Balance

B. Principle

C. Principle of Unity of organization

D. Principle

214. A teaching method proceeds from the details of a lesson towards generalization is called: A. Deductive C. Debate solving

B. Inductive D. Problem

215. To accomplish many good activities in a particular class period, the teacher must

A. Add curricular activities B. Provide unique teaching aids C. Institute a systematic plan D. Use expensive material 216. one way to encourage students who lack in reading to love reading is to:

A. Reprimand himB. Ask the students to buy books C. Provide reading materials D. Isolate Him 217. Which of the following style or technique would make the teacher a good classroom manager– teacher with eyes at the back of the head? A. Give your students the “head to toe” look when angry B. Uses eye contact technique with your students C. Fear and fight dominates the classroom atmosphere D. Being aware of all the actions and activities in the classroom 218. The sociogram reveals that four students is your class formed a “clique”. This means that the teacher should: A. Allow them to be together all the time B. Encourage them to join the groups C. Discourage them from joining other groups D. disband the group

219. Parents negatively react to the inclusion of play in children’s curriculum. A teacher who knows the importance of play would tell the parents that __________?

222. The recommended method to use if the teachers wishes each learner to concentrate in learning a topic to his skills are properly assessed is.

A. Playing is important in the child’s cognitive development

A. Semantic webbing

B. playing is a good form of exercise C. Playing gives children pleasure D. Playing is a stage of child’s development 220. The ability to perceive how objects are related in order to mentally perceive what is seen, thus creating concrete visual images from memory refers to A. Visual spatial intelligence B. Musical Intelligence C. Linguistic intelligence D. Logical reasoning intelligence 221. The _________ method is used when the learners are made to observe things in a certain place like the market.

B. Independent study C. Role playing D. Field trip 223. The ________ method is observed if we wish the pupil learn from real life situation dilemmas. A. Situation

B. Lecture

C. Textbooks Observation

D.

224. The ________ method is used to make the learners study in detail a specific thing, person or place not known to them A. Case study Participation C. Project

B. D. Field trip

A. Participatory study

B. Case

225. Which of the following method will you used to verify a certain findings and to make the learners handle apparatus properly?

C. Simulation

D. Field trip

A. Textbook method Laboratory method

B.

C. Field trip method Project method

D.

226. The ________ method is utilized if the learners are trained to do creative products. A. Project Field trip

B. Case study D. Simulation

C.

227. If the material is dangerous for the learners to handle, which of the following method will you use? A. Textbook discussions

B. Group

C. Lecture-demonstration Eclectic

D.

228. Which method is used to develop scientific inquiry among the learners? A. Project

B. Case study

C. Problem solving Simulation

D.

229. The ________ method is used to find out the learner’s knowledge about a certain topic assigned to them. A. Independent study C. Lecture answer

B. Textbook

D. Question and

230. The ________ approach is utilized when the learners are trained to ask intelligent question. A. Process

B. Discovery

C. Inquiry certification

D. Value

231. If you wish to relate a subject matter to one of the four principles of learning, the ________ approach should be used. A. Multidisciplinary Interdisciplinary

B.

C. Conceptual Integration

D.

232. The ________ approach is used if the teacher wishes to solve a problem being met in the school. A. Mastery

B. Integration

C. Action learning D. Value clarification 233. You wish to make the learners learn or internalized fully a subject matter to be taught to them. Which of the following will you need? A. Integrated media C. Master disciplinary

B. MultiD. Multi-

234. Which of the following approaches will be used if you wish to relate a particular subject to all disciplines of learning? A. Multidisciplinary Interdisciplinary

B.

C. Value clarification Integration

D.

235. To enable the learners to learn by their own pace of growth, _________ approach is used. A. Inquiry Mastery

B. Discovery D. Modular

C.

236. The ______ approach is observed when the learners want to meet the criterion level of success act set by the teachers. A. Mastery learning Interdisciplinary

B.

C. Conceptual

D. Modular

237. Which approach is used to emphasize the skills in informing conclusions? A. Multidisciplinary Interdisciplinary

B.

C. Conceptual

D. Inquiry

238. Which of the following approaches is used to include issues confronting the societies?

A. Integration Interdisciplinary

B.

C. Conceptual

D. Inquiry

239. The _______ approach is used to make the learners enunciate their feelings or attitudes about certain issues. A. Value clarification Mastery

B.

C. Integration Interdisciplinary

D.

240. The _________ method is used if the learners are to use their senses effectively. A. Textbook demonstration

B. Lecture

C. Observation Independent study

D.

241. The most important criterion in test construction is A. Preparation of table of specifications B. Congruency of the items with objectives C. Stem should contain the central problem D. Options should be almost the same length

242. Which of the following is the most important contribution of Gestalt psychology to the theories of learning?

245. What is possessed by the learner when he can use language with ease and fluency in any given situation?

A. Cognitive insight

A. Bilingual ability cognitive skills

B. The use of multimedia approaches C. The concept of readiness in learning D. The use of reinforcement 243. What psychological principle is used when teacher links the new information to the previous one to enable the students to gain a holistic view of the topic? A. Stimulation Accommodation

B.

C. Assimilation conceptualization

D.

244. Young children have a short attention and interest span. What kind of task should the teacher give them? A. Challenging and interesting activities B. Long but interesting activities C. Easy and difficult activities D. Short, varied, interesting activities

B. High

C. A photographic memory D. Communicative competence 246. What is the main concern of spiraling a curriculum? A. curriculum renewal and revision B. Horizontal articulation among the students in a grade level C. Vertical articulation of a given subject across a grade level D. Incorporating government thrusts and societal concerns 247. Which of this information is not entered in Form 1 or the School Registrar? A. Alphabetical list of students, boys separated from girls. B. Daily attendance record of each student. C. Grade obtained by each student in all his/her subjects. D. Personal data of the students in the registrar.

248. Which type of test is used to discover further attitudes about self and

A. Personality test Intelligence test

B.

C. Achievement test Diagnostic test

D.

249. Which behavior is exhibited by a student who is strong in interpersonal intelligence? A. Works by his/her own. B. Spends time meditating. C. Keeps interest to himself/herself. D. Seeks out a classmate for help when problem occurs. 250. A parent visited you regarding his son’s low grades. He showed you his quizzes, unit tests and projects. You discovered that his name is Mel and you wrongfully put his name on the girl’s list. What will you do? A. Recognize your mistake and promise to correct the grade B. Insists: that you are right in grading him.

C. Refer the matter to the principal

A. Call for their parents.

D. Ignore the complain

B. Refer them to a doctor.

251. What should a cooperating teacher do to help the student teacher who has been assigned to him/her? A. Provide opportunities for the student teacher to acquire the skills and competencies to be an effective teacher. B. Show your lesson plan and let him/her follow what is in your lesson plan.

C. Give them remedial class Suspend them from classes

D.

253. Children learn what they live by. Treat them with respect and they will respect others. Shout at them and they will be shouting at others, too. How would you explain this behavior? A. They are easily impressed B. They are imitative C. they cannot tell right from wrong D. They are observant

C. Write or prepare activities to be done and let him/her execute these in class

254. What should be done with a student in the upper grades who is a non-reader?

D. Don’t allow him/her to make her own decisions as to how the lesson is to be introduced.

A. Encourage him to join a reading club

252. Which of these techniques is BEST suited to developing skill in asking and answering questions? A. Interviews analysis

B. Song

C. Pictorial review

D. Riddles

252. Most delinquents are found to have low IQ. They can hardly read and comprehend. How can a teacher help them?

B. Give him comic books C. Have him attend remedial reading class D. Transfer him to lower section 255. In which situation is learning most likely to happen? A. When students work by themselves

B. When students are quiet and well behaved

C. Teacher-made test Standardized test

C. When all the needed materials are available

259. A high school principal would like to know the causes of dropouts in his school so he could find solutions to this problem. What type of researched is used?

D. When students know the importance of the task at hand 256. Social adjustment means the ability to behave in accordance with? A. Universal truths

B. Action

C. Pure Experimental

D.

260. If we aim to produce globally competitive graduates, the Philippine education should give major emphasis to _______________.

B. Self-concept C. Stereotyped behaviors D. Social expectations 257. How can a teacher avoid “breakdown and interruptions” I daily class procedures?

A. Punish the misbehaving students B. Establish routine for daily tasks C. Assign a leader to assist everyone D. Allow students to make their own regulations 258. Which is made after certain norms has been established? A. Departmental test Local-city wide test

A. Applied

D.

B.

A. English, Science and Mathematics B. Technology and citizenship education C. Humanities and work education D. Bilingual education and values education 261. Which of the following can be considered a form of civic engagement? A. Patronizing Filipino products B. Critical of government officials C. Bribing government officials D. Electoral participation

262. A teacher wants to make a rubric for scoring students' output. Which format will use differential weighs for the qualities of a product or a performance? A. Performance based scales

B. Rating

C. Holistic rubric rubric

D. Analytic

263. How can a teacher develop the value of time such as punctuality and maximal utilization of time? A. Avoid disruptions due to improper behavior B. Consistently follow schedule for classroom routine C. Rush if you are getting late for the right time D. Have a big clock installed in the classroom for everyone's guidance 264. One learns by association and also by insight. This shows that the association and cognitive theories of learning are: A. Diametrically opposed B. Complementary C. Partly wrong Partly correct

D.

265. Mothers who demand their 3 to 5 year old child to suspend their time in serious academic study, forget that early childhood is in the A. Gang age Questioning age

B.

C. Initiative age

D. Toy age

266. Identical twins are more than alike than fraternal twins. Which of the following statements/principles is supported by this? A. Heredity has a part in determining physical appearance B. Intelligence is determined partly by pre-natal nutrition C. Environment affects both fraternal and identical twins D. Intelligences hinges in physical structure 267. Which of the following activities is stressed by humanistic education?

A. Enjoy the great works of man such as classics B. Learn the philosophy “know thyself” C. Make the distinctly civilized, educated and refined

D. Develop man into a thinking individual 268. For a teacher to be competent, he/she is required to specialize in certain area. This pillar of learning is

271. Multiple choice test is considered the BEST type of test because _______________ A. It is easy to conduct B. It contains many responses

A. Learning to do to live together

B. Learning

C. measure several competencies in one test

C. Learning to be to know

D. Learning

D. It possess the qualities of other types of tests

269. What pillar of learning is concerned on material development rather than of human development?

272. While serving during the elections, some trouble makers enter your precinct. What do you think is the most appropriate thing to do?

A. Learning to do B. Learning to live longer

A. Challenge them to a duel

C. Learning to be to know

D. Learning

270. What is the advantage of using computers in processing test results? A. Its processing takes a shorter period of time. B. Test results can easily be assessed. C. Its statistical computation is accurate. D. All of the above.

B. Close the precinct and go home C. Ignore them and look for a safe place D. Seek the assistance of law enforcers 273. Which of the following will you recommend to a senior high school scholar who is impregnated by a fellow student? A. Force her boyfriend to marry her B. Tell the parents about the condition

C. Stop schooling till after she gave birth

most likely inspire her to cling to her idealism?

A. Idealist Empiricist

D. Direct her to an abortion clinic

A. Introduction of educational innovations

281. The process by which certain potentials are inherited from the parents for his development

274. Which describes an inappropriate practice in the education of young children? A. Individual differences are expected and accepted B. Integrated teaching-learning C. Isolated skills development D. positive guidance techniques 275. Honesty remains a value even if nobody in a organization values it. This pronouncement comes from the mouth of a(an) A. Pragmatist

B. Idealist

C. Reconstructionist Progressivist

D.

276. I cannot forget my friend’s birthday for it comes one day after my birthday. Which principle of association as applied to memory explains this? A. Contiguity Similarity

B.

C. Frequency

D. Content

277. A fresh teacher graduate is usually idealistic. Which one will

B. Support of living models

A. Life

C. Pressure of work D. High salary 278. I cannot help but recall the sister’s convent which served as my boarding house in high school now that I am in a noisy boarding house. Which principle of association explains this? A. Similarity Contiguity

B.

C. Frequency

D. Contrast

B. Realist D. pragmatist

C.

B. Birth C. Heredity D. Character

282. This theory states that there are 8 basic development stages that the individual has to pass through his life A. Learning Theory B. Psychoanalytic Theory C. Psychosocial Theory D. Cognitive Development

279. Which principle of association ass applied to memory is this? The recall of an object or idea triggers recall of other objects like it A. Contrast B. Contiguity Similarity D. Frequency

C.

280. Whose thought is this: “Although there is an external world from which human beings acquire sensory information, ideas originate from the workings of the mind.”

283. Transition age from childhood to adulthood where rapid physical changes and sex maturity occur resulting in changes in ways of feelings, thinking and acting. A. Puberty

B. Adolescence

C. Early Adulthood

D. Stage V

284. Modifying an existing scheme after an individual’s interaction with the environment, resulting in the creation of a new scheme. A. Assimilation Interaction

B.

C. Recognition Accommodation

D.

C. Laurence Kohlberg Martin Lesley

285. Theory stating that a person’s behavior can be motivated by urges towards self-satisfaction.

D.

289.“Intellectual appreciative experience” is … A. Base on the premise that all learning has emotional correlates

291. Any change in the behavior of an individual A. Learning

B. Response

C. Change

D. Development

B. Obtained in the field of music, art and literature

292. Which of the following principles IS NOT considered under Classical Conditioning by Ivan Pavlov?

D. Moral development theory

C. the acquisition and retention of facts and information

A. Excitation Adhesive Principle

B.

286. The ability of a child to conceptualize the retention and preservation of the same quantity under various changes.

D. assumes that human activities are based on stimulus and response

C. Stimulus Generalization None of the above

D.

A. Psychoanalytic Theory B. Cognitive Development Theory C. Psychosocial Theory

A. Recognition

B. Reversibility

C. Assimilat8ion D. Conservation 287. Refers to the idea that no individual are exactly the same or alike. A. Cognitive theory Exclusivity theory

B.

C. Individual Differences D. Emotional Quotient 288. He is known as the Father of Modern I.Q. Test A. Lewis Terman Erickson

B. Erick

290. These statements imply that children at the early learning stage consider parents and teachers as authorities and models. A. Parents and teachers should always coordinate children’s activities B. Parents should enforce strict discipline at home and teachers in school C. Parents and teachers should be the role models at all times D. Parents and teachers should always consult each other with regards the child’s intellectual development

293. This stimulation of action best explains the behavior of an individual to take what he perceives to be the shortest route to his goals.

A. Recognition Assimilation

B.

C. Response Motivation

D.

294. The process by which an individual acquires the social and cultural heritage of the society where he belongs. A. Socialization B. Internalization

C. Integration D. Acquisition 295. What is the main function of the philosophies of education? A. To aid the leaner to build his/her own personal philosophy.

B. Man should learn the different philosophies of education C. Build a man who is distinctly civilized, educated and refined D. Develop man into a thinking individual

B. Define goals and set directions from which educational efforts should be exerted.

298. A teacher who advocates the pragmatic philosophy of education believes that experience should follow learning, thus, she has to?

C. Provide guidelines in the foundation of educational policies and programs

A. require her student mastery of the lessons

D. Provide norms and standards for evaluation purposes. 296. According to Froebel, kindergarten is also known as A. “A place where children have fun and enjoyment” B. “Garden where children could grow and develop” C. “The learning center of life” D. “A place where new beginnings begin” 297. Which of the following statements is given emphasis by “humanistic education?” A. The great works of man such as the classics should be enjoyed.

B. encourage her students to memorize facts C. equip her students with basic skills and abilities D. provide her student with opportunities to apply their skills and abilities 299. How are institutions of learning encouraged to set higher standards over and above the minimum requirement for state recognition? A. Scholastic achievement B. Faculty development C. Academic freedom Voluntary accreditation 300. The period of physical, especially sexual, and mental

D.

maturation which is characterized by rapid somatic growth is known as A. infancy childhood

B. early

C. Puberty

D. Adulthood

301. Claustrophobia is an irrational fear of A. Darkness

B. Strangers

C. Closed Space

D. Height

302. An eye defect characterized by clear vision in one dimension but unfocused vision on the other is called A. Myopia

B. Astigmatism

C. Hyperopia Presbyopia

D.

303. Which of the following statements does not apply to adolescents? A. They desire the approval of their peers B. They seek dependence on their parents C. They have a marked sex development D. None of the above

304. As young people mature, society expects them to develop competencies and assume social roles in a conventional manner.

D. there is no marked differences in their time of maturity

A. it remains fairly constant is highly changeable

B. it

308. Rationalization is used by student who

C. it is affected by attitude is never constant

D. it

A. expectation of parents B. influence of peers groups C. influence of formal education D. cultural demands 305. The founder of the theory of psychology called psychoanalysis was A. Lock B. Hume D. Leibnitz

C. Freud

306. When the learner reaches a point where no further improvement can be expected, he is in a so-called A. Development crisis plateau

B. Learning

C. Regression Depression

D.

307. Regarding the sexual maturation of boys and girls, teachers should bear in mind that: A. Girls mature at a late stage than boys B. Girls mature at an earlier stage than boys C. Boys and girls mature at the same time

A. Always give explanation or reason for their failures rather than own their faults B. Like to take the blame for their faults C. bribe their elders with promises

312. Transfer of training easily takes place if the activities involved A. Are different B. Have identical element C. Occur in the same place D. Vary in difficulty

D. substitute words for deeds

313. When the learner is wellmotivated, he performs his task

309. Which of the following is true of Abnormal Psychology?

A. with indifference B. with disinterest

A. It studies the cause of personality defects

C. with arrogance

B. It measures the accomplishments of the individual

D. with enthusiasm

C. it concentrates on the scholastic performance of the individual

314. A six-year-old child who has a mental age of eight years has an IQ of

D. it investigates the educational background of the individual

A. 120 133

310. Which of the following is a continuous variable?

315. The ratio obtained by dividing mental age by chronological age times 100 is called

A. Weight Nationality

B. Sex D. Race

C.

311. Which of the following is true about one’s IQ?

B. 130 C. 132

A. derived quotient deviation

D.

B.

C. Intelligence quotient or IQ D. Intelligence ratio 316. Which of the following was written by Plato? A. Sic et Non School and Society

B. The

C. The Republic

D. Emile

317. Who among those below asserted that “Education is for complete living?” A. Dewey

B. Spencer C. Kant D. Froebel

318. The right of an educational institution and its faculty to prescribe the methods/strategies of teaching refers to: A. Building style B. Choice of curriculum C. Academic freedom D. Co and extra-curricular program 319. The 1987 Constitution provides that religious instruction can be given A. with the students’ consent B. with the parent/guardian approval

C. with mayor’s permit

320. Public schools in the Philippines are the contribution of which colonizer?

323. A teacher who gives a uniform assignment to be worked out by all learners in Arithmetic is not observing a characteristic of a good assignment. Which characteristic is overlooked?

A. American

B. British

A. It should be definite

C. Japanese

D. Spanish

B. It should be stimulating

D. with the school’s support

321. Hardship allowance is given to a teacher when

C. It should emphasize the essential

A. s/he’s assigned in a depressed area

D. It should provide for individual differences

B. s/he’s given additional teaching load

324. If a student asks a question which the teacher does not have a ready answer, the latter should:

C. s/he’s assigned in a barangay high school D. s/he’s assigned in a hazardous area 322. Which of the following principles of human growth refers to the situation that girls mature earlier than boys? A. Growth follows a sequential pattern B. Development rates vary C. Each stage of development has characteristics traits D. Maturation should precede certain type of learning

A. Dismiss the question as irrelevant B. Offer a bluff C. Admit the fact that he doesn’t know the answer D. Ask volunteers to answer the question and do research on it later. 325. The heredity traits acquired by a person in his lifetime A. Are transmissible to his offspring B. Reappears in his future grandparent

C. Have no influence on the offspring

C. That the girl has a chronological age of 15

A. Visual intelligence Linguistic Intelligence

B.

D. Become recessive traits

D. That she has a mental age of 13

C. Feelings sensitivity

D. Jargon

326. When student are given a chance to settle differences of opinion by discussion, they develop:

329. Which statement is not necessary to achieve the learner’s interest in a learning activity?

332. The sensitivity to tone and pitch, allowing one to produce musical scoring is intelligence in?

A. The activity must lead to a practical end

A. Musical Ability

B. The activity must be within the ability of the learner

C. Quantitative exercises D. Qualitative analysis

C. The activity must fill a need recognized by the learner

333. One’s ability to do abstract reasoning and manipulate symbols refers to what type of intelligence?

A. Fair play

B. Tolerance

C. Irritants

D. sociability

327. The school’s responsibility towards teenagers “gang age” is: A. Provide the gang all the freedom it needs B. Gives classroom activities to give direction to out-of-school youth activities C. Supervise gang activities D. set up norms of conduct or the member of the gang

328. In an intelligence test, a 13year old girl got a score equivalent to that of a 15-year old. This means: A. That the girl must be accelerated B. That the girl 12-years older mentally

D. The learner must have the experience that will furnish the background for the activity 330. He is responsible for the theory which recognizes the importance of developing multiple intelligence? A. Jean Piaget Gardner

B. Howard

C. Frederick Froebel Freud

D. Sigmund

331. The need to recognize and develop special sensitivity to language, thus helping the learners to use the right word, phrase and/or graph to grasp new meaning refers to

A. Musical identification

B. Verbal

B. Personality

C. Mental ability D. Mathematical-logical 334. The ability to perceive how objects are related in order to mentally perceive what is seen, thus creating concrete visual images from memory refers to? A. Visual-spatial Intelligence B. Musical C. Language D. Logical reasoning 335. The capacity to analyze one’s feelings and thus be able to understand and be able to know

the motives of other people’s actions.

other, adjustments to varying situations, behavior an motivations

A. Spatial

B. Personal

A. Emotional Quotient (E.Q.)

C. Logical Diametric

D.

B. Intelligence Quotient (I.Q.) C. Maladjustment personality D. Anticipated behavior

336. The type of intelligence which enables a person to understand other person’s feelings, behavior and motivation. A. Emotional Spatial

B.

C. Multiple intelligences

337. The type of intelligence which characterizes actress, actors, mimes, dancers and people of the Arts?

C. Research Emotions

A. Emotional quotient (E.Q.) B. Intelligence Quotient (I.Q.)

C. Social intelligence D. Quantitative and Qualitative

A. Bodily-kinesthetic Scientific

339. It is a measurement of personality which is the result by dividing the mental age by the chronological age.

B. D.

338. An emerging thrust in determining one’s personality, whether pleasant or unwholesome, this type of personality measurement is the wholesomeness of one’s virtues, i.e., values, relationships with

D. Forecasted behavior quotient 340. The teacher must be aware that both heredity and environment represent complex factors, exerting many specific influences on an individual’s growth. Which of the following statements best represents the influence of heredity and environment? A. Heredity counts; environment is less important. B. If the environment is changed, heredity becomes less important.

C. The relative influences of heredity and environment can vary widely in an individual’s growth D. In the long run, both tend to cancel each other’s influences 341. Heredity has a part in determining intelligence. Which of the following statements support this principle? A. Environment affects both fraternal and identical twins B. Intelligence hinges in physical structures C. Intelligence is determined partly by pre-natal nutrition D. Identical twins are more alike than fraternal twins 342. Educators who contributed to the “open education” movement include: A. Neill and piaget Kohl and kozol

B.

C. Bruner and Silberman D. All of the above 343. A child’s social skills can be measured by: A. direct observation and parentteacher conferences B. psychological test

C. adaptive behavior scales D. A and C above 344. A teacher uses behavioral modification techniques in his classes. Which of the following student behaviors would he find most difficult to change?

B. They have short attention spans and experience difficulty in generalizing C. Their reading, writing, and arithmetic skills cannot be improved D. A and B above

A. Aggressive tendencies towards classmates

347. Which of the following are characteristics of a dyslexic child?

B. Poor habits in organizing work materials

A. Mirror writing listlessness

C. Interrupting a speaker

C. Below-average intelligenceD. Hyperactivity

D. Abandoning a project before it is finished 345. Learning-disabled children most characteristically have:

B.

C. An average level of intelligence D. minimal brain damage 346. Which of the following is true about educable mentally retarded children? A. Their IQ range between 50 and 70

B. Eliminate testing C. Eliminate extrinsic motivations D. maintain a certain anxiety level for increased competition 350. The knowledge explosion has led to crowding more and more information into curriculum courses. A likely result is that: A. the textbook will no longer be the main instructional medium in many classes

348. Students with secondary reading problems have capacity to read, but are non-readers because of:

B. the child may spend more time in school

A. auditory problems

D. all of the above

B. congenital defects

351. During the learning process the teacher has most control over:

A. low IQ B. poor socio-economic backgrounds

A. Meet both physiological and intellectual needs of students

C. visual-acuity impairment D. Environmental or Emotional factor 349. If a teacher accepts Maslow’s theory on the hierarchy of needs, he or she will probably structure objectives to:

C. the teacher may have to rely more on these multimedia materials

A. The learners B. The learning environment C. The Learning process D. The behavior of the learners 352. Which of the following conditions does NOT contribute to

a climate psychologically suited to learning? A. The teacher acts like a “real person.” B. The teacher makes all of the decisions about students learning activities.

D. A sensitivity-training meeting for the purpose of helping students ace their school-related problems and learn how their actions can affect others.

LET REVIEWER GENERAL EDUCATION- SOCIAL STUDIES

C. The teacher accepts students as they are

1. Which statement is TRUE of the pre- Spanish Filipino government?

D. The teacher shows trust in students’ decisions

a. The datu exercised all the powers of government.

353. William Glasser advocates the frequent use of classroom meetings, with teacher and students sitting in a small circle. Which one of the following types of discussion would NOT be appropriate in such a setting?

b. Laws were formulated by a law making body elected by the datu.

A. An educational-diagnostic conference on the learning weaknesses of individual students B. .An open.-ended meeting for the purpose of exploring and discussing student’s ideas about the curriculum. C. A social-problem-solving meeting to resolve teacher or student problems relating to the school, the class, or any individual member.

c. Laws were formulated by a law making body elected by the community. d. There was a court created by the datu to hear complaints. 2. What characteristic/s of the government is established by the 1987 Constitution? I. Presidential system of government with three branches. II. Parliamentary system of government. III. The three branches of government are separate and independent of one another.

IV. The three branches of government have a check and balance over one another.

5. In the decade of the 70’s, one clamor of the activists who staged street demonstration on

a. I only

the streets was “Down with the oligarchs!” What did they accuse government of as

b. II only c. II, III, and IV d. I, III, and IV 3. If our present government is a democracy, where does power reside?

d. Capitalism

b. A form of dictatorship

9. In which form of government does power or authority reside in a few persons who govern

c. Anti-poor

6. With the Batasang Pambansa performing legislative and executive powers in the Marcos

about the Philippine Commonwealth? The Filipino people _____________________. a. Were fully independent. b. Were partially independent. c. Were granted total freedom to course their destiny only in matters of education. d. Were citizens of the United States.

b. Totalitarianism

a. A rule of the few rich

b. In Congress

4. In President Quezon’s time, the country had the Philippine Commonwealth. What is TRUE

a. Colonialism

c. Democracy

d. Elitist

d. In the Supreme Court

8. To which type of political system do we belong?

suggested by the underlined word?

a. In the Filipino people

c. In the President

d. Monarchical

regime, which form of government was implemented? a. Parliamentary b. Dictatorial c. Monarchial d. Presidential 7. What form of government is characterized by the separation of powers? a. Parliamentary b. Presidential c. Aristocracy

for their own interest? a. Oligarchy b. Monarchy c. Democracy d. Parliamentary 10.Which of the following represents the smallest subunit of government in the Philippines? a. Purok b. Barangay c. Sitio d. Zone 11. The Philippine government is divided into three branches: executive, legislative and

judiciary. Which among the following doctrines best describes the model of governance in the Philippines? a. Tricameralism b. Separation of powers c. Emancipation of state d. Division of labor 12.The Philippine legislature is divided into two major bodies, the Senate and the House of Representatives. Which among the following describes this division of the legislative body of the country? a. Bipartisanism b. Co- legislative power c. Unicameralism d. Bicameralism 13.Which type of governance is characterized by a union of partially self- governing states or

c. Democracy

b. Right to secure persons

d. Totalitarianism

c. Right to process paper

14.In England, Queen Elizabeth acts as a head of state. This country also has a parliament

d. Right to protection

and a Prime Minister. Which among the following describes the form of government? a. Totalitarianism

integration of persons with disabilities into the mainstream of society?

b. Constitutional monarchy c. Oligarchy

a. Republic Act No. 7277

d. Aristocracy

b. Republic Act No. 7784

15.What right is violated when one opens a letter without permission from the addressee?

c. Republic Act No. 7722

a. Right to privacy of communication and correspondence. b. Right to read a letter. c. Right to open the envelope without permission. d. Right to private affairs.

a. Federalism

16.When an individual is imprisoned without proper investigation, what right is violated?

b. Socialism

a. Right to due process of law

regions united by a central government?

17.What law was passed by the Philippine Congress in 1995 which affirms the total

d. Republic Act No. 7776 18. An individual has the right to file writ of amparo before the investigation of an administrative case filed against him/ her. What fundamental right is invoked by the individual? a. Right to life, liberty and security b. Right to due process c. Right to be defended by a public attorney d. Right to self- defense

19.What is the writ of habeas corpus?

c. Private enterprise and incentives to needed investments

a. Right to information privacy

d. Rural development and agrarian reform

b. Right to accumulate data c. Right to transmit data d. Right to search for private information 20. Which principle states that no man in his country is above the law and that laws must be obeyed by all and applied to everyone – rich or poor, lowly or powerful, without fear or

22. Filipino citizens have the power to participate in the establishment or administration of government such as right to vote and be voted upon as an exercise of ____________________. a. Political rights b. Civil rights

favor?

c. Social rights

a. Rule of law

d. Economic rights

b. Rule of the majority

23. Which right is violated by wiretapping?

c. Separation of church and state d. Social justice 21.In connection with government transactions involving public interest which policy is adopted in the Constitution to assure public of accountability and transparency?

c. Claro M. Recto d. Eulogio Rodriguez 25.Who first introduced the Islamic religion to the Philippines? a. Rajah Baguinda b. Idi Amin c. Mukdum d. Abu Bakr 26. The Japanese successful invasion was climaxed by the surrender of the joint Filipino American forces on May 6, 1942. Where did this happen? a. Bataan b. Corregidor c. Capas

a. Freedom of conversation

d. Manila

b. The right to private property

27.Who is the President who is known for his “Filipino First Policy” and Austerity Program?

c. The right to privacy of communication d. Right to information on matters of public concern

a. Full public disclosure

24.In Philippine history, who was known as “The Great Dissenter”?

b. Balanced and healthful ecology

a. Camilo Osias b. Manuel Roxas

a. Diosdado Macapagal b. Carlos P. Garcia c. Manuel Roxas d. Ramon Magsaysay 28. With whom is the “Strong Republic” associated?

a. Fidel V. Ramos b. Joseph Estrada

c. The total value in pesos of services produced during the year.

b. Gloria M. Arroyo c. Corazon C. Aquino

d. Corazon Aquino

d. The total value in pesos of goods and services produced for 2 quarters.

29.When one company controls the supply of sugar, which term applies?

32. Which term refers to social aspects of sex or to socially defined roles and expectations

a. Monopoly

that are associated with sex?

their marriage customs

b. Inflation

a. Social differentiation

c. Economic stability

b. Social class

b. A student who tutors an immigrant in English

d. Equilibrium

c. Gender

30. Tomatoes as off season and so price for tomatoes are up. Which law/principle in

d. Cultural context

c. Gloria Macapagal- Arroyo

Economics explains it?

33. Which does one portray when he thinks that what is foreign is best and that what is local

a. Substitution effect

is inferior?

b. Law of cause and effect

a. Xenocentrism

c. Principle of marginal utility

b. Relativism

d. Law of supply and demand

c. Ethnocentrism

31. Which explains GNP?

d. Favoritism

a. The total value in pesos of goods and services produced during the year.

34. The statement that success often comes to those with humble beginnings would apply

b. The total value in pesos of goods produced during the year.

best to which of the following figures? a. Ramon Magsaysay

d. Joseph E. Estrada 35.Which of the following represent ethnocentric behavior? a. A tourist who lectures his foreign hosts on the "uncivilized" nature of

c. A Hispanic community group demands that public aid forms be published in English and Spanish d. A peace Corps volunteer who helps dig wells in Central Africa 36.With the promotion of social justice in mind, which does NOT belong to the group? a. Equitable access to education b. Profit sharing c. Diffusion of wealth d. Absolute right over property 37.Which part of the Visayas receives the least precipitation? a. Northern

b. Eastern

d. price ceiling

c. Western

41.The following are legitimate children, EXCEPT ___________.

d. Central 38.When the Filipino reformists asked for the assimilation of the Philippines by Spain, what

a. Those born by artificial insemination.

c. a geographical aggregate who live a common life and interact with one another d. all of the above e. none of the above

b. Those legitimated.

44.Sociologists define culture as:

did they ask for? For the Philippines to ________.

c. Those born during a valid marriage of parents.

a. a state of refinement

a. become independent from Spain

d. Those born out of a valid marriage of the parents.

b. become a province of Spain c. be independent from Spain with certain conditions d. be represented in the Spanish Cortes 39.How is the so-called colonial mentality manifested? a. Cultural relativism b. Cultural diversity c. Xenocentrism d. Ethnocentrism 40. The maximum price that can be legally changed for a good or service is called _____. a. minimum wage b. price floor c. legal wage

42.Which location should have most nearly twelve hours of daylight and twelve hours of darkness during December? A town that is located __________. a. Halfway between the Equator and the South Pole. b. Close to the Equator. c. Close to the North Pole. d. Close to the South Pole. 43.Society denotes ______________________.

b. the plastic and graphics arts c. activities such as TV sitcoms, soap operas or rock music d. the totality of meanings, values, customs, norms, ideas and symbols relative to a society e. none of the above 45.The obligation to repay a person for whom one has received a favor with undefined quantification is ___. a. Pakikisama d. utang na loob b. kusang loob e. none of these c. magandang loob

a. the totality of social organization and its complex of social relationships

46.The occurrence of graft and corruption, nepotism and favoritism in the Philippines if

b. a group of people who share a common culture

often attributed to the value of: a. Personalism d. hospitality

b. Impersonalism e. none of these c. utang na loob 47.The type of residence which permits the newly married couple to reside independently of the parents of the bride and the groom is – a. patrilocal d. neolocal residence b. matrilocal e. none of these c. bilocal 48.The movement of a person or group of persons to another place more or less for permanent residence is – a. migration b. population growth c. population change d. ecological movement e. none of these 49.____ is the material wealth given to the bride and her family before marriage. a. bride wealthd. fetish b. fosterage all of these c. dowry

50.The division of society into layers in termed as –

b. Right to be given the due process of law.

a. stratification d. life chance matrix

c. Right to be defended by the organization of teachers to which he/ she is a member.

b. status inconsistency e. none of these c. status defects 51.The most preferred form of marriage in human societies is – a. monogamy d. polyandry b. polygamy e. all of these c. bigamy 52. What is writ of habeas data? a. Right to information privacy b. Right to accumulate data c. Right to transmit data d. Right to search for private information 53.When a teacher is charged with an administrative case committed in the lawful discharge of professional duties, what right may the teacher invoke for her defense? a. Right to receive compensation for the duration of the case.

d. Right to be provided with free legal service by the appropriate office. 54.Can you be arrested without a warrant of arrest? a. No, if you are a minor. b. No, if you are more than 60 years old. c. Yes, if you were reported to have committed a crime. d. Yes, if you are in the act of committing a crime. 55.Can a person be imprisoned for debt? a. No, if he can’t pay the interest of his debt. b. Yes, if he has no property with which to pay his debt. c. No. d. Yes, if he stubbornly refuses to pay.

56.What does “presumption of innocence” mean in so far as human rights are concerned? a. A suspect is considered guilty until proven otherwise. b. A suspect has the right to remain silent.

59.Who has the power to declare the existence of a state of war?

b. It has been an independent nation ever since.

a. Chief justice

c. It has evolved from a colony to a fully independent nation.

b. President c. Senate President d. Congress

c. A suspect has the right to a legal counsel.

60.How is the crime of rape classified?

d. A suspect remains innocent until proven guilty.

a. Heinous

57.No person may be elected as President of the Philippines unless he/ she is a ____________. a. Natural born citizen b. Resident of the Philippines for at least 2 years c. At least 21 years old d. Professional 58.A bill becomes a law even if not signed by the President after ___ days. a. 60

b. Homicide c. Slander d. Malicious Mischief 61.All Filipino citizens have the right to vote and to be voted upon as a government official.

d. 30

63.Basically, the Philippine agrarian reform program is a question of _______________. a. Land distribution b. Nepotism and corruption c. Graft and corruption d. Environmental degeneration 64.Which is TRUE of the Philippines?

What is this constitutional right called?

a. It has been colonized by two European colonial powers.

a. Passive right to vote

b. It has never been united as a nation.

b. Political franchise c. Suffrage d. Electoral right

b. 40 c. 20

d. It has not achieved full independence from the very beginning.

c. There have been attempts to change its government to a parliamentary form.

62.Which is TRUE of the historical development of the Philippines?

d. It is the only Christian country in Asia.

a. It has never been an independent nation.

65.Which is TRUE of former president Corazon Aquino?

I. The first female president of the Philippines. II. The first female president in Asia. III.Described as the “icon of democracy”.

68.During the Spanish era, who was the revolutionary leader who waged a long war against the government because it did not allow proper burial for his brother? a. Macario Sakay

a. I, II, and III

b. Andres Bonifacio

b. I and II

c. Diego Silang

c. I and III

d. Francisco Dagohoy

d. I only

69.What was the first book published in the Philippines?

66.Which of the following was first to happen?

a. Del Superior Govierno

a. Aguinaldo was captured.

b. Doctrina Christiana

b. Aguinaldo declared Philippine independence.

c. Pasiong Mahal

c. Guerilla warfare against the US was initiated. d. The Philippines was ceded to the US by the Treaty of Paris. 67.In 1565, Legaspi concluded a blood compact with the chief of Bohol. Who is referred to? a. Raja Sulayman b. Sikatuna c. Lakandula d. Rajah Tupas

d. Barlaame Josaphat 70.While in the capitalist system, “to the strongest goes the spoil”, what happens in the cooperative system? a. All benefits are shared to the members proportionate to the shared capital. b. Goods are distributed among the members but benefiting mostly the policymaking body.

c. Goods are distributed among the members. d. Goods are distributed among the members but elected and appointed officials get more to comprehensive their service. 71.I rent out for Php 10, 000 per month my only building and lot to the Jesus is Lord Group which uses it exclusively for religious purposes. I am exempt from payment of ________________. a. Property tax b. Professional tax c. Income tax d. Community tax 72.Which is an unlawful act of escaping from payment of taxes? a. Evasion b. Avoidance c. Exemption d. Shifting 73.What does “capital” in economics refer to? a. Investment and loss computed

b. Outcome of business transaction

c. Favoritism

b. Communistic

c. Money/ machines invested to transact business

d. Racial discrimination

c. Capitalistic

76.The primary purpose of taxation is to _____________________.

d. Bureaucratic

d. Profit and labor spent spent for business 74.Which term refers to duties payable on goods, whether imported or exported? a. Assessment b. Tariff c. Subsidiary d. Revenue 75.The expression promdi connotes ______________________. a. Ethnocentrism b. Xenocentrism

a. Raise revenue for the support of the government. b. Reduce inequalities in wealth and incomes. c. Fortify the government against invaders. d. Make the country a leading industrialized country in the world. 77.How do you describe a typical social structure which describes a pattern through which relationships at work are ordered? a. Technological

78.When parents decide to send their children to a private school and pay their tuition fee instead of sending them to public school, they don’t get their share in their benefit derived from the taxes they pay. This is a case of ____________________. a. Social injustice b. Double jeopardy c. Double taxation d. Social inequity 10

View more...

Comments

Copyright ©2017 KUPDF Inc.
SUPPORT KUPDF